Medscape articles

126
A journey of a thousand miles begins with a single step | Yazid Jibrel Royal Medical Services OBGYN resident TWITTER@JIBREL MED STUDENTS AND RESIDENTS ARTICLES

Transcript of Medscape articles

Page 1: Medscape articles

A journey of a thousand miles begins with a single step | Yazid Jibrel Royal Medical Services OBGYN resident

TWITTER@JIBREL MED STUDENTS AND RESIDENTS ARTICLES

Page 2: Medscape articles

Table of Content

1

Table of Content Table of Content ........................................................................................................................................... 1

When did you know? .................................................................................................................................... 3

Do you have any suggestions on how to rank residency programs for the Match? .................................... 6

When I apply for residency, whom should I ask to write a letter of recommendation? Are these letters

really important? .......................................................................................................................................... 8

Don’t Ask for a Letter of Recommendation… Ask for a Strong Letter of Recommendation! ..................... 10

How Can I Distinguish Myself on Clinical Rotations? .................................................................................. 11

I am getting ready to apply for residency. Who should I ask to write my letters of recommendation, and

how should I approach them? .................................................................................................................... 13

Get the Residency You Want: Tips and Tools: USMLE Scores, Matching Formulas, and More .................. 17

How Can I Find Love During Training? ........................................................................................................ 22

Every time I work with a new resident or attending, it seems like the requirements for my clinical case

presentations change. Do you have any tips to help me predict how I should present? .......................... 24

It is hard for me as a young student to interview and examine patients. I feel like I am intruding into their

personal world without having a lot to offer them, given my limited knowledge base. Do you have any

advice? ........................................................................................................................................................ 26

How Should I Deal With Gunners? .............................................................................................................. 28

Are there any advantages to working outside medicine before you begin medical training? ................... 30

Once I start practicing in the "real world," can I still change specialties if I feel that I made a mistake? Or

will I be stuck in that career, no matter what? ........................................................................................... 31

I am starting my residency soon, and I am worried about being prepared. What resources should I bring

to the hospital? How will I know what to do when I'm alone during night call? ....................................... 37

Twelve Steps for Choosing a Specialty ........................................................................................................ 40

How Should I Choose a Medical Specialty? ................................................................................................ 42

I'm really worried that maybe I shouldn't be a doctor. What should I do? ................................................ 44

Loving What You Do .................................................................................................................................... 46

Top 10 Reasons Why I Love My Job ............................................................................................................ 48

How to be a Good Intern ............................................................................................................................ 50

Who Should Be a Doctor? ........................................................................................................................... 51

Developing Empathy in Future Physicians .................................................................................................. 52

In what order should I schedule my clinical rotations? Does it depend on the specialty I am choosing? . 56

I would like to be a research assistant or pursue my own research project. How can I find a suitable

professor to work with, and how should I approach one? ......................................................................... 58

Page 3: Medscape articles

Table of Content

2

Taking Time Off Between Medical School and Residency .......................................................................... 60

Resume Design for Success ......................................................................................................................... 62

I'm thinking about doing an elective away from my school during fourth year. What do I need to do to

arrange that? .............................................................................................................................................. 64

Can You Learn How to Break Bad News? .................................................................................................... 67

Are Board Scores Important When Applying for Residency? ..................................................................... 70

How Can I Increase My Confidence in Doing Physical Exams? ................................................................... 72

I didn't enjoy my third year of medical school and now I'm not sure if I want to do a residency anymore.

What should I do? ....................................................................................................................................... 74

How Can I Be Successful on the Job? .......................................................................................................... 76

How Do You Handle Pre-examination Stress? ............................................................................................ 78

What If I Match a Residency Program I Don't Want? ................................................................................. 80

Why Doesn't My Resident Trust Me? ......................................................................................................... 82

What is more important in practicing medicine, the theoretical knowledge of the science or the practical

intricacies and skills? ................................................................................................................................... 84

How can I remember and integrate the basic subjects of medical school with real clinical practice? ...... 86

I just finished medical school and am now an intern. How can I already be expected to teach medical

students?..................................................................................................................................................... 88

How Can I Predict My Success in the Residency Match? ............................................................................ 90

How Can I Improve My Surgical Skills? ....................................................................................................... 92

How Can I Find a Mentor? .......................................................................................................................... 94

How Should I Choose a Residency Program? .............................................................................................. 96

What Is the Best Way to Learn Procedures? Doing Them .......................................................................... 98

Do I Need To Do Research?....................................................................................................................... 102

You will Survive ......................................................................................................................................... 103

Spicing up medical education ................................................................................................................... 107

My First Day as a Doctor ........................................................................................................................... 112

Writing A Medical Case Report ................................................................................................................. 117

PDAs and Smartphones: Clinical Tools for Physicians............................................................................... 120

Page 4: Medscape articles

When did you know?

3

When did you know? Joshua Batt, Medical Student, Emergency Medicine, 10:19PM Nov 6, 2010

As I entered the gross anatomy lab within my first week of classes -- white-washed walls,

stainless steel dissection tables, and fresh white lab coats moving about the room -- I knew I was

in medical school. The feeling of making progress hit me when I saw the sea of cleanliness, the

closest thing to a sterile environment and the resting place

of our first patients.

That feeling was not to be the last. The next significant

affirmation was getting "pimped" on my first rotation. I

knew then that I was beginning a career of integrative

thinking seasoned with a little medical Trivial Pursuit.

Although not as positive as the former experience, getting

the question wrong was exactly the introduction I

expected from this caliber of education; no matter, I'm in

medical school, right where I want to be.

Such defining moments may have a make or break effect. Whether rocking a board exam or

losing a patient, colorful experiences paint our character and how we will one day practice

medicine. These seem to be the very same milestones that pave our way to graduation. But the

first is where the marathon begins.

I have often wondered what other students define as landmark experiences of entry into their

profession. Is it the endless reading assignments, the account-emptying tuition costs, suturing a

laceration or overwhelming responsibility of caring for the life of another individual? Do you

remember the feeling or event that made you feel like a soon-to-be-doctor or a physician? I

would like to know... when did you know?

Page 5: Medscape articles

A 12-step Program for Step 1?

4

A 12-step Program for Step 1? Rosalyn Plotzker, Medical Student, Family Medicine, 04:23PM Jul 16, 2010

Hello from the No Man's land, between the boards and third year. I just got back from

post-USMLE rehab.

While I was studying, I found unlimited advice about Step 1 preparation. Mysterious

experts with personal websites weighed in on everything -- what to do early second

year (buy First AID), the weeks before the test (do practice questions until you go

blind), up until Step 1 Eve when you're packing your lunch and resorting to prayer

("just stay calm"... right).

But where were the experts afterward? How do people recover from the inevitable post traumatic

stress? Should you sprint from the Prometric center to the nearest happy hour? Seclude yourself

in your living room and watch Transformers movies? Get on the first boat to the Bahamas?

You can take the med student out of med school. But how much sunburn and bad acting does it

take to get the school out of the student?

My plan was to flee the country. Then, one month before the test, my vacation plans fell through.

I was left with an empty week, and not enough student loan money to finance a trip to, well,

anywhere. I needed something to distract me, a place totally strange, preferably beautiful, and

cheap.

A month later I was at the Tuscon international airport. I signed up to volunteer in the desert.

Like a lot of students, I'm a sucker for volunteer work. First of all, it's free (or there's a small

donation, way less than a week's vacation). Second, there are no decisions. Why fumble through

a guidebook? Third, other volunteers tend to be good company. No risk of a beach full of whiney

high maintenance tourists.

Besides, I had my personal reasons for volunteering. I wanted to use my body to work, rather

than my mind. I wanted to be with people who did not know me as a med student. Lately there

had been moments when I wondered, "Has medical school taken out my soul and replaced it with

a pathology textbook?" Maybe being with strangers could answer that. And, like most med

students, I wanted the heat to evaporate memories of Goljian Lectures, First AID, Kaplan books,

USMLE World, BRS - I'll stop there.

A friend told me about No More Deaths, which is a non-profit that provides medical aid to

migrants crossing through Sonora, especially people who are severely dehydrated or injured. As

part of the desert team, I hiked through trails, looking for people who needed help. We carried

food, water, and medical supplies in our backpacks.

The boiling point for Step 1 is 108 degrees Fahrenheit. That's about when it evaporates.

My team was wonderful. The landscape was unreal. But, by the third day my body was as

exhausted as my mind. And, rather than hide my career path, I blurted out obscure medical facts

at any opportunity. "No, I haven't seen any scorpions around my tent. Hey, did you know that

scorpion bites cause pancreatitis?" So much for taking the student out of school. I couldn't stop.

Keep reading. The silver linings are much more impressive than this story's clouds.

Page 6: Medscape articles

A 12-step Program for Step 1?

5

Obviously, this group deals with a sensitive political issue, especially given the controversy over

SB1070 right now in Arizona - not to mention the general attitudes towards immigration in the

southwest. My personal feelings are that it's a good idea to help migrants if they need it, because

it's a good thing to help anyone if you can. I'll leave the politics there for now.

I will add that I was originally drawn to medicine because of health care activism work that I did

in college. When I decided to be a physician, I imagined myself working in third world

countries, volunteering for Doctors without Borders, or maybe starting my own clinic. (Who

didn't?)

I hardly had time for any of it in med school. There was barely time to get through the

coursework. I treated multiple choice questions, not patients. No wonder the soulectomy-

textbook-transplant.

But, while I was in Arizona, I remembered the potent excitement of my career: the rush of

treating someone who wouldn’t have been okay otherwise. The thrill of trying to heal someone.

It knocked the Step 1 right out of me.

Someday I will write a personal website on how to recover from the boards. It will be a bright

yellow background, and in bold black letters it will say:

Why did you apply to med school?

Okay. So, now go do that.

Page 7: Medscape articles

Do you have any suggestions on how to rank residency programs for the Match?

6

Do you have any suggestions on how to rank residency programs for the

Match?

Response from Daniel J. Egan, MD Associate Attending Physician, Department of Emergency Medicine, St. Luke's-Roosevelt Hospital Center, New York, NY

The other day I was interviewing an applicant for a position in our residency program. She told me, "I

think that I know who I want to rank at the top of my list." (This program, of course!). "It's the middle of my

list that I'm having trouble with. Do you have any suggestions?"

It's that time of year. All of you fourth-year medical students are wrapping up or have completed your

interview journey. You may have traveled all over the country. You certainly spent a small fortune for

airfare and hotels. Hopefully you accumulated some frequent-flier miles in the process. You also likely

have sat through 10-15 sessions describing programs, listened to residency directors try to convince you

that their program is the best, and had one too many "night before" social events with the residents in

each program.

So how do you make your list? Is it clear to you which program should be first? Is it less clear to you, like

my medical student, what to do with the middle of your list?

The most important thing for you to take home from this process is that your rank list should be truly that.

You should rank programs in the order in which you would like to match. Many people believe that there

is a way to outsmart the algorithm. Or maybe one program has given you feedback that makes you think

they really want you. At the end of the day, you are dealing with perhaps the most formidable time in your

development as a physician, and you should go where you want to be. So the list should be your own. It

should not be the order that your dean or faculty advisor says it should be (they have their own interests

in mind as well) but the rank order that would make you most happy.

There are a number of factors to consider as you think about each program. The first and foremost is

something that cannot be determined by a formula or list of pros and cons. The question is, Can you see

yourself as a resident in that program? As you have gone through the process, you likely spent a day

somewhere where you thought there was no way you would fit in. Additionally, there were likely others

that left you almost giddy with the feeling that it would be a perfect fit. I still recall the chairman of the

program where I trained meeting with us on the interview day. He casually spoke about a gut feeling of

whether or not the glove fits. There is something important about your gut. Frankly, it is something that

will get you through many difficult decisions in medicine, and choosing a residency is certainly one of

those.

Certain criteria should always be considered when evaluating a residency program: patient population,

tertiary care exposure, clinic experience for the specialties with outpatient components, resident

independence in regard to decision-making, electives, and where graduates go for fellowships or jobs.

Other criteria that should play a role in your decision-making include:

Page 8: Medscape articles

Do you have any suggestions on how to rank residency programs for the Match?

7

Location. This is not insignificant. There are many glamorous cities in the country, but not every city is

made for every person. Do you know anyone in the city that you are considering? Will you have any kind

of support system outside of the hospital? You will immediately establish lifelong friendships during

residency, and there is something exciting about starting over in a new place. But everyone has a

different personality, and some people find it easier if a network already exists during a very challenging

time.

Housing and salary. Is housing provided? If not, will the program's salary allow you to rent an apartment

and still have money left over for your usual expenses? (See our previous columns on the accrual of

credit card debt and how this should be avoided at all costs!)

Are the residents happy? You will hear rumors along the interview trail. Pay attention. Obviously,

rumors are rumors, but hearing that residents in a particular program are very happy is an important piece

of information.

Do you know any graduates from your medical school who are there? We all have a loyalty to our

school. We want the best for students from our school. Often, these residents will be fantastic resources

as you try to get real, honest information.

What is the academic vigor of the program? If you have a long-term goal related to academics, you

want a program that will help foster that. If faculty members are not doing research and publishing, it may

be more difficult for you to find a mentor. What are the daily or weekly conferences like? What are the

academic requirements of the residents?

Teaching environment. With shorter work hours and increased patient volumes affecting all programs,

many residents feel that the clinical teaching suffers. Ask about bedside teaching. The accrediting agency

mandates a core content of lectures and conferences, but it is really the day-to-day operations that help

you develop your clinical skills. You may best experience this with a second look. Show up on a

noninterview day. See how the residents interact with each other and their attending staff.

Finally, I'll share some advice that I got from my med school dean. He told us to put Post-It notes around

our apartments with each program listed in order. Put them somewhere that you will see every day,

preferably multiple times. Believe it or not, that gut will start talking again. Imagine yourself opening the

envelope with each program named inside, and likely you will have a different response to each one.

Looking at this list and thinking about that feeling may help you tweak the order. It certainly helped me

solidify my final list, as I grew more excited about the prospect of my first choice. And remember, the

formula should work in favor of you, the student. Don't try to play a game. Do not shorten your list

because you think you are guaranteed a spot based on program feedback. Take a phone call or email

from a program with a grain of salt. Do what YOU want to do with your list!

Page 9: Medscape articles

When I apply for residency, whom should I ask to write a letter of recommendation? Are these letters really important?

8

When I apply for residency, whom should I ask to write a letter of

recommendation? Are these letters really important?

Response from Daniel J. Egan, MD

Associate Attending Physician, Department of Emergency Medicine,

St. Luke's-Roosevelt Hospital Center, New York, NY

As someone who routinely reviews applications to our residency program, I can tell you that letters of

recommendation are very important to a successful match. The initial screening process typically involves

board scores, rotation grades, medical school grades, and Dean's letter summary statement. However,

after an applicant has made that cut, letters of recommendation play an important role in the evaluation

process and truly give insight beyond the standardization in the rest of the application.

Several approaches to letters of recommendation can be taken. The most obvious person to ask for a

letter is a mentor who knows you well. If you have worked with a mentor on a research project or have

formed a relationship outside of a rotation or the classroom, this person should write a letter on your

behalf. Most likely, his or her letter will provide more personal insight about you as an individual and

capture the reader's (i.e., the interviewer's) attention. I often pull interview questions from these letters to

ask about the mentorship or project on which a student worked.

The second most obvious “letter writer” for you should be someone involved in a position of leadership in

the clerkship of your specialty choice. Hopefully you have done well in the clerkship of your future

specialty, and you will need to have someone from the faculty speak on your behalf. If you do not include

a letter from someone in your specialty, it will be a glaring red flag to those of us reading through your

application. In fact, some specialties have requirements about a certain number of letters from staff in

your specialty. Be sure to find this out. Additionally, some specialties (such as emergency medicine)

require a specific form letter; obviously, it is important to make sure you follow these guidelines.

The question becomes, Who should write your letter within a given rotation? A letter from a resident is not

an option. A brand-new attending in his or her first year out of residency may also not be the ideal

candidate. It may be to your advantage to approach someone who is more senior within the department.

Ultimately, the content of the letter is important, but supportive statements from a chair, associate or full

professor, or residency director will be taken seriously. Additionally, many of these faculty are nationally

known, and the reader next to me may know them personally or by name, giving more credence to the

letter.

Faculty members are frequently asked to write letters. People who routinely write letters include the

medical student director for a rotation and members of the residency leadership. Those of us reading a

letter look for certain phrases and comments. If a faculty member is not familiar with letter writing, he or

she may not include these statements and it may be hard for an interview committee to interpret the

letter. A medical student clerkship director will also probably include statements or at least a summary

from multiple faculty members and give a consensus opinion. One individual faculty member can only

Page 10: Medscape articles

When I apply for residency, whom should I ask to write a letter of recommendation? Are these letters really important?

9

speak for himself or herself. Another tip: if you are unsure of a person's opinion of you, avoid asking him

or her to write a letter. Clearly, you do not want a “bad” or even mediocre letter in your file.

Some tips for deciding whom to ask:

Speak to upperclassmen who have completed the rotation. Some faculty members in your school

have letter-writing reputations.

Consider your approach to the faculty member. I have been asked explicitly, “Do you think you

would be able to write me a strong letter of support?” Many faculty will tell you that they are not

comfortable writing a letter even without this specific question, but it is always hard to say "no" to

someone. However, if you deliver this more directed question using the adjective "strong," it gives

someone an out to say "no" if they do not think they can write you an adequate letter.

Ask the residency director at your school for advice. Tell the residency director whom you are

considering asking to write letters for you. He or she may steer you in another direction if that

faculty member may not be the best option.

Speak openly with your Dean of Student Affairs. The medical school is acting in your best

interest, and the leadership wants students to be successful and match at competitive programs.

One last piece of advice: Ask for your letter as soon as possible after your clerkship, when you are still

fresh in the mind of the faculty member. Also, remember that every letter you collect does not ultimately

need to be included in your application. Better to have more than you need than not enough.

Page 11: Medscape articles

Don’t Ask for a Letter of Recommendation… Ask for a Strong Letter of Recommendation!

10

Rules of the Road

Don’t Ask for a Letter of Recommendation… Ask for a Strong Letter of

Recommendation! Joel Schofer, MD, Emergency Medicine, 08:20AM Aug 11, 2010

Students often approach their faculty and teachers and ask, "Can you write me a letter of

recommendation?"

The literal answer to this question is always yes. They "can" write you a letter of

recommendation. Anyone can.

The implied meaning of this question is, "Can you write me a STRONG letter of

recommendation?" No one wants a letter of recommendation that is anything but strong. A

mediocre or poor letter of recommendation can be the kiss of death to any application, but, sadly,

unless you ask about the strength of their letter, a less than stellar letter is what you may wind up

with. And because of the privacy of most letters of recommendation, you may never even find

out about it!

I encourage all students to ask their faculty for exactly what they want, asking, "Can you write

me a strong letter of recommendation?" (or some other similar variant). You may be surprised

by the answer to this question. Some students will be shocked when a supportive faculty

indicates that they could write a letter, but that it wouldn't be all that supportive. It is much

better to know this, however, before a weak letter is written and submitted on your behalf.

In summary, don't ask for a letter, ask for a STRONG letter. If you can't get one, find someone

else to write you the letter.

Page 12: Medscape articles

How Can I Distinguish Myself on Clinical Rotations?

11

How Can I Distinguish Myself on Clinical Rotations?

Response from Graham Walker, MD Resident, Emergency Medicine, St. Luke's-Roosevelt Hospital Center, New York, NY

Here are 10 tips to help you earn recognition as an outstanding member of the clinical team:

1. Be enthusiastic. Energy is contagious, and we all need more of it in medicine. Bring it to your

rotations! Even if you're not interested in, say, surgery, look for a particular aspect that you do find

interesting. Help out where you can. Volunteer to do more. It goes a long way.

2. Avoid complaining. As hard as you're working (and I'm sure you are!), your residents are working

even harder (or at least have more responsibilities and are balancing more spinning plates). It's fine to

join in when people are venting about how big your team's census is getting, but you won't be winning

any points by saying how rough your job is. (We all know scut sucks, but if you don't do it, your resident

will be doing it for you.)

3. Anticipate tasks that need to be done. Once you've spent a week or two on a service, you can start

to see some patterns: calling for a nutrition consult, getting social work involved, collecting a patient's list

of home medications, and requesting the old chart. Surprise your resident by thinking ahead, and you'll be

at the head of the pack.

4. Take ownership of your patients. The point of clerkships is to prepare you to be a good clinician and

a good resident. Write a good history and physical ; get your notes ready for rounds; and be the first one

to know about any changes, such as a consultant's recommendations or the medications added while

your team was post-call.

5. Help your intern or resident stay informed. Write good patient notes and provide frequent updates.

Your residents are ultimately responsible, so let them know when laboratory results are back or if the

radiologist just called with CT results, for example.

6. Ask questions. This is huge! It's your responsibility to learn patient management, so if you're not sure

why your team is ordering a test or giving a certain medicine, ask. It will show your interest and allow your

residents to do some teaching (and for most of us, it's enjoyable to pass on knowledge to someone at a

stage of training). Just make sure to raise your questions at the right time: Asking to see the ligament of

Treitz while your resident is trying to get control of a bleeding surgical field will not win you any points.

7. Don't lie. If you don't know the answer to a question, go ahead and say, "I don't know, but I will find

out." This is especially true if you're asked about patient-related information; if you forgot to listen to the

lungs, just admit it. It's much better than saying they were normal and then finding out the patient has

rales and has been fluid overloaded all night.

8. Be nice to nurses and other staff. This should be as obvious as the Golden Rule, but sometimes

students can overlook it. Be the first to introduce yourself, say "please" and "thank you," and try not to

leave a mess. Behind any excellent doctor is a bunch of excellent nurses, no doubt.

Page 13: Medscape articles

How Can I Distinguish Myself on Clinical Rotations?

12

9. Show up early. Or at least be on time. (See "Avoid complaining," above.) Your interns and residents

on the team are getting even less sleep than you, and they are still making it in!

10. Come up with your own patient assessment and treatment plan. There's no better way (besides

maybe #6, above) to show your colleagues that you're thinking and learning. You'll sometimes be wrong,

but you're supposed to be wrong, right? Otherwise there'd be no reason to go through medical school and

residency.

Page 14: Medscape articles

I am getting ready to apply for residency. Who should I ask to write my letters of recommendation, and how should I approach them?

13

I am getting ready to apply for residency. Who should I ask to write my

letters of recommendation, and how should I approach them?

Response from Geoffrey Talmon, MD Surgical Pathology Fellow, Department of Laboratory Medicine and Pathology, Mayo Clinic, Rochester, Minnesota

As a medical student applying for residency, I found it very uncomfortable to solicit letters of

recommendation (LORs). After all, I needed letters from professor-level attendings who had the potential

to affect my future. On top of that, I had heard that I was not supposed to review the letters before they

were submitted. Questions percolated through my brain, ratcheting up my stress: Who, and how, should I

ask? Would they say "yes"? Did they know me well enough to recommend me? What do these letters

even say? It was not until I was involved as a resident in recruiting other house officers that I truly

understood how LORs are used by programs and what constitutes a good one.

In several ways, the residency Match process mirrors the job application process in the business world.

LORs can be viewed as professional references. As such, they should discuss attributes that are not

explicitly outlined in other application materials. Qualities, such as work ethic, willingness to learn,

enthusiasm, and teamwork skills, give the audience (ie, interviewers and program directors) insight into

how you would function as a resident in their program.

In keeping with this purpose, the individuals who you ask should have worked with you enough to

comment on these points, hopefully in a positive manner. To carry the most weight, this person should be

a faculty member or practicing physician, not a resident. This can prove to be more difficult in certain

specialties. On an internal medicine service, a medical student may work with the same team for an entire

rotation. In my field of pathology, however, I worked with no fewer than 10 attendings during my 1-month

rotation. I ended up having to ask for recommendations from those staff members who I believed were

the best teachers and who saw glimpses of my enthusiasm.

Asking an attending physician to write an LOR can be a daunting experience, especially if it is a person

with whom you have not worked extensively, as is often the case with a department chair. Keep in mind

that most physicians who regularly deal with medical students have experience in writing LORs and

understand what is required. They often will ask for the perspectives of your supervising residents and

other faculty to garner the required information. In fact, you can explicitly suggest this to the person

writing your letter. In reviewing applications, I have seen writers mention that their letter represents a sort

of "consensus opinion," which usually makes that letter at least as strong as any other.

It is most effective to approach your potential writers during a time when you are not on their service; the

earlier in your application process, the better. I found that scheduling an appointment with the staff

physician's secretary was the most practical approach. Having a dedicated time gives you the opportunity

to discuss your goals and plans while refreshing their memory about your time working with them. At the

very least, you should provide a copy of your curriculum vitae and personal statement.

Page 15: Medscape articles

I am getting ready to apply for residency. Who should I ask to write my letters of recommendation, and how should I approach them?

14

The best LORs that I have encountered integrate specific observations from the writer with information in

the applicant's personal statement. (For example: "On a daily basis I saw the dedication to patients'

spiritual needs that Ms. Smith mentioned in her personal statement as being an important part of her

life.") The more information that you can provide (conversations, application materials, etc), the easier it

will be for your writer to write a strong LOR. Finally, agree upon a "due date" for the letter, preferably

about a month in the future.

Resist the urge to review an LOR before it is submitted; although it may be difficult to accept that this

portion of your application is outside your control, you should realize that residency programs view this

"blindedness" as a control for bias. If you are overly concerned about what someone might write, you may

want to reconsider asking that person to provide a recommendation.

Another important aspect is ensuring that your letters are submitted on time. The writers likely are

incredibly busy individuals, and they may need to be reminded of the deadline; at the same time, you do

not wish to be a pest. If you see that your LOR has not been submitted by the agreed upon date, send a

polite email asking whether the writer needs more information, and then reiterate the due date. This is

where giving your writer ample time is helpful.

In summary, the LOR is a professional reference that is meant to provide residency programs with

additional information to determine whether you will fit in with their team. Strategies, such as setting up an

appointment to converse with your potential writers, providing them with as much information as possible,

and giving them plenty of lead time, can help them write an effective letter that ultimately benefits you the

most.

Quoted from Dr. Louay (Sam’s Friend)

I think this is an important question because in this day and age the LOR are really what make or break

you. Since USMLE scores are no longer reliable to differentiate people (every one scores high), US-written

letters are what will carry you. I will try to break my answer into 3 categories to cover everything you

asked.

Content:

1. LOR has to be “excellent” otherwise it is damaging. After all, you chose which ones to send so

everyone expects them to be at least very good. A half hearted letter is disastrous.

2. The length of the letter is less important than its content. A “great” letter of 1 paragraph is better than

a “good” letter that is a 2-pager. Of course a long great letter is best.

3. The best letter for an FMG is the one that reports outstanding knowledge (better than our

residents/students), being able to fit in the US system and carry patient load, excellent communication

skills (includes language), and excellent rapport with the staff. If there is some “cool” personal info then

it is even better.

Page 16: Medscape articles

I am getting ready to apply for residency. Who should I ask to write my letters of recommendation, and how should I approach them?

15

4. It is near impossible to discover that a US letter is pre-set (cut and paste) unless the writer made

mistakes. Sometimes you find in the middle of the letter a mention of a name other than the applicant’s.

Or sometimes you find “he” instead of “she”!

5. The shorter the time of contact the less valuable the letter.

6. “Although” can be good or bad base on the context: -“Although he has an accent, his communication

skills r great” is 50-50

- “In spite of different culture and language, he was able to deal with pt well.” is also 50-50.

- “Although the medical system is different the one in Syria, he was able to deal with it like our students

did” is excellent!

-“Although he had some difficulties with English at the beginning, he did very well and improved

throughout the month” is very bad!

Source:

1. The most important factor of the letter is the writer. After all you are taking someone’s word, so

he/she better be credible. Since most PDs will not know the writer, they look at the academic titles and

leadership positions. Someone who is a professor is more credible than an assistant professor (older and

wiser). Someone who is a department chief or a chairman of medicine is highly credible. Someone who

deals with residents and students frequently (residency PD or a clerkship director) has credibility.

2. MD or DO does not make a difference. What makes a difference is academic rank/position.

3. American names are more credible than foreign names in general. A PD may wonder whether the

“foreign” name is a relative or a family friend otherwise. This will be less of a factor if the rest of your

“American” letters are consistent.

4. The institution is very important, the stronger the better. Excellent letters from the Mayo clinic or Duke

are 10 times more important than excellent letters for UTMB (if you are better than their students then it

is good to know what students are they comparing you to?).

5. LOR from outside the US are almost useless unless the program is NOT used to FMGs. Programs who

see FMGs regularly know that these are usually generic letters that have no real meaning. Programs that

seldom see a FMG may take them seriously for lack of experience with them.

6. Programs may call your letter writers if they are having a tough time making a decision how high to

rank you. If they take the trouble of doing so, at least it means they are interested.

General Strategy:

1. The more US letters the better (as long as they are very good). The Dean’s letter is enough from your

school.

Page 17: Medscape articles

I am getting ready to apply for residency. Who should I ask to write my letters of recommendation, and how should I approach them?

16

2. Inpatient medicine rotations are stronger than elective consultation. Observer as the least significant.

There is no difference in regards to type of elective (except if you have an elective where you mostly

watch, such as cath lab, then it is no good). Inpatient is generally better than outpatient because it is

more likely to be rigorous.

3. In general, send the strongest letters (10/10 or 9/10) better than 7/10 no matter what they cover, as

long as the writers are of the same caliber. A 8/10 from a famous tough guy is better than 10/10 from a

young assistant professor.

I hope this covers the topic adequately. Feel free to post follow up questions if you have any.

Page 18: Medscape articles

Get the Residency You Want: Tips and Tools: USMLE Scores, Matching Formulas, and More

17

Get the Residency You Want: Tips and Tools: USMLE Scores,

Matching Formulas, and More Introduction

Medical students constantly hear that the fourth year is the best year of medical school. That may be true,

but for most students, fourth year also means figuring out how to navigate the residency match process,

which can seem chaotic and mysterious, full of myths and speculation.

Students basically want answers to some simple questions: What is a particular program like? How

competitive am I for a given program? To how many programs should I apply? What are residency

directors really looking for? And how can I really shine in an interview?

Although there is no one answer to any of these questions, there are experts who can provide some

guidance. At the recent annual meeting of the Association of American Medical Colleges (AAMC), leaders

from 2 schools offered their ideas about how students can be successful residency applicants. The

following tips represent a summary of the insights shared by Paul Jones, MD, Associate Provost of

Student Affairs and Director of Pediatric Otolaryngology and Resident Education at Rush Medical

College, Chicago, Illinois, and Angela Nuzzarello, MD, MHPE, Associate Dean for Student Programs and

Professional Development and Assistant Professor in the Departments of Psychiatry and Medical

Education at the Feinberg School of Medicine, Northwestern University, Chicago, Illinois.

USMLE Scores, Matching Formulas, and More

Characteristics of applicants in the 2005 residency match are outlined in "Charting Outcomes in the

Match,"[1] a publication of the AAMC and the National Resident Matching Program (NRMP). The report

includes average scores on the US Medical Licensing Examination (USMLE) for applicants in the 2005

match. For some specialties, median USMLE scores and the ranges of scores were quite broad, so

students should use these data only as a gauge of their competitiveness, not as a reason to choose a

particular specialty, Dr. Jones said.

Although some residency programs do not look at applicants' scores on the Step 2 Clinical Knowledge

exam (known as the Step 2 CK), a growing number of programs do want to review these scores and are

putting greater emphasis on students' performance in this area, Dr. Jones said. Students should check

with individual programs to see whether they require Step 2 CK scores before they interview with the

program or complete their rank lists, he advised.

Dr. Nuzzarello recommended that students find a research project in the specialty area that they want to

pursue. Rather than just listing publications or job experience on a résumé, students should present

research projects by explaining their particular roles in the project, as well as describing the skills they

acquired. Thus, even without a long list of publications, students can convey their level of understanding

and proficiency in laboratory skills.

Each residency program has its own "formula" for selecting applicants, and program directors may or may

not share this formula with students. However, students should focus on trying to be themselves, Dr.

Jones and Dr. Nuzzarello agreed, so that both the interviewer and the applicant can see what the other is

really like. Despite what many students believe, residency program leaders can be just as surprised on

Match Day as the applicants; they really do not know who will be filling their residency slots, and they are

Page 19: Medscape articles

Get the Residency You Want: Tips and Tools: USMLE Scores, Matching Formulas, and More

18

eager to impress applicants just as students hope to impress them. In the end, the match process seems

to work, the speakers said, so students should trust the process.

Dr. Jones reviewed studies that looked at how some program directors use data to rank residency

candidates. One survey of program directors in orthopaedics found that the top 10 things they valued

were (in diminishing order):

the student's completion of a rotation at the program director's institution;

performance on USMLE Part 1;

rank in medical school;

formality and politeness at the interview;

personal appearance;

performance on questions of ethics at the interview;

letter of recommendation from an orthopaedic surgeon;

membership in the American Orthopaedic Association;

medical school reputation; and

the dean's letter.[2]

Dr. Nuzzarello identified red flags that dissuade residency directors:

Failure of USMLE Step 1

Failure of a core clerkship

Unexplained leave of absence

Formal disciplinary action

"Extra" requirements or "extra" time

Obtuse personal statements

Negative letters of recommendation

Falsified or exaggerated entries on the résumé

If, for whatever reason, a student took time off during medical school (even for a research rotation), he or

she should explain this fully, because some faculty will only notice that the student did not graduate in 4

years, Dr. Jones said. Any student with one of the above "red flags" in their application should be

prepared to address it during an interview. Dr. Jones added that, in his experience, students who are

unsuccessful in the residency match are unaware of how they measure up against other applicants.

Page 20: Medscape articles

Get the Residency You Want: Tips and Tools: USMLE Scores, Matching Formulas, and More

19

Students who have a red flag, but who also realize it and address it in their interviews, seem to do better,

he said.

Interview Tips and Tools

Dr. Nuzzarello provided an overview of what students need to know and do before they begin the

residency interview process. The following lists summarize her recommendations.

Interview tips:

1. Know what is unique about the program

2. Be aware of any recent changes in the program or department

3. Learn about the department's faculty and ongoing research projects

4. Use the Internet to learn about the program and hospital

5. Speak with alumni

6. Study the geography of the program, ie, remote clinical sites

7. Research the community

What to bring to the interview:

1. Directions and contact phone numbers

2. Any information you have on the program

3. Multiple copies of your CV

4. Copies of your papers and published articles

5. Portfolio, pen, pencil

6. Small suitcase, if traveling

7. Any updates to the Medical Student Performance Evaluation (MSPE) or transcript

8. An interview checklist

Tips for the interview:

1. Start with a handshake and make good eye contact.

2. Don't be afraid to smile.

3. Be aware of your body language (eye contact, position in chair).

4. Show enthusiasm for the program, no matter how interested you are.

Page 21: Medscape articles

Get the Residency You Want: Tips and Tools: USMLE Scores, Matching Formulas, and More

20

5. Answer each question and then pause; know when to stop.

6. Pay attention to interviewers' cues.

7. Be wary of programs that do not allow you to interact with residents.

Questions and topics to anticipate in the interview:

1. Tell me about yourself.

2. What would you do if... (ethical dilemmas).

3. What attracts you to this specialty?

4. Describe the most interesting case you've seen.

5. Why should we choose you over other candidates?

6. Where do you see yourself in 10 years?

7. What are the biggest issues facing physicians today?

8. What is your greatest weakness?

Questions to ask the faculty (interesting questions will make you stand out, as faculty often hear

the same questions all day):

1. Are any program changes anticipated?

2. What percentage of residency graduates have passed the specialty board exams on the first try?

3. Do you assist graduates in finding a job?

4. What do you feel are the strengths of this program?

5. What do your graduates do after residency? (what types of jobs, where they are located)

6. What is the balance of didactics and clinical work?

7. How much contact do the residents have with faculty?

8. Are there opportunities for research? (only if you are interested)

9. Are there nonclinical requirements (research, writing, administrative tasks)?

10. Is there elective time?

Questions to ask residents:

1. What aspects of the program need improvement?

2. What is one thing that you would change about this program?

Page 22: Medscape articles

Get the Residency You Want: Tips and Tools: USMLE Scores, Matching Formulas, and More

21

3. How much contact do the residents have with faculty?

4. What do residents generally do with elective time?

5. How are residents evaluated?

6. Are you receiving adequate feedback?

7. Are there international opportunities?

Questions not to ask:

1. What is the salary?

2. What is the benefit package? (This should be in the information packet.)

3. How much vacation do I get?

4. When can I start moonlighting?

Finally, always treat the residency coordinator well. Your interview starts with the first contact you have

with the program, and this is often with the coordinator. Many program directors ask the coordinators

what they thought about applicants, so it pays to be nice to them.

Also, students should follow up interviews with thank-you notes. For interviews with multiple faculty

members, a single thank-you note to the program director will suffice. In fact, this is an opportunity to

remind the interviewer about something interesting about yourself.

In summary, the presenters emphasized that this is a match process where both sides are trying to get to

know each other. Students cannot predict what program directors are looking for, so they should follow

the above recommendations but they should also try to be themselves. This way, programs can

determine which candidates are the best fit for them.

Page 23: Medscape articles

How Can I Find Love During Training?

22

How Can I Find Love During Training? Question

Developing a romantic relationship seems very difficult during medical training. Is there anything I can do

to improve my prospects?

Megan Fix, MD

Attending Physician, Maine Medical

Center, Portland, Maine

Although I wish it were not so, there is no "love potion number 9" to magically establish a relationship.

Maintaining strong relationships with others, however, is vital to your well-being during training. Can you

find someone during this time? Yes. How do you find someone? My advice is to start by looking inward

and be honest and true to yourself. Once you build a good relationship with yourself, do the same with

your friends and family, and when those are strong and healthy, you will best be able to find and hold

onto a significant other. There are many books and articles that focus on the "medical marriage" and on

nurturing an established relationship, but in the interest of space, this article focuses on finding love

during medical training.

First, it is so important to love yourself. It seems simple, but with the long hours, negativity, and grueling

demands of training, it is hard to remember. Some of us look outside ourselves to fulfill what we need

inside. We are smart, accomplished physicians but we still feel small. We look for good grades and good

comments from superiors to make us feel better inside. This need for approval can spill over into our

relationships. The fact is that you are good enough just being you. If you really believe that, then others

will, too. If they do not agree, or try to put you down, then they are not worth your time. If you are happy

with yourself first, you will have so much more to give and to gain when you are in a relationship with

another.

Once you learn to truly love yourself, the next step is to be honest about what you want and need. This is

important in residency not only because you have limited amounts of free time, but also because it is the

time when most of us are emotionally maturing. Is it important to share religious beliefs with your partner?

Is it important to share outdoor interests with them? Is it important to share interests in politics? Whatever

it is, give yourself the liberty to trust what you need. When you think of a golden weekend, what is it that

you want to do? Run, write, paint, or sit in a cafe? Whatever it is, make sure that you nurture your core

values.

Next, share these core values with others. Start with friendships. If you value cycling, for instance, find

others who do as well. Join a club. Your happiness in doing things that you love will show, and it will spill

out to those around you. If you love music, find a live music spot and bring along a medical colleague. Not

only will you strengthen your friendship, but perhaps you will find a potential significant other. Avoid going

out just to "find someone," because it often is superficial. Spend time being you and doing the things that

you enjoy, and kindred spirits will be nearby. In his handbook on residency, Dr. Lyle Victor writes, "a

Page 24: Medscape articles

How Can I Find Love During Training?

23

happy marriage involves the definition of mutually agreed upon lifestyles and goals.[1]" Establish goals for

yourself; share them with someone else; and they will be much easier to embrace in a long-term

relationship.

People often debate dating someone who is "in medicine" or "not in medicine." There are pros and cons

to each, but no right answer. Some will say, "I only meet people at the hospital" in defense of the former.

Others will say, "We always talk about medicine; I need someone who lives in the real world" to support

the latter. My advice is, again, be yourself and find people who are in line with what you like -- no matter

where they work.

What about Internet dating? I am a big advocate of this for medical residents. Three of my fellow

residents have found their partners online and are amazingly happy. Internet dating can reduce the time

you have to spend searching for someone, and it can help you find like-minded people in your area. The

only pitfall to avoid is getting too overwhelmed. Remember that it is okay to be selective with your time.

Your priority should be first to yourself and your patients, then to your dating. Your time is precious, and if

you are not connecting with someone, politely say "no, thank you."

Once you do find someone, however, whether they are in medicine or not, it is important to communicate.

Focus on the positive aspects of being a physician. Share your passions and remember why you went

into medicine in the first place. There are many negative aspects of training, but try not to dwell on those

when you are meeting someone. For example, if you had a difficult patient encounter before a date, feel

free to share what bothered you but don't dwell on the medicine aspects. Emotions are common to all of

us. Share your passions and joys.

Finally, be honest about your schedule. It is hard for anyone to understand the hours we put in during

medical training. Do not try to meet someone at 6:00 pm when you know that you may not be out until

6:45. Just as you nurtured your relationship with yourself, you must nurture your relationship with another

person. Even when you come home and are tired, remember that a healthy relationship is a 2-way street:

You give, and they give. Feel good about giving. In his article on the well-being of physician relationships,

Dr. Michael Myers states that "all relationships require care, patience, and nurturing.[2]" Start by nurturing

yourself, and that foundation will serve as fertile ground for nurturing your relationship with your partner.

Medical training is a defined period of time: for most of us, somewhere between 7 and 11 years. That is a

long time to be down on yourself for not having a relationship, or for being in the wrong one. So, do what

you can; enjoy who you are; share with those around you; and be excited about the potential of true

happiness with someone who shares the good parts of you.

Page 25: Medscape articles

Every time I work with a new resident or attending, it seems like the requirements for my clinical case presentations change. Do you have any tips to help me predict how I should present?

24

Every time I work with a new resident or attending, it seems like the

requirements for my clinical case presentations change. Do you have any

tips to help me predict how I should present?

Response from Thomas E. Robey, MD, PhD Resident, Emergency Medicine, Yale-New Haven Hospital, New Haven, Connecticut

I've been mulling over my response to this great question for several weeks, only because there are so

many potential answers but also a number of pitfalls. So when my new senior resident on an off-service

floor month gathered our team before rounds this morning, the answer jumped out at me: Just ask! In

about 5 minutes, I knew how to structure my presentations for the rest of the month.

It's natural to be apprehensive about rounds, and the guessing game about oral presentations doesn't

end when you graduate from medical school. As long as you are junior to another doctor (in effect for

your foreseeable future), you will need to adjust your presentation to accommodate your supervisor. And

isn't that true when you talk to anyone? You'd use different language, mannerisms, and sentence

structures for outreach at a grade school vs when you defend your PhD dissertation, right? In the same

way, you can expect to use different styles on surgery or medicine clerkships.

That said, I had a tough time with rounds in my third year of medical school. This was usually to my

detriment, as evaluation comments would come back to the tune of, "Thomas understands the clinical

information but struggles to present it in a concise manner on rounds." Of course, one attending

complimented me on my monologues as she chastised a senior resident trying to interrupt me. She said,

"I find Thomas' presentations refreshing. They remind me of reading Faulkner." I'm not sure whether that

was actually a compliment.

But I digress. The only contact you may have with your attending is on rounds, so it's important to make

sure you're giving the presentation that he or she wants to hear. When you start a clerkship, ask your

resident what he or she expects and what the attending prefers. Be specific! Here are some concrete

questions to ask your senior resident:

What do you expect from my clinical presentation?

Do I present my plan by problem or by system?

Am I shooting for 10 minutes, 3 minutes, or 3 sentences?

Should I expect interruptions?

What does Dr. Attending like and hate in a presentation?

Does the team generally read from notes?

How are my presentations going?

Could you meet me 15 minutes before rounds to go over my presentation?

I found that last one the most useful. Usually interns are too busy to help out before rounds, and the

senior resident is better situated to offer feedback. Are you still unsure of the general skeleton of your

Page 26: Medscape articles

Every time I work with a new resident or attending, it seems like the requirements for my clinical case presentations change. Do you have any tips to help me predict how I should present?

25

presentation? At some point in medical school, you should have received coaching for this. If you need

more, the UCSD School of Medicine has an excellent online guide for comprehensive oral case

presentations. As a medical student, you are allowed to err on the side of too much information, and as

your experience grows, your presentation will shrink. Soon enough, you'll be an expert yourself!

Page 27: Medscape articles

It is hard for me as a young student to interview and examine patients. I feel like I am intruding into their personal world without having a lot to offer them, given my limited knowledge base. Do you have any

advice?

26

Question

It is hard for me as a young student to interview and examine patients. I

feel like I am intruding into their personal world without having a lot to

offer them, given my limited knowledge base. Do you have any advice?

Response from Daniel J. Egan, MD Associate Attending Physician, Department of Emergency Medicine, St. Luke's-Roosevelt Hospital Center, New York, NY

First of all, I commend you on recognizing a level of discomfort with what you are doing. In medicine, we

tend to normalize things that would not be considered normal anywhere else. Early on in your training,

you are more likely to recognize and be uncomfortable with what the rest of the world may consider

"abnormal."

There is a unique and relatively rapid transition that occurs as you become a medical student. Most

schools have now introduced a clinical exposure to some degree early in training. In the setting where I

work, first-year students shadow me and begin interviewing patients as part of the Fundamentals of

Clinical Medicine course. Just a short time ago, these students were regular college students; suddenly,

after putting on the white coat, they have taken on a very different role.

Your statements raise several issues. The first is the basic difficulty in examining and interviewing

patients. You are suddenly prying into the history (both previous and present) of these individuals. We

frequently label patients by their diagnoses (eg, the "chest pain" in room 2; "stroke" in room 7). However,

behind each diagnosis is a person with a medical history, a social history, and family members who care

about them. You are expected to ask questions about all of those things, and in return, you have an

expectation that the patient will share his or her answers with you. But what if the patient has a question

for you about his or her illnesses or history? You should understand that it is okay to tell the patient that

you do not know. A good rule of thumb for your entire career in medicine is that making something up is

never the right thing to do. Explain that you are still in the process of learning, and you will find out and

get back to your patient.

You use an interesting word when you say you are intruding. In some ways, your interview is an intrusion

into intimate and private details of a person's life, but certainly your physical examination can be intrusive.

Reading this made me immediately remind myself that this is truly what the privilege of being a physician

(or physician in training) is all about. We put on a white coat, or merely introduce ourselves as doctors or

student doctors, and patients share very private details with us. Additionally, they allow us to examine

them to try and determine the etiology of their symptoms. It truly is an honor and a privilege, and

sometimes it takes the novice to remind us of that. You are correct. It is an intrusion. However, the bigger

picture is that the intrusion is necessary to get at the greater good, which is the successful diagnosis and

treatment of your patients.

Page 28: Medscape articles

It is hard for me as a young student to interview and examine patients. I feel like I am intruding into their personal world without having a lot to offer them, given my limited knowledge base. Do you have any

advice?

27

So, let me congratulate you. You are truly a grounded person who has not yet forgotten the humanity in

medicine. Please try to remember this feeling from time to time as you move forward in your career.

Although we play a certain role as physician, our patients are people as are we. The physician who does

not forget his or her roots, recognizes each patient as a unique individual, and remembers that this is an

honor will serve his or her patients better.

With time, your feelings will change. Some things will always seem a little strange but likely more

"normal." This is neither good nor bad, just part of the process. I hope you can remember asking this

question in the years to come.

Page 29: Medscape articles

How Should I Deal With Gunners?

28

How Should I Deal With Gunners?

How do you deal with a classmate who is a gunner? Is it something that I even need to worry about?

Response from Sara Cohen, MD

Fellow, Department of Physical Medicine and Rehabilitation, Harvard

University, Boston, Massachusetts; Fellow, Department of Physical

Medicine and Rehabilitation, VA Boston Healthcare System, Boston,

Massachusetts

During one of the first anatomy labs in medical school, I was approached by the professor. After he

examined our dissection of the abdominal cavity, he reached his gloved hand deep into the cadaver and

spoke to me for the very first time as he pointed at a blood vessel. "Sara," he said, "can you tell me the

name of this artery?"

I was pleased because I knew this one. I opened my mouth to answer the question, but before I could get

a word out, my lab partner piped up, "The gastroepiploic artery."

I was shocked. The professor had clearly addressed the question to me. He used my name and he was

looking right at me. There was no way that could have been misunderstood. Why would my lab partner

yell out the answer to a question that was obviously not directed at her before I even had a chance to

speak up?

It turned out that this was my first experience with a "gunner."

The definition of "gunner" varies depending on who you talk to, but it generally refers to an especially

competitive medical student. Every medical school class has at least 1 gunner, and usually many more

than that. Some people use the term to refer to students who study much more than average and are

especially concerned with grades. However, the term may also be used to refer to medical students who

exhibit behavior that is either borderline unethical or even blatant cheating.

Gunner behavior in the preclinical years that is borderline unethical includes (to name a few) hoarding

study materials, making comments in front of professors that are meant to make the gunner look smarter

and other classmates look unprepared, and dominating small group discussions. More malignant gunner

behavior includes ripping pages out of textbooks in the library, sending out erroneous study materials to

the class, or even cheating on examinations.

As a medical student, you will almost certainly encounter a gunner classmate at some point. (If you don't,

the gunner might be you!) Many specialties have become very competitive, which puts pressure on

students who want to match in these specialties to be at the top of the class. If a gunner is making you

feel unprepared or making it difficult for you to learn, the easiest approach is to avoid him or her in an

academic setting. If you have a study group, make sure it only includes classmates who are conducive to

your learning, rather than people who are trying to top you or make you feel insecure. Just because a

classmate is your friend, that doesn't obligate you to study with that person as well.

Page 30: Medscape articles

How Should I Deal With Gunners?

29

Another strategy is to set a good example. If you share your study materials with the class, it will

encourage others to do the same. This is a subtle way to let gunners know that information should not be

hoarded, and it is best for everyone to learn the material.

If a gunner is making class very unpleasant by dominating the lecturers' time, and the subtle hints aren't

working, the only option might be to approach him or her and explain your concerns. Most students don't

want to be perceived as gunners, so pointing this out to the student might make him or her change. When

you speak to your classmate, be nice and respectful when you address this issue -- after all, this is

someone you're going to be working with for the next several years and you don't want to make an

enemy.

Finally, if the gunner is outright cheating, it is your responsibility to report this to the appropriate

authorities.

Unfortunately, during the clinical years, gunner behavior often escalates, because grades are largely

based on evaluations from the attending physicians who are observing you. A gunner may try to be the

first person at work every day and the last to leave. He or she may try to leap in and do every available

procedure or surgery, even on a patient who belongs to another student. I've heard of students who

looked up the laboratory results on their co-student's patients to be ready if the other student dropped the

ball.

The best thing you can do in that situation is try to ignore the gunner's behavior and do the best job you

can on your own patients. If you are very diligent and knowledgeable, that will be apparent to your

attendings and residents. However, if the gunner's behavior is very disruptive, the best recourse may be

to confront him or her.

Keep in mind that all attendings and residents were once medical students, and they're often able to

recognize gunner behavior. Although the gunner may impress some attendings, others will be turned off

by attempts at showing off. You may be gratified to discover that your attending dislikes your gunner co-

student as much as you do.

Although gunners are a common occurrence in medical school, there's no reason for them to make your

life miserable unless you let them. Work hard and learn as much as you can and take gunners for what

they are: harmless distractions.

Page 31: Medscape articles

Are there any advantages to working outside medicine before you begin medical training?

30

Question

Are there any advantages to working outside medicine before you begin

medical training?

Response from Sarah Bernstein, MD

Resident, Department of Obstetrics and Gynecology, St. Luke's-

Roosevelt Hospital,

New York, NY

Before I started medical school, I worked as a waitress for a year. It was mainly a way to make ends meet

while I finished my premed requirements, but it turned out to be a very beneficial experience and one that

has definitely affected my career as a physician. Approaching a table of customers is very similar to

approaching patients. You have to read your customers; be approachable, friendly, and professional; and

then do your best to serve them well. You need to work well in adverse circumstances: If the chef forgets

their meal, a free drink is always helpful! You also need to make everything accessible and

understandable to the everyday person.

Back in those days, I often found myself describing the composition of a beurre blanc sauce; now, it is the

warning signs of preeclampsia and preterm labor that I must explain. The endurance developed by long

nights on my feet carrying trays of food up and down stairs certainly helped as well. Perhaps the most

important lesson that I learned was to always check in on your customers. A customer can be content at

one moment and then ready to throw the spaghetti at you the next, just as a patient can move from stable

to crashing in a matter of seconds.

You may ask whether I'm advocating that all medical students and residents join the food service industry

before becoming physicians. Definitely not! I do think, however, that life experiences, regardless of their

relevance to medicine, are very helpful in residency. Older residents who may have worked or traveled for

a few years generally approach residency with a very different perspective; they understand that it is a

very extreme and temporary situation. Many older residents also enter residency with a spouse, family,

and maybe even a dog. They are challenged with budgeting their time and not neglecting their loved

ones. On the plus side, they are often able to maintain a better life balance with the love and support of

their families.

No matter where you are in life, residency will be some of your most challenging years. Once you begin,

there is very little time for more than just medicine, so any life experience that you bring from the years

before training will be incredibly helpful.

Page 32: Medscape articles

Once I start practicing in the "real world," can I still change specialties if I feel that I made a mistake? Or will I be stuck in that career, no matter what?

31

Question

Once I start practicing in the "real world," can I still change specialties if

I feel that I made a mistake? Or will I be stuck in that career, no matter

what?

Response from Geoffrey A. Talmon, MD

Assistant Professor, University of Nebraska Medical Center, Omaha,

Nebraska

A friend of mine during residency had switched to pathology after being a rural family physician for many

years. When I was a frustrated intern, I often asked him why he had chosen to go back to residency; his

wife had a successful career, his children were all in elementary school, and he had already "paid his

dues" as a resident. He said that in his previous practice, he was confident in handling common problems

but often referred complex and interesting patients to specialists. Over time, he discovered that making

one difficult diagnosis was more fulfilling than treating 10 patients with hypertension. Now in a pathology

practice, he reiterates that although the move was initially hard for him and his family, he has found

tremendous professional and personal gratification in his work and does not regret the switch.

The perceived dogma is that being a physician represents a state of terminal vocational choice. Although

it is not uncommon for residents to change specialties, many doctors believe that it is "too late" to alter

their career path after entering practice, partly due to their substantial investment in training. Few (if any)

residency programs broach the topic with their trainees. Practicing physicians may be reticent to discuss

a partner's departure from their practice, even if the change had nothing to do with group dynamics. A

lack of information perpetuates the notion that doctors are stuck with their initial career choice.

The reality is that changing one's specialty is not unprecedented. In fact, as more Generation X'ers and

Millennials populate medicine (with the increased tendency to job-shop), switching specialties likely will

become even more common. Because there are few studies on this phenomenon and because mentors

may be difficult to find, you may feel as though you are going where no one has gone before.

For obvious reasons, making the decision to try a new field after being in practice is substantially more

complex than when one was a resident. Re-entering training (with the attendant staff-resident hierarchy)

may be less palatable. Relocation to a teaching hospital in a larger city may be necessary, placing strain

on spouses with careers and families with older children. Further, the switch will engender a substantial

salary decrease for several years, which may require special financial planning or a lifestyle change.

Obviously, the concept requires careful discussion with and assistance from your family.

Despite all of these potentially contentious issues, the ultimate factor to consider is your long-term

happiness. Many other physicians have made these situations work, sometimes through their own

creativity.

Page 33: Medscape articles

Once I start practicing in the "real world," can I still change specialties if I feel that I made a mistake? Or will I be stuck in that career, no matter what?

32

Which new specialty to enter may or may not be obvious to you. Perhaps you have always had a "second

choice" in the back of your mind since medical school. Regardless, it is important to consider several

issues: What is the primary source of dissatisfaction with your current specialty? Which aspect of

medicine interests you more? For example (as with my friend), you may find that you enjoy diagnostic

challenges more than urgent care or health maintenance. Perhaps emergent care provides greater

stimulation than long-term treatment of chronic illnesses. Do you want to do more procedures than your

current specialty involves? Perhaps your most serious concerns lie with work hours and scheduling.

Whatever the reason, this decision represents a substantial psychosocial investment, and it is imperative

that the new field you enter is professionally and personally satisfying.

Much like the first time you sought a residency, research is important. Spend time talking to multiple

people from different types and sizes of practices about their experience and views. Specifically,

determine the employment outlook and potential practice settings that would be available when you have

completed training. To this end, it is beneficial to get a feel for the future of the specialty: What issues will

you face in practice (economic liability, political pressures, the impact of mid-level practitioners, etc)?

Length of training is also a consideration. Starting a 1- or 2-year fellowship is obviously less complicated

than starting a 6-year surgery residency. Note that residencies may be willing to grant exemptions from a

preliminary or intern year due to your experience, so be sure to inquire.

When applying to training programs, be prepared not only to justify your decision but also to demonstrate

that you have a good comprehension of the specialty and the previously mentioned issues associated

with switching. Most program directors with whom I have spoken see the benefit and value of having an

"uber-experienced" intern on service and on the educational team. Their primary concern is to maximize

the training of the candidate and their contemporaries. Directors wish to ensure that you are making an

informed, contemplated choice and are prepared to accept being a trainee again. Be certain to explicitly

articulate your reasons for changing, and ask interviewers whether they believe that their program will

provide what you are seeking. This reinforces to each person that you understand precisely what you

want.

One benefit of being a "nontraditional resident" is that you may be exempt from the traditional application

process. This allows you to receive offers and accept a position outside of the Match, which affords a

modicum of flexibility, as it is possible to enter residency at a time other than July 1.

Once in training, continue to critically evaluate the new specialty that you have chosen. Taking into

account the special challenges that are part and parcel of being a resident, is this new path giving you

what you expected? Do not hesitate to discuss any perceived issues with your program director as soon

as possible. Their advice is just as applicable to you as to traditional first-year residents.

In summary, the process of changing specialties after residency is more complex than it is during training,

but it likely will become more commonplace in coming years. Research is the most powerful tool to help

you make informed decisions. The psychosocial and financial ramifications are not insurmountable, and

the increased satisfaction may be worth the cost.

Page 34: Medscape articles

Once I start practicing in the "real world," can I still change specialties if I feel that I made a mistake? Or will I be stuck in that career, no matter what?

33

Question

I will soon be entering my fourth year of medical school, and I just learned that I've been assigned to a

surgical rotation of which the surgeons are known to have very high standards and the failure rate for

students is very high. However, I've decided not to swap rotations. How should I prepare myself for it?

Response from Ted Melnick, MD

Attending Physician, Department of Emergency Medicine, North

Shore University Hospital, Manhasset, New York

The surgical culture is very demanding and can intimidate the unprepared student. I remember struggling

through my third-year surgical clerkship and being quite apprehensive of my required fourth-year surgical

subinternship. However, I was surprised to discover that my third-year clerkship had prepared me well for

the subinternship.

As a fourth-year student, the skills that you have gained over the previous year can make you a valued

member of the patient care team. The expectations of a final-year medical student are much different

from those of the inexperienced, timid third-year student. You have a different knowledge base; you

hopefully have some procedural skills; and you understand the workings of the hospital. The mere fact

that you are already thinking ahead and have decided to accept the challenge of a more "difficult" rotation

will put you in a better position for success.

The overwhelming amount of work expected of the surgical resident can be lightened by a helpful,

hardworking fourth-year medical student. If you are able to reduce a resident's workload, he or she will

likely think positively of you and will provide positive feedback to the clerkship director.

Here are a few guidelines to follow in order to survive, pass, and maybe even flourish on your fourth-year

surgical rotation.

First, arrive on time. Surgical rounds occur as a team. If you are late, the whole team will be waiting for

you and will be unable to round on time. In fact, if you arrive a few minutes early every day, you will stand

out as someone who understands teamwork and is there to help.

Second, don't draw negative attention to yourself. Especially in the operating room (OR), don't speak

unless spoken to. Although this sounds authoritarian, remember that surgery requires incredible dexterity

and concentration. Asking a question at an inopportune time in the OR may distract the surgeon at a

critical point in the procedure that he or she is performing. This also holds true on rounds. Time is limited

there, so the team is trying to accomplish as much as possible as efficiently as they can. If you draw

attention to yourself on rounds for anything but contributing to getting the job done, you may leave a bad

impression.

Page 35: Medscape articles

Once I start practicing in the "real world," can I still change specialties if I feel that I made a mistake? Or will I be stuck in that career, no matter what?

34

Third, help with floor work. An overworked surgical intern will sing your praises to his or her seniors if you

help with some of the floor work, allowing him or her to get home a little earlier or to spend more valued

time in the OR. You may want to read and carry The Surgical Intern Pocket Survival Guide.[1] This small

reference offers all sorts of pearls that will come in handy while taking care of routine floor work and

charting.

Fourth, familiarize yourself with the book, Surgical Recall.[2] This book is indispensable. It provides just

about any question that a surgeon has ever used to "pimp" a medical student. Find out which procedures

you will be scrubbing for the next day, and read that section beforehand. You should also review anatomy

relevant to the procedure in your first-year anatomy textbook. You can even go the extra mile by

familiarizing yourself with the procedure in a surgical textbook. To the attending and senior residents, you

will appear prepared, interested, and dedicated to the patient.

Fifth, sleep and eat whenever you can. This statement is true throughout your clinical years in medical

school. However, it is even more important on time-intensive rotations, such as surgery. A little nutrition

and rest can make you much happier and more productive.

Page 36: Medscape articles

Once I start practicing in the "real world," can I still change specialties if I feel that I made a mistake? Or will I be stuck in that career, no matter what?

35

Question

The hospital environment is so drab and depressing. How can I stay motivated when I have to spend so much time there?

Response from Daniel Egan, MD Attending Physician, Department of Emergency Medicine, St. Luke's-Roosevelt Hospital Center, New York, NY

Ahh... We have all been there. My guess is that you are in the midst of your clinical rotations and you feel

like all you do is wake up in the morning, go to the hospital, leave, sleep, and start the cycle again the

next morning. Well, my friend, we have all had those feelings and wondered how to keep on going. There

is no magic answer, but there are a couple of key elements to maintaining your sanity throughout medical

school.

The most important thing that I kept telling myself during my clinical rotations was that everyone in the

class ahead of me got through it. Additionally, many of your classmates who have already completed the

rotation that you are on have survived it. To me, that was motivating: They had endured, and so could I. It

is a little reality check as you try to keep your eyes open during hour 24 in your surgical clerkship.

I also kept telling myself throughout my medical school clerkships -- and especially residency -- that I

could do anything for a fixed period of time. In residency, we changed rotations every month, so I would

repeatedly say, "I can do anything for 4 weeks."

But you are correct: The hospital environment can seem drab and can be depressing. Therefore, it is

critical for you to maintain a life outside of the hospital. We all know that it is possible to eat, sleep, and

dream medical school. There is so much to learn and there are not enough hours in the day. However,

you need to remember that you are a person who had a life and interests before you got there.

I have written about this before, but I often remember something that a dean told us students early in our

education: Even if you spend 24 hours a day studying and trying to learn, you will still never know

everything. So make a schedule. Make an agreement with yourself. Fit outside activities into your day

(even if that means just an hour at the gym each evening). Consider it as mandatory as prerounding on

your patients. Consider it essential to your health and well-being.

There is a certain time of year (coincidentally about the same time as when you submitted your question)

that seems to predispose students and also residents to feelings of "drabness" and depression. You wake

up in the morning and it is dark outside. You leave the hospital to go home and it is dark outside. The rest

of the world has enjoyed a full day of sunlight while you roamed the halls of the hospital and only caught a

glimpse of the sun if you went into a patient's room. One of the coping mechanisms for this is to talk about

it. Go out to dinner with your classmates and share your feelings. Of course, try not to talk only about

work. Sometimes a nice, long venting session can do a lot of good; things are put in perspective and you

recognize that you are not alone.

Page 37: Medscape articles

Once I start practicing in the "real world," can I still change specialties if I feel that I made a mistake? Or will I be stuck in that career, no matter what?

36

This pattern will continue for the next several years as you go through residency, so developing

distractions and coping mechanisms now will only help later. Perhaps the most important step that you

can take is devoting at least part of your day to a nonmedical activity that you enjoy. This can provide the

motivation for getting through the day. For me, that activity was participating in a music group. Although

we didn't rehearse every day, that 1 night a week of doing something that I love helped me to get through

those very long other 6 days.

Keep a novel at your bedside. Spend even just 20 minutes before going to sleep reading something

completely unrelated to a medical journal or textbook. Make a few phone calls to friends who are in other

fields. Read the newspaper. And remind yourself regularly that there is a whole world out there that you

will soon be able to embrace.

Page 38: Medscape articles

I am starting my residency soon, and I am worried about being prepared. What resources should I bring to the hospital? How will I know what to do when I'm alone during night call?

37

Question

I am starting my residency soon, and I am worried about being prepared.

What resources should I bring to the hospital? How will I know what to

do when I'm alone during night call? Response

Sohil H. Patel, MD Sohil H. Patel, MD, Resident, St. Vincent's Hospital, New York, New York

Beginning your intern year is daunting. I know of few other times when one's level of responsibility

changes as dramatically as when a medical student becomes an intern. In some respects, the transition

requires an understanding that the first time through many new experiences, you may be slow and you

may make mistakes. There are certainly ways to minimize such mistakes and, most importantly, to learn

from them so that they are not repeated.

As a subintern, I lumbered around in my short white coat with pockets brimming full of "survival guides,"

including my personal digital assistant, a pharmacopoeia, my stethoscope, a reflex hammer, various

scraps of paper and notecards with illegible notes and phone numbers, and maybe a flattened granola

bar or two. Early in my subinternship, I was called to see a patient with abdominal pain and lower

gastrointestinal (GI) bleeding.

My thoughts immediately became as scattered as the contents of my white coat. In fact, my first 3

thoughts were: Is this an emergency? Should I be worried about colon cancer or angiodysplasia? What

are those other causes of lower GI bleeding that I memorized for the boards? As I pondered these deep

questions, my resident thankfully arrived and ordered me to check vital signs and do an examination.

Only afterward did I look through the various resources in my white coat and find at least 3 resources with

the same algorithm for dealing with GI bleeds.

So, my first piece of advice is: know what's in your white coat!

If you buy a survival guide (and I recommend the Washington University series), read through it before

you put it in your white coat. Use only 1 survival guide, and become very familiar with it. Likewise, use

either a personal digital assistant or a pharmacopoeia or an online pharmacology Website (if your hospital

has enough computers), and learn how to use whichever resource you choose.

I also found it helpful to carry a few blank note cards. Every day, I jotted down important phone numbers,

or room codes, or simple instructions on how to get things done in the hospital. My notes were a mess, so

each night I would rewrite them neatly on a new note card.

Page 39: Medscape articles

I am starting my residency soon, and I am worried about being prepared. What resources should I bring to the hospital? How will I know what to do when I'm alone during night call?

38

Other white coat essentials include a stethoscope, a (functioning) pen light, a prescription pad, your sign-

out, and at least 2 black ink pens. For internal medicine, the Mass General Pocket Medicine book is an

excellent resource. If you can fit a snack somewhere in your pockets, that always comes in handy.

Recognize Emergencies

This is probably the most important skill learned during intern year. When a nurse calls you about a

patient, particularly early in your intern year, you should try your best to always see the patient. See for

yourself what types of calls and complaints are emergent and which are non-emergent.

When you see a patient, the best way to determine whether the problem is an emergency is to talk to the

patient and get a set of vital signs. Then, compare these findings with the patient's baseline findings.

These 2 simple steps will provide critical information about a patient's neurological and cardiopulmonary

status.

Certain nursing calls and patient complaints will mandate that you see the patient. Chest pain, shortness

of breath, acute abdominal pain, hypotension, mental status change, new neurological deficit, GI bleed,

and new fever all qualify. Learn which causes and consequences of such complaints are emergent, and

direct your history, physical examination, and management accordingly. Thus, if a patient complains of

chest pain, your first objective is to rule out a myocardial infarction, pulmonary embolism, or aortic

dissection. Only after doing so should you explore whether the chest pain is actually musculoskeletal in

nature.

Finally, especially early on, accept a low threshold for calling your senior resident about all suspected

emergencies.

Arm Your Brain

The more medicine you know, the more you will enjoy and learn from your experiences as an intern.

Unfortunately, the fourth year of medical school is a well-known cause of brain atrophy. Thus, I found it

helpful to brush up before starting intern year. Review the most high-yield, commonly encountered

diseases in your specialty, and know them well (including presentation, diagnosis, and treatment). You

may also benefit from reviewing school textbooks and study aids.

At a minimum, every new intern should know how to read an electrocardiograph, interpret an x-ray, and

understand a blood gas result.

Prepare Yourself Mentally

Anxiety naturally accompanies unfamiliar situations, particularly when your decisions will affect someone

else's health. Avoid adding more stress whenever possible.

A common stressor is getting yelled at by a superior because you made a mistake. It is important to

recognize 2 components to the message.: The first is the yelling, which is mostly a reflection of your

senior's (probably somewhat dysfunctional) personality; the more this is ignored, the better. The other

component is the actual message, which often contains very useful advice to avoid making the same

mistake in the future. This second component deserves your full attention.

Other ways to avoid needless stress are as follows:

Page 40: Medscape articles

I am starting my residency soon, and I am worried about being prepared. What resources should I bring to the hospital? How will I know what to do when I'm alone during night call?

39

Try to build a good rapport with the support staff, and never engage in petty arguments with them;

Know your limits with difficult patients and recruit the help of social workers when needed;

Build good relationships with other residents;

Don't be afraid or embarrassed to ask for help from anyone (especially nurses, who are often

your best friend on a busy call night); and

Do not constantly remind yourself how terrible an intern's life is. If you direct your thoughts to your

patients and the amount of good you are doing for them, the intern year can be an incredibly

rewarding experience.

Page 41: Medscape articles

Twelve Steps for Choosing a Specialty

40

Twelve Steps for Choosing a Specialty Question

I need to declare a medical specialty before long, but I have so many conflicting feelings and thoughts

about various clinical areas. How can I make the right choice?

Response from Anne Vinsel, MS, MFA

Project Administrator, Graduate Medical Education, University of Utah

Medical Center, Salt Lake City, Utah

It's time for fourth year students to get serious about choosing their specialty area. Some of you are lucky,

and everything lines up: you know which clinical area interests you most, your board scores and

grades/letters are all in the correct range, and you have helpful professors on your side. For you, it's just a

matter of doing the paperwork on time. You can stop reading here.

But I know there are many others of you out there who aren't sure what specialty to choose. Or, you're

torn between 2 or 3 specialties. Or you know what you don't want but aren't sure what you do want. Or

you know what you want, but aren't sure if your qualifications are strong enough. Read on!

If you're stuck, here's a decision tree to follow:

1. Find or make a list of all the specialties available directly after medical school (ie, skip

fellowships).

2. Cross off the ones you definitely don't want. You don't need a string of reasons beyond the fact

that you simply can't see yourself doing it long term.

3. Perform a Google™ search with the phrase "choosing a medical specialty." When I tried it, I got

about 89,800,000 entries. Set a timer for no more than 1 hour and browse through the first

several pages. Take some of the "what specialty are you?" quizzes. If nothing else, they will give

you some ideas and possibly make you think about specialties you haven't explored. You can

safely avoid making an exact ranking of specialties at this point. Just see which specialties you

seem to be most suited to and which you should rule out.

4. Now, list several specialties you can see yourself doing long term, no more than 6.

5. Research those specialties in your institution. Go to the departments and make friends with the

residency program coordinators. If you haven't already done so and haven't rotated in the

program, arrange to shadow a faculty member for a day. Talk with 1 or 2 residents and check out

the pros and cons of the specialty. Finally, ask the program coordinator if your board scores

would be in a competitive range. Most program coordinators won't share their board score cut-off,

but they likely would tell you if your scores are within range.

6. Narrow your list to 2 or 3 specialties. Now, and only now, talk with family and friends. Tell them

you're thinking of these specialties, and get their opinions. Listen hard, and get them to articulate

the basis for their opinions.

Page 42: Medscape articles

Twelve Steps for Choosing a Specialty

41

7. Delete any reasons related to job shortages or oversupply of physicians in a specialty. You don't

need 200 jobs, you only need 1, and you should be prepared to relocate somewhere less

attractive if you choose a specialty that's overcrowded or not in much demand. Plus, demand can

change by the time you finish training.

8. Delete any reasons related to lifestyle or money, unless those concerns come from your

significant other.

9. Delete heritage reasons ("Your father is a surgeon; you should be one, too").

10. Now, write down your own pros and cons, independent of all the advice and aptitude testing and

board scores. Be honest here. If your priorities are lifestyle, having children during residency,

income, opportunities for foreign travel, or avoiding rough circumstances, then rank them

appropriately. What fascinates you, what could you be passionate about? Don't be at all logical

here.

11. But do be logical in this next step. And brutally honest with yourself: Did you barely pass the

boards? Internal medicine might not be for you, even if you really enjoy outpatient medicine. Do

you tend to avoid or dislike patient contact? Don't consider family medicine or pediatrics. Do you

have high board scores, want a benign lifestyle, but aren't very visual? Don't pick radiology.

12. If you follow all these steps, combining thoughtful reflection on what makes you happy with an

objective look at your strengths and weaknesses, one option should start singing out louder than

the others. And that's your specialty.

Note that you should take other people's views of your strengths and weaknesses into account, but not

necessarily follow their advice. Spouses are a special case because you are making a joint life together.

Still, the final decision should be yours, informed by some actual data that help you determine "the best

fit" between you and your specialty-to-be.

You can do this in a week; don't procrastinate and don't make the problem bigger than it is. If you choose

a specialty that turns out to be a bad fit, you can still change after the first year.

Be practical, but don't limit yourself. I know a physician who started medical school at age 38, one who

had to take the boards several times, one who barely passed one of her steps by 1 point, and another

who doesn't like patient care. The first one is now practicing radiology in a large private clinic, the second

is a fellow in a high-risk obstetric anesthesia program at a very prestigious academic medical center after

switching from surgery because of physical limitations, the third is a fellow in a neonatal intensive care

unit after completing a successful pediatrics residency, and the last is working for a large drug company

doing information technology, his real love.

Even if you are "nonstandard," you can find a specialty you will love and which will value you. Good luck!

Page 43: Medscape articles

How Should I Choose a Medical Specialty?

42

How Should I Choose a Medical Specialty? Question

I realize that the specialty I choose will largely define my medical career, but there are so many things to

consider. How do I decide which specialty will suit me best?

Response from Daniel J. Egan, MD

Daniel J. Egan, MD

Attending Physician, Emergency Medicine Department, St. Vincent's Hospital, New York Medical College,

New York, NY

There is something very strange about medical training. Early on, students are forced to make decisions

about their future on the basis of relatively minimal information. How should you make this decision? For

me, the choice was easy; I had always dreamed of being an emergency physician. But my decision was

also based on real-life experience, and I think that is the key element in reaching your decision.

Once you reach your third year of medical school, you really begin to practice clinical medicine. You are

given a responsibility that is uniquely different from the shadowing experiences you have had during the

preclinical years. Every 4-8 weeks, you rotate to a new specialty and learn the subject matter in great

detail. During this time, you likely will be trying to impress your residents and attendings, and you will be

studying hard so that you can do well on the shelf exam. You also need to pay attention to the subtleties

of each specialty to see if it may be a good fit for you.

Medical students base a lot of their decision on their clerkship experiences. However, most practicing

physicians will tell you that what happens during your medical school rotation is quite different from

everyday life in that specialty. For example, in the world of internal medicine, many practitioners spend

most of their time in the outpatient setting, forming long-term relationships with their patients. For a

surgeon, not every day is spent in the operating room as it is when you are the student. In obstetrics, the

labor floor and postpartum evaluations are only a small part of the practice. It is clear that what you see

as a student will help you understand what it will be like as a resident in that specialty. But one could

argue that even residency may not perfectly emulate a long-term career in that specialty.

As you try to decide, I recommend going back to the preclinical years and thinking about role models you

may have encountered. Call those people up and ask them if you can spend time with them in their

practice. Perhaps there is a subject matter that grabbed your interest and you can find a clinician in that

field. As you go through your clinical rotations, identify mentors who may be able to show you what life is

like outside the hospital as a physician in that specialty. Also, pay attention to the consultants you

encounter. Maybe radiation oncology is something you would love, but your only chance to see what they

do was when they consulted on your medicine patient with newly diagnosed spinal metastases. In my

opinion, there are many options available but our medical education exposes us to so few.

The difficult part for physicians is that when we choose a specialty, we are locked into that field for the

rest of our careers, unless we choose to pursue additional training in another field. Unlike nurses or

physician assistants, who can change career paths, we are somewhat committed to one area of medicine

for the long haul. Obviously, this needs to be a specialty you love. The content of the medicine must

excite you. You need to try to find an area in which reading about a topic or treating a patient with a

particular disease makes you crave more. Perhaps your interests are limited to the nervous system, for

Page 44: Medscape articles

How Should I Choose a Medical Specialty?

43

instance. Or, perhaps you have enjoyed all of your clinical rotations and want a field that can incorporate

all of them, like family practice or emergency medicine. Or perhaps your clinical years showed you that

interacting with patients on an everyday basis is not something you desire, so you might consider

radiology or pathology. In addition, many would argue that certain specialties have personalities that are

drawn to them. Look at the residents during your clinical rotations and see if they are people with whom

you could see yourself spending several years in training.

Finally, ask questions. Ask questions not only about the specialty itself but about life outside of work.

People love to talk about their own specialty, and frankly, many people in medicine just love to talk about

their own work in general. Try to get a sense of what life is like once residency is over. Residents have a

unique perspective on things that may be somewhat limited to the lifestyle they experience during

training. These are important issues to understand, as you will spend several years of your life in that

role, but the rest of your life involves many more years after residency training

Page 45: Medscape articles

I'm really worried that maybe I shouldn't be a doctor. What should I do?

44

Question

I'm really worried that maybe I shouldn't be a doctor. What should I do?

Response from Daniel J. Egan, MD

Associate Attending Physician, Department of Emergency Medicine,

St. Luke's-Roosevelt Hospital Center, New York, NY

Don't panic. Although that may be your first instinct after investing years of planning to become a doctor,

you should not despair. In fact, despite what you are feeling, there is a good chance that you may still

want to be a doctor but need to do some exploring and self-reflection.

Surprisingly, you may not even have a good grasp of what it means to be a doctor. Depending on where

you are in your education, you may have observed various doctors in the hospital. The reality, though, is

that for most medical specialties, life in the hospital does not truly reflect what day-to-day life will be like

later in your career.

My first piece of advice is to gather your thoughts and try to pinpoint what it is that you are feeling. Given

the rigorous academic course you pursued to get into medical school, it is unlikely that you have a

sudden distaste for the sciences. It could be the overwhelming amount of information that you are

expected to retain. It could be the feeling that you have no life beyond medicine. It could be the sense

that you have lost touch with reality and the rest of the world. But those are normal feelings, and you

should not think that you are alone if you are experiencing them.

However, if you worry that the career may not be right for you, it is time to start addressing those

concerns. Medical school is a huge financial investment. Add the residency years to that and you are also

talking about a substantial time commitment.

I recommend thinking back to what it was that drew you to medicine in the first place. Was it your

pediatrician? Was it a mentor you encountered while doing research? Was it a personal illness or that of

a family member? Take time to reflect on what brought you this far. Remove yourself from the constant

studying and memorizing -- or if you are in your clinical years, the sleepless on-call nights -- and try to

remember your earlier inspiration. You may even want to renew contact with someone who positively

influenced you.

In addition, I advise you to speak with your student affairs dean and to try to arrange some different

medical experiences. After spending an afternoon in a community physician's office, away from the day-

to-day grind, you may feel reinvigorated by the vision of yourself in that role.

If, after taking these steps, you still have doubts about practicing medicine, you do have options. All

schools have mechanisms in place to deal with students who are struggling with your exact scenario.

Schedule a meeting with your dean. You may want to take a leave of absence to explore other career

possibilities or just to get away from the med school environment for a while. Criteria for such

Page 46: Medscape articles

I'm really worried that maybe I shouldn't be a doctor. What should I do?

45

arrangements usually are strict, and the decision should not be taken lightly. Remember that any break in

your education will follow you on your paper trail (eg, on residency applications and in letters from the

dean). Ultimately, you need to do what is right for you.

I hope this helps. As I frequently say, it also helps to talk with students in classes ahead of you. Believe it

or not, you are not the first one to have these thoughts, and another student may be able to recommend a

peer who can provide some good counsel. Keep your head up, take it one day at a time, and believe that

things will work out! Good luck.

Page 47: Medscape articles

Loving What You Do

46

Loving What You Do Kendra Campbell, MD, Psychiatry/Mental Health, 07:40PM Jul 27, 2010

I’ve learned quite a few “life lessons” during my time on this planet. But one of the most

important lessons, which I’ve learned time and time again with respect to life satisfaction is to

“do what you love and love what you do.” I think that my life experiences might just be a perfect

example of the validity of this lesson. I have held a full-time job for the majority of my life, and I

frequently joke with my fellow interns about my extensive work history. Someone will bring up

a job or profession, and I find myself frequently interjecting, “yeah, I’ve done that.”

My previous job titles include (but are not limited to!) the following: tour guide,

manure shoveler, marketing assistant, Taco Bell cashier, car parker, telemarketer,

McDonalds cashier, program assistant, psychiatric technician, assistant

photographer, label factory worker, research assistant, goat milker, technical data

analyst, pizza deliverer, housekeeper, babysitter, waitress, web designer,

receptionist, Checkers cashier, IRB member, writer, dog groomer, and now

psychiatry resident and clinical assistant professor.

So, I believe that I do have a reasonable inkling of and appreciation for a wide variety of

professions. I have also experienced the spectrum of job satisfaction and reward. I’ve had jobs,

which paid very little that I absolutely adored. And I’ve had jobs, which I completely detested

that paid very well. So what is the lesson that I’ve learned over the years?

That job satisfaction is immensely critical to happiness and overall satisfaction in life. In

addition, reimbursement/salary is a variable, but is not the most important factor with regards to

job satisfaction. I’ve also learned that loving your profession has a profound impact on general

life satisfaction. If you are “doing what you love,” it’s so very much easier to “love what you

do.”

The reason that I’m even bringing up this topic is because I have friends who are currently in

their intern year of residency. They have just begun their journey into a new specialty. And while

most of my friends are happy with their work, I know some who are having their doubts about

choosing the right specialty, and choosing the right residency. I even know a few folks who are

doubting whether or not medicine is the right field for them.

I must disclose that I am somewhat delirious at the moment. My cognitive powers are diminished

due to my somewhat sleep deprived state. I had a very long call at the hospital yesterday, and I

have every intention of going to bed early tonight (in fact, hopefully shortly after writing this

post.)

I have been working incredibly hard as of late. I have found my first in-patient psychiatry

rotation and calls to be very challenging, physically and psychologically draining, and testing of

my own personal limits. However, my job satisfaction is very high at the moment. Even though I

feel drained by the end of the day, I find my work to be immensely rewarding. And I say all of

this without yet receiving even a single paycheck!

I truly love what I do, and am oh so grateful to have the privilege/honor/gift of pursuing my

dreams in life and attempting to gain some sort of enlightenment vis-a-vis my profession. I can

tell by my digression at the moment that I should actually stop writing and start eating. But I

know there was a point somewhere in my ramblings above. And I think it’s that I truly love what

Page 48: Medscape articles

Loving What You Do

47

I do, and hence don’t mind putting in the long, draining hours required to have some effect on

the patients I treat. And I challenge everyone of you reading this post right now to do the same.

Ignore the challenges. Ignore the pain. Achieve your dreams, and above all else, love what you

do!

Page 49: Medscape articles

Top 10 Reasons Why I Love My Job

48

Top 10 Reasons Why I Love My Job Kendra Campbell, MD, Psychiatry/Mental Health, 04:35PM Nov 2, 2010

I sat down at my computer just now to write a lovely post for the Ink Blot. But

then I quickly realized that I was feeling a little down, and not much like

writing. Perhaps it’s Seasonal Affective Disorder, perhaps it’s personal

stressors, or perhaps it’s just the normal ups and downs in life, but I have

feeling a little down lately. As a psychiatry resident, I feel like I can share this

information with the world at large. So, rather than focus on all that is negative,

I thought that focusing on the positives would make more sense. Plus, lists are

fun to make. So here we go...the top 10 reasons why I love my job (in no

particular order):

1. I get to help people. Sometimes.

2. Working in psychiatry involves laughing as much as you possibly can. It helps to counter the

crying. A day does not go by that I don’t laugh so hard that I find it difficult to breathe.

3. I get paid to learn.

4. I get to introduce myself as “Dr. Campbell,” and people sometimes will share their entire life

story with me.

5. I get paid to teach.

6. Dealing with insanity all day long makes me feel more sane.

7. I don’t have to wear a white coat.

8. I get paid to talk to people.

9. I can hear people say things like, "I am the son of God, and I have alligator feet which help me

swim, and can change my skin like a chameleon to help me blend in, which is a skill that I’m

using to win today’s elections," and not bat an eyelash.

10. I get paid to dance with people.

There, I feel a little better now!

Page 50: Medscape articles

Acting Like a Doctor

49

Acting Like a Doctor Kendra Campbell, MD, Psychiatry/Mental Health, 04:18PM Jul 5, 2010

I "successfully" made it through my first week of residency without any major

catastrophes. However, that is not to say that it's been a walk in the park. I still feel

completely lost. I have no idea how to navigate the electronic medical records system.

I keep forgetting the secret code to the employee bathroom. And I really have no idea

what I'm doing.

To make matters worse, I am rotating at a psychiatric hospital. I worked as a

psychiatric technician in a psych hospital many years ago. And I rotated as a medical

student at a psych hospital, but that was about two years ago. Since then, I was working as a

medical student in various medical rotations. I haven't had to take a psychiatric history since

then, and although I've dealt with patients who had psychiatric illnesses, my focus for the past

two years has been on their medical needs.

Although I have those two magical letters after my name (MD), I really don't feel any different

right now than I did a few months ago as a medical student. Let me give three examples:

1. As a psychiatric technician, my primary duties involved interacting with the patients, and also

maintaing their safety. When a patient began to escalate, or was about to attack the psychiatrist, I

was the one who jumped in to put "hands on" the patient, and I assisted in applying the restraints,

if necessary. On my second day of residency, a patient started escalating, and she was becoming

physcially agressive towards my attending. My first instinct was to jump in and intervene.

However, my attending quickly called out for a Behavioral Health Associate (the equivalent of a

psych tech), and my primary job was to alert the nurse to prepare a PRN (medication to calm the

patient down).

2. Another moment where I realized that my mind was not in "doctor mode" was when it became

apparent that a patient needed to have an order written for a medication. My instinct was to run

and alert the doctor to write the script. And then I realized that I WAS THE DOCTOR. Man, was

that mind-bending!

3. Later on that same day, my attending asked me to write an order for another patient. However,

because I still couldn't figure out how to use the computer system to write an order, I asked the

nurse what to do. She quickly told me that the patient did not need that medication, and decided

that the patient needed an EKG instead. As the doctor, it was my duty to stand by my/our

decision, and insist on placing the order. However, I knew that the nurse had many years under

her belt, and it was my second day as a doctor on the unit. Who the heck was I to tell HER what

to do, even if I was the doctor?!

I'm looking forward to my second week of residency. But I'm also frightened and dumbfounded

about introducing myself as "Dr. Campbell," when I don't feel anything like a doctor! I'm

supposed to "act like a doctor," but that's just it...I feel like it's all an act, and I was a very poor

drama student in high school. Any suggestions from the audience?

Page 51: Medscape articles

How to be a Good Intern

50

How to be a Good Intern Kendra Campbell, MD, Psychiatry/Mental Health, 12:34PM Aug 8, 2010

I have only completed the first month of my residency, but I’ve already learned so very much!

While I am still a “green intern,” I do feel like I have picked up some pointers here and there on

how to be a good intern, which I could share. Many of these tips are equally applicable to being a

med student!

1. Show up early, stay late. Really, this is self pretty explanatory. If you take the extra time to

pre-round on your patients, or stay late to check on your patient, you will learn more and be a

better intern!

2. Know as much as possible about your patients. This is definitely applicable to med students,

as well. The more you know, the more you can help your patient, and it looks like you really

know what you’re doing!

3. Be BFFs with the nurses. I’ve mentioned this many times, but I’m mentioning it again because

it’s just so incredibly important!

4. Do things without having to be asked. Figure out what needs to be done, and do it (unless it’s

a questionable procedure). Nothing makes a resident/attending happier than to find out that their

work was already done for them!

5. Don’t do anything you feel uncomfortable doing! In contrast to the above tip, NEVER

perform a procedure or do anything you don’t feel equipped to do.

6. Always ask questions and inform your higher ups, even if you don’t think it’s a serious issue.

The other day I had a patient who was slightly hyponatremic. I knew it wasn’t a serious issue,

and I almost didn’t even tell my attending, because it was such a minor finding. But I then heard

the words of our departmental chair in my mind, “You will never get in trouble for asking

questions or asking for help, but you WILL get in trouble for NOT asking

questions or not asking for help!” My attending actually ended up thanking me for

notifying him.

7. Have quick access to medical information. This seems rather obvious to me, but

some folks forget it. Download medical apps onto your phone or PDA. Keep

guides and reference books handy so that you can access information very quickly.

My attending asked me a question the other day, but then the nurse distracted him

for a moment. In that quick moment, I whipped out my iPhone and found the

answer. He was impressed when he turned around and I gave him a long reply. (It’s not cheating,

it’s being smart!) Plus, you will be a better doctor to your patients! As the saying goes, it’s not

what you know that’s important, it’s knowing when to look it up!

8. Teach your medical student well! At my residency program, all residents are given the title of

"Clinical Assistant Instructors." Us interns are the closest to medical students in the hospital food

chain. We have the responsibility to teach the students, and because we were not long ago med

students ourselves, we should remember how important it is to pay it forward and pass the

knowledge on.

I hope these tips are helpful to some of you med students and interns out there. Feel free to add

to the list!

Page 52: Medscape articles

Who Should Be a Doctor?

51

Who Should Be a Doctor? Kendra Campbell, MD, Psychiatry/Mental Health, 06:17PM Jun 2, 2010

There are many aspiring doctors in the world, who are very interested in pursuing medical

school, but aren't sure if they have what it takes, or if they should really be a doctor at all. I can

understand their feelings of confusion and apprehension at taking the plunge into medical school,

as I was once in those same shoes. It took me over six years after obtaining my bachelor's degree

to decide to finally go to medical school.

Now that I have successfully made my way through four years of med school, and am about to

begin my psychiatry residency, I feel like I have a good idea of what it takes to be a doctor.

While it's possible that my perspective might change during residency, I think I currently have a

good idea what it takes to make it through med school, and I have a good idea who will

ultimately end up as a successful and happy physician.

Let me first address who I think would not make a good physician. In my opinion, you should

not be a doctor if your motivation is to:

1. Make a lot of money.

2. Be famous.

3. Make your parents happy, and so they'll stop bothering you.

4. Feed your God complex.

Let me now put a positive spin on things. What does it actually take to become a

successful doctor?

1. Reasonable intelligence. No, you do not have to be a member ofMensa, but

yes, you need to be able to process and understand large quantities of

information.

2. Dedication and hard work. These are perhaps the most important characteristics. Medical

school and residency are a very long series of hoops. If you aren't dedicated to the path, you will

find yourself on the ground, rather than in the air.

3. Compassion. This might seem rather obvious, but it's still very true, and very necessary.

4. That you love what you do. In other words, it's important that you choose medicine for reasons

other than the ones listed above. Actually, I believe this rule applies to all fields and career

paths.

5. Creativity. Albert Einstein said it best: "I am enough of an artist to draw freely upon my

imagination. Imagination is more important than knowledge. Knowledge is limited. Imagination

encircles the world."

I'm sure there are a lot of people who disagree with my ideas and assumptions. So, I'd like to

hear your ideas. Who do YOU think should be a doctor?

Page 53: Medscape articles

Developing Empathy in Future Physicians

52

Developing Empathy in Future Physicians Introduction

One has only to read memoirs of recent medical graduates, such as Melvin Konner's Becoming a

Doctor, [1] Shem Samuel's House of God, [2] and Robert Marrion's Learning to Play God, [3] to marvel at the

anecdotes and stories of tired interns and residents to whom patients become nameless, faceless

obstacles or interruptions of much-desired sleep. This, coupled with patients' fear that when they are most

vulnerable they will be in the hands of an albeit competent but callous physician, has led to increased

focus on the issue of empathy in the medical profession.

School Ties

Some of the factors that seem to drive empathy to a low point in physicians-in-training result from 2

traditional views doctors absorb during medical school. One is the "care:cure dilemma." This usually

asserts that it is doctors who do the curing and nurses who do the caring. The other is the traditional

format of interviewing and the social ethos of medical training and medical practice, which stress clinical

detachment.

Indeed, Nisker[4] humorously refers to medical education as the Yellow Brick Road, where "from the time

the medical school's acceptance letter is opened, students set off in pursuit of the attributes of the good

physician: intelligence, compassion and courage." Nisker adds:

The students already possess these traits, just as the Scarecrow, Tin Man and Lion did before they set

out for Oz, but they may dissolve in education systems that still decree that the initiation to medicine

involve tons of tutored words and consuming call schedules.

The rigors of medical school can take other tolls. Factors that can drain a doctor-hopeful include:

Overexhaustion

Insufficient skills to deal emotionally

Organizational demands or limitations

Lack of example or correction of behavior

Scientists who have studied the transformation in medical students during their clerkships tell us that in a

psychological attempt to deal with first encounters of illness, death, pain, and suffering, students may

swing from empathy to overidentification, or objectivity to avoidance. Those who study the problem call

for a vertical support structure for coping with the emotional demands and reactions to what one sees and

hears in the hospital. Spiro

[5]

puts it eloquently when he speaks of the need for "conversations about experiences, discussions of

patients and their human stories, more leisure and unstructured contemplation of the humanities.

Physicians need rhetoric as much as knowledge, and they need stories as much as journals if they are to

be more empathetic than computers."

Page 54: Medscape articles

Developing Empathy in Future Physicians

53

Re-education

A number of investigators have suggested that reading medical prose or good literature on death,

sickness, dying, and the doctor-patient relationship can help one to sort out one's own feelings. An

excellent Web site in this regard that I can recommend is http://www.medicalprose.com/. Another useful

idea is to rent a video after exams that relates to a class you took. For example, in our biochemistry class,

after studying the lipid disorders, we watched the movie Lorenzo's

Oil (see http://www.teachwithmovies.org/guides/lorenzos-oil.html for a summary of the film and some

ideas on using it in your classes). It was heartening to see tears in the eyes of several future doctors as

they watched the parents and child fight the disease. It brought new feelings to the bland text and

equation descriptions of the disease in our biochemistry book.

Finally, in some residency programs, first-day residents are assigned a clinical scenario to act out and are

instructed to pose as patients and approach their department. The nurses at the triage usually do not

know who they are and they get treated just as any other patient would who is visiting the department.

For most of them, it is an eye-opening experience.

Call to Compassion

Patients in pain, suffering, and illness seek relief from medicine and the healthcare system, but they also

seek human comfort, understanding, and empathy. Most of the measures can be easily assimilated into

the structure of most medical schools. In all fairness to the overworked, sleep-starved drones of the

medical world -- the interns and residents -- they are there when you need them. Their compassion is

spoken with action and response to call. As Spiro has so eloquently stated, "Computed tomographic

scans offer no compassion and magnetic resonance imaging has no human face. Only men and women

are capable of empathy."[6]

Factors shown to enhance empathy in medical students and future physicians include:

A liberal and humanistic education

Exposure to literature and philosophy

Training in interpersonal communication skills

Workshops on attentive listening, sensitivity, and empathy

Early and continued exposure to the hospital environment and patients

Experience as a patient -- real or simulated

Page 55: Medscape articles

Developing Empathy in Future Physicians

54

Question

I feel like my residents and attendings always expect more of me, which causes considerable stress. What can I do to reduce this constant feeling of pressure?

Response from Geoffrey Talmon, MD Surgical Pathology Fellow, Department of Laboratory Medicine and Pathology, Mayo Clinic, Rochester, Minnesota

As a third-year medical student, I was assigned to an obstetrics rotation off campus with a group of

private physicians. They were notorious for pushing their medical students hard, and consequently I was

dreading those 6 weeks. During the first half of the rotation, we were absolutely swamped. Every delivery

room was always filled, and we always seemed to have a very long inpatient census to round on each

morning. Added to this, it seemed as though I did nothing right. My attending would tear my notes to

shreds as he critiqued them, I never seemed to answer any question correctly, and in surgery I was

apparently incapable of holding a retractor. This was a level of pressure that I had never experienced

before. I certainly did not talk about it; weren't medical students supposed to be able to deal with all of this

with a smile on their face?

One night, I was in the doctor's lounge while on call, and my attending asked me how I was doing. I must

have been really tired because I actually told him about all of my anxiety. He smiled and admitted that he

tended to ride medical students hard, but he said that his expectations were simple: See your patients in

the morning and write a note, participate in rounds, and show that you were reading about your patients.

All of the other criticisms were in an attempt to make me improve and be more efficient. He said that I was

at least meeting if not exceeding all of his expectations. After hearing this, the pressure that I felt while on

service was reduced incredibly, and I actually enjoyed the rotation.

Handling pressure from superiors is a constant in life. From school to business to sports, it is something

that everyone must find a way to deal with in order to succeed. It is one of the skills that employers often

use to determine who is promoted and who cannot handle their current position. As such, a number of

books, presentations, and symposia have been developed to provide advice on handling pressure.

Medicine is no exception. On top of the stress that comes with making decisions that affect patients' lives,

perceived demands from residents and attendings create more anxiety. The hierarchy of academic

medicine makes coping with these stressors even more complex. Attending physicians' diverse

backgrounds engender numerous styles of teaching and interaction with subordinates, as well as differing

levels of patience with student doctors. As a result, determining what is wanted of a trainee can seem like

aiming at a moving target.

On top of all this, medical students often have high self-expectations and a desire to please. This can

make handling pressure even more difficult. When I discussed high-pressure rotations with my friends

during medical school, many of them would say things such as, "Well, at least it only lasts for a month."

This thought offered little solace as I was often very concerned with my evaluation or the potential for a

Page 56: Medscape articles

Developing Empathy in Future Physicians

55

letter of recommendation. For those like me, how does one deal with pressure from more senior

physicians?

To start, it is important to recognize that "pressure" does not equal abuse. The days are over when

medical students had to put up with verbal abuse from supervising physicians. Even if an attending is

glaringly upset with your performance, there is no excuse for name-calling, swearing, or insults. In such

instances, nearly every medical school has mechanisms for reporting unacceptable behavior, often

starting with the rotation director.

It is also important to determine the source of your pressure; is it coming from your superiors or from

inside yourself? Almost all medical students are driven to overachieve, so we often set unrealistic

standards. We feel that we need to answer every question correctly, present every patient perfectly, write

the best clinic notes, etc. Whenever we fail at one of these self-imposed goals, we experience internal

anxiety. Then, the more that is asked of us, the more pressure we feel. In hindsight, I realize that most of

the time when I felt "under pressure" while on service, it was of my own making. Two nonthreatening

ways to determine the source of your pressure is to ask other students who were on the rotation before

you what the attending expected of them, or talk to your residents about your performance.

Similarly, you may be able to reduce the pressure from senior doctors if you know precisely what they

want from you. Thinking back, some of the best attendings and experiences I had on service came when I

was told at the very beginning of my rotation what my responsibilities were. I did not have to "guess" what

I was supposed to be doing, which often removed a large amount of stress from my day-to-day activities.

It is almost never incorrect to ask your supervisors this question, although it is advisable to have such

discussions in private and at a less hectic time than during the middle of rounds or a surgery.

So, what should you do when there is true pressure from your supervisors to perform, either due to the

volume of work or high expectations? Unfortunately, these situations are occasionally unavoidable, and

they often occur as a student gains more responsibility (eg, a new fourth-year medical student or a new

first-year resident). In most cases, the goal is to make you more efficient -- shorter notes, more focused

histories and physical exams, and so on. Again, the key to handling these situations is talking with your

senior physicians. Ask them for their assessment of your performance and for hints and/or advice on how

to improve.

The overriding theme in all of these tactics is communication. When you, as the student doctor, feel that

the weight of demands placed upon you by senior physicians is causing excessive anxiety, talk to them

about it. Ask them what is expected of you, whether you are meeting those expectations, and what you

could do to improve. Because the goal of academic medicine is to turn you into a better physician, such

questions are not inappropriate, and they can help reduce the amount of stress and pressure that you are

feeling.

Page 57: Medscape articles

In what order should I schedule my clinical rotations? Does it depend on the specialty I am choosing?

56

Question

In what order should I schedule my clinical rotations? Does it depend on

the specialty I am choosing? Response from the Expert

Response from Daniel Egan, MD

Attending Physician, Department of

Emergency Medicine, St. Luke's-

Roosevelt Hospital Center, New York,

New York

Medical schools take varied approaches to scheduling students during the clinical years, with different

amounts of elective time and mandatory rotations. For instance, one school may require a month in the

intensive care unit or in the emergency department, while another may require a rotation in geriatrics.

Despite these variations, certain general principles do apply. If you are certain about the specialty you

plan to pursue, you should avoid scheduling a rotation in that clinical area first. During your earliest

rotations, you will be learning basic medical practices, how to pre-round and round, how to write a note,

and how to use the computer system. The best place for you to experience that learning curve is not in

the specialty you intend to enter. However, it may make sense to do a so-called "difficult" rotation first,

such as surgery, if you are not going to pursue a residency in that clinical area. More difficult rotations can

accelerate the process of learning the ways of the hospital as well as decreasing your anxiety level (and

requirement for sleep) as the year moves forward.

Certain disciplines require special considerations. For example, students generally do not get formal

exposure to specialties such as emergency medicine or ophthalmology until their fourth year. If you plan

to enter one of these specialties, you should schedule a rotation during your third year so that you can

meet people, possibly arrange an audition elective, and most importantly, make sure that you actually

enjoy the specialty. Another unusual situation arises with specialties that require an early match. If you

are considering one of these areas, you should schedule a subinternship rotation or "audition" early in the

fourth year so that a potential residency program can see who you are and how you work.

Aside from these special cases, let's consider a student applying to one of the more generalized

specialties, such as surgery or pediatrics. Most advisors and mentors would advocate doing an elective

rotation at an away site early in your fourth year. Similarly, a subinternship at your home institution also

should be scheduled early in the year. This will give you a chance to really perform like an intern,

significantly integrate yourself into the team, and secure that letter of recommendation.

In fact, the role of the fourth year student is quite different from that of a third-year student. In a

subinternship, you will get admissions from the emergency department. You will have direct patient care

responsibilities. Residency programs will likely want to read your evaluation from the subinternship as part

of your application packet. As a result, it may be advantageous to do this rotation at your home institution

Page 58: Medscape articles

In what order should I schedule my clinical rotations? Does it depend on the specialty I am choosing?

57

before doing an away elective. You already know the system at your home institution and won't struggle

with mundane tasks that could detract from your performance, such as ordering tests, logging onto the

computers, or finding the radiology department.

Finally, take full advantage of your last year of medical school. There are few times during the rest of your

life when you will have the chance to learn something truly unique. Your residency will include some

elective time, but at that point, you will already be focused on your specialty. So, during your fourth year,

find something different to explore. Consider doing a rotation abroad. You might also want to schedule a

rotation outside your chosen specialty at a hospital where you would like to do your residency. This would

give you a chance to see the city and the hospital without having to worry about being at the top of your

game 24 hours a day.

And don't forget to speak with classmates at your own school. Senior students can be a valuable

resource and can tell you what worked well for them and what did not.

Page 59: Medscape articles

I would like to be a research assistant or pursue my own research project. How can I find a suitable professor to work with, and how should I approach one?

58

Question

I would like to be a research assistant or pursue my own research

project. How can I find a suitable professor to work with, and how

should I approach one? Response from Daniel Egan, MD

Attending Physician, Department of

Emergency Medicine, St. Luke's-

Roosevelt Hospital Center, New York,

NY

Research experience has become an important part of your development as a medical student and your

attractiveness as a residency applicant. Many of the most competitive residency programs now have the

luxury of selecting from applicants who have all of the desired elements (good grades, strong

recommendation letters, research experience, and possibly even a publication). So, the more you can do

to make yourself one of those candidates, the better.

Ideally, you should look for a mentor who can help you develop your own research project. More

realistically, you may find a mentor who is already working on a project but who would welcome the help

of a medical student.

The first question to ask yourself revolves around your area of interest. Do you have a particular specialty

in mind for residency? If you are thinking about dermatology or orthopaedics, for instance, you may be

expected to have worked specifically in those areas. Do you have research experience? Have you ever

worked in a lab? Mentors may be most willing to take on a student who requires minimal teaching and

supervising, but who will reliably get the work done. Of course, faculty who work primarily in the lab or

have a significant amount of protected research time likely will be more available for teaching and

mentoring, which would certainly be an ideal situation. In that case, you will likely forge a relationship that

is quite deep and have a worthwhile educational experience, in addition to enlisting someone who can

write a personal recommendation letter down the road.

In order to identify and ultimately meet the right person, you have several options. I would begin with your

medical school's administration. Many schools have a research office geared toward engaging medical

students in research. They may have a database of faculty members who are looking for student

research assistants. When I was in medical school, the student research office published a listing of

faculty members looking for help on specific projects. Additionally, the director of your research office may

be able to help guide you in finding the best experience depending on your long-term goals and time

availability.

The next option would be to investigate the department in which you are most interested in working. In

many of the larger academic centers, each department has faculty members designated as the research

Page 60: Medscape articles

I would like to be a research assistant or pursue my own research project. How can I find a suitable professor to work with, and how should I approach one?

59

directors. They would likely be the best individuals to approach first. They will be in tune with the current

activities and opportunities in their department, and can link you with other faculty members. This is also

an opportunity to meet a faculty member in your future specialty who can act as a mentor and guide as

you go through the process of applying for residency.

Finally, if you are left without clearly identified research faculty or administrators, you can go directly to the

source. Identify the names of faculty members in your department of interest and then head for the

Internet. Search www.pubmed.org for articles written by faculty. You could even perform broader

searches to see whether certain faculty are recognized for particular areas of interest. Once you have

identified a possible mentor, email him or her. I do not think that you need to attach your curriculum vitae

or list any other experience, but merely describe your interest and ask to set up a face-to-face meeting.

Most medical school faculty are happy to hear from students; this is the reason that we have chosen to

work in an academic environment. I would engage the faculty member by asking whether you can meet to

discuss his or her research on "x" topic, and then ask during your meeting whether there is any potential

area for your involvement. If the faculty member does not have opportunities, he or she may be able to

refer you to someone else.

Once you have identified a person or project, don't forget about your primary goals. For most students,

this means publication. Ideally, a published paper on your curriculum vitae when you are applying to

residency will show motivation and academic success. However, an abstract presentation at a regional or

national conference would also be impressive. Be honest with your potential mentor. Walk the fine line of

expressing interest in learning about the fundamentals of research and the project, while also

acknowledging that part of your goal is a tangible result (ie, an abstract or paper). Some projects take

years and are not realistic for your involvement other than the academic exercise. Others may involve

short chart or record reviews that may work much better for the summer or your research elective.

In summary, do some investigating and you will likely find a mentor. Do not be afraid to send an email. Be

persistent because there are always projects out there. Finally, be realistic about your own goals and time

constraints, because you still have to do well in school.

Page 61: Medscape articles

Taking Time Off Between Medical School and Residency

60

Taking Time Off Between Medical School and Residency

Tradition and the inherent design of medical education prevent most new medical graduates from even

considering taking a year off before starting their residency. In fact, the very idea is perceived negatively

by many in the medical community who consider it a detrimental interruption in training, a sign of

weakness, and an outright risk to one's career.

As such, the "year off" has become less an opportunity for travel and leisure, and more an opportunity to

improve your resume, especially for graduates applying to competitive residency programs. Of course,

there are those graduates who have unforeseen personal circumstances who can justify taking a year off

to residency selection committees. But for everyone else choosing this path, it is hardly a year "off" at all.

It may be even more time-consuming and stressful than the years spent in medical school. This is a year

that requires personal initiative and meticulous planning, and it puts extra pressure on the applicant who

is now arguably under greater scrutiny than the rest of the applicant pool.

So, for those graduates who have a year "off" due to an unsuccessful match or because they opted out of

their first match opportunity to improve their applications, this article will provide some basic suggestions

and directions. Even those taking a break for other reasons should afford some time to add to their

resume, because it is difficult to "sell" that decision to residency programs, however sympathetic the

program leaders might be. This is especially true for competitive programs that have an ample pool of

good applicants.

To start your journey, it is safe to begin by formulating a plan with set goals in mind. You have to cover

various areas where you need improvement, starting with those areas you deem most important to your

application. Here, one cannot overemphasize the cliché: Know thyself! Know your areas of weakness and

target them head on. Look at yourself objectively and ask others to help you.

Returning applicants can benefit by contacting the people who provided references the first time around.

This is a great idea, especially if you plan to use their letters again. Approaching program directors and

other selection committee members in person is invaluable. If you are a returning applicant, do not feel

embarrassed to approach such members, as ultimately they will be judging you again. The key is to

obtain as much constructive feedback as possible before you formulate a plan.

Areas that warrant particular attention are academics and research. Academics include grades, clinical

elective work, rounds presentations, teaching, awards, and graduate studies. An applicant must decide

which areas require improvement and whether it is feasible to improve within the short time frame.

Choosing to set up more electives can be an efficient choice. This shows your dedication to the program

and may also provide new reference letters.

Choosing electives from other specialties can widen your breadth of knowledge and possibly set you up

for a good back up-plan if you do not match to your program of choice. Consider informal clinical

experiences such as observerships. These can be easier to set up and just as effective at times, and they

may lead to opportunities for rounds presentations where you can demonstrate your qualities.

Some students may enroll in a master's program. There are many 1-year programs in areas suited to any

medical specialty, such as a master's in clinical epidemiology. If committing to a full-time classroom

setting does not fit with your plan, there are many online correspondence programs offering the same

degrees. They require less commitment and can be extended for completion during or after your

residency. This can allow for time to tackle other potential deficiencies in your application.

Page 62: Medscape articles

Taking Time Off Between Medical School and Residency

61

Research is an area that many people can improve on. Medicine does not give much time to pursue

independent projects during the school year, and those without the initiative or foresight to set up projects

in the summer months often fall behind in this area. There are many simple ways to add to your research

portfolio. If your goal is merely to publish something quickly, look into doing a case report. Many

physicians keep a pile of interesting cases to publish but just never have the time. They welcome help

from eager individuals to do the job for them, which often secures your name as first author.

If you haven't found cases to publish, why not start an independent review article on a topic of your

interest? These are highly regarded, as you become somewhat of an expert on the topic at hand. The

peer review process for publishing in most journals can take a long time, so start such a project early. If

you are pressed for time, try writing for non peer-reviewed medical journals. There are many you can find

on the Internet that allow various forms of exposition, including letters, reviews, opinion pieces, and more.

If you still feel the above is inadequate to account for your lack of research, try to find a formal research

project at an academic center or institution. Find an area that is relevant to your residency of interest. This

will require approaching many facilitators to make sure you get what you want. Do not waste time here

because medical students can quickly pick these up as summer research. Another option that is

becoming more widely available is to do a research fellowship. These are postgraduate positions usually

offered to those having already completed a residency, but they can be done before residency as well.

Formal fellowships exist that you may apply for, but you can also design something on your own after

speaking to a facilitator who is willing and has the available funding at hand.

Needless to say, these are paid positions and as such lighten the financial burden of taking a year off.

Paying back loans can sometimes be deferred, especially if you are enrolled in an academic institution.

Research, aside from offering an opportunity to publish, also offers a chance to travel and present your

work at various conferences. Look for positions that offer this as they can add an extra few crucial lines to

your application. A talk at an international conference in the field of interest is a priceless addition to

anyone's application and will set you apart.

Above, we provide options to tackle the major areas in a residency application. It is crucial, however, to

work on other areas besides academics and research at the same time. Remember that programs will

look at the overall picture of your application. Show your many dimensions to prove you are the well-

rounded candidate so many programs seek. Continue with the hobbies you love and the ones you wish to

explore. Sports and recreation should not be compromised as they offer stress relief and can actually

work toward achieving your goals. This is the case for volunteering and community work as well. A few

hours a week helping out at your local community center or medical facility can add variety and insight. In

trying to satisfy a selections committee, do not lose sight of who you are and what you enjoy doing. The

more you enjoy the activities you choose this year, the more fruitful they will be.

In conclusion, you have to show that your progress during this year is a valuable asset to the residency

program. The best way to do this is by using every bit of time you have constructively in the areas

mentioned above. This is the harsh reality when taking a year off to improve your application. You do not

have the luxury of wasteful free time. That being said, if you organize yourself well from the beginning,

attacking key weak areas efficiently and spending free time doing constructive things you love, you will

not dread the task at hand and you cannot fail to show your worthiness as a student and medical

professional.

Page 63: Medscape articles

Resume Design for Success

62

Resume Design for Success Introduction

The time has come for you to leave the cozy confines of medical school and find a job. It may be a

summer job, a residency, or your first "real" job out of residency. In any case, it's very important to

present yourself in the best light possible. To help you in those efforts, this column will focus on cover

letters and resumes.

Resumes: Be Concise

Too often, says Katie Abby, Senior Vice President and Chief Operating Officer of Vista Staffing Solutions

(www.vistastaff.com), the tendency in a resume is to try to give too much information.

Try to limit yourself to 1 page.

Be sure to include your name, address, phone number, and email address at the top of your

resume.

Training and experience sections are obviously important, but much of the "filler" material (every

summer volunteer job, every published article, every research experience) should be kept to a

minimum. Highlight those pertinent to the position, and note that a full list is available upon

request. Of course, if you only have a few publications, list them all.

List awards, but again, be concise.

Include dates of employment and/or schooling (high school is not important).

If you have room, include a "hobbies and interests" section. It separates you from the crowd. Also

include volunteer experiences. Consider an "other skills" section for languages spoken or

certifications earned.

It is not necessary to list references. Include a statement such as "references available upon

request."

Cover Letters: Again, Be Concise

Too many readers, according to Abby, are not willing to scroll down or turn a second page to read a long

dissertation.

Include a brief statement of where you found out about the opening, and the reason for writing.

Make the letter specific to the person or place you're sending it, instead of a generic letter

blanketed to all potential employers.

Give a brief example of why you believe your specific talents fit the employer's needs.

Do you know someone at the company or program? Ask that person if you can mention his/her

name in your cover letter.

Email

If you're sending your curriculum vitae (CV) via email, the format and tone of the cover letter should

reflect a professional business communication rather than a conversation. Finally, whether on email or

paper, the rules of grammar and spelling always apply.

International Graduates

Page 64: Medscape articles

Resume Design for Success

63

In addition to following the above advice, physicians from other countries should state their US

employment status, USMLE and TOEFFL results, and postgraduate education simply, clearly, and early

on in their resume.

CV or Resume: What's the Difference?

This is an open question, but in general, the CV is more a synopsis of a person's life rather than simply a

summary of his or her work history. Medical professionals typically use the CV.

Email applications are becoming more popular. Abby reports that the number of inquiries her company

receives via email has tripled in the last 3 years.

"More and more, facilities prefer we send them CVs via email because it's immediate," she says. "But we

never skimp on etiquette just because it's faster and easier. Whether you have an agency send your CV

or you're sending it yourself, traditional grammar and etiquette sells itself and grabs the reader's

attention."

Page 65: Medscape articles

I'm thinking about doing an elective away from my school during fourth year. What do I need to do to arrange that?

64

I'm thinking about doing an elective away from my school during fourth

year. What do I need to do to arrange that?

Response from Adam Kawalek, MD Resident, Internal Medicine, Mount Sinai School of Medicine, New York, NY

Away rotations can be an amazing way to spend a month of elective time. First of all, you gain experience

maneuvering through another academic institution. Everything from the hospital staff to computer

technologies will likely be different. Learning how to adapt to a new healthcare delivery system not only

gives you something to compare with your own medical center, but it also teaches you that medical

training can vary dramatically from city to city or state to state.

Some hospitals have computer systems that can enhance your experience by making it easier to care for

patients with conveniences, such as paperless consent forms and clinical reminders. Oftentimes, the

residency structure itself will be different, with some institutions having eliminated overnight call and

others still adhering to more traditional residency hours.

The level of formality between attendings and residents can also vary widely; cities, such as Boston and

New York, are notorious for formal interactions and "more serious" residents, whereas the Midwest is

known for a more casual work environment.

Also, away rotations are a great way of making contacts in a field of medicine that may interest you.

"Auditioning" at a prospective program is a great way to challenge your clinical acumen and enhance your

educational experience. I, for one, elected away to 4 different programs!

Whatever your motivation, here is what you need to do:

1. Decide 3-4 months in advance that you would like to do an away elective.

2. Contact your dean's office and ask for a list of common away electives, either in your home

country or abroad. If the list does not exist or you can't find what you are looking for, then do the

following: Log on to the Internet and start looking around. If your interest is dermatology, for

instance, do a Google search and find programs that accept away rotators. Other resources to

check include lists prepared by the Association of American Medical Colleges[1] and by the

American Medical Student Association,[2] which include international electives.

3. Contact the institution that you would like to attend. Rotations of this sort are usually arranged

through the dean's office, not through the actual department.

4. Fill out all the appropriate paperwork. This can be tedious and time-consuming. Occasionally, a

letter of recommendation may be required.

5. Request information about housing.

Page 66: Medscape articles

I'm thinking about doing an elective away from my school during fourth year. What do I need to do to arrange that?

65

6. If you plan to go abroad, make sure that you have obtained the appropriate visa and

immunizations or other prophylaxis.

7. Finally, enjoy your month away. Remember to take pictures or keep a journal of contacts,

experiences, and even procedures that may come in handy for credentialing purposes.

When you return, be sure to let your medical school know about your experiences so that future students

can follow in your path.

Submit a Question on Medical School and Residency

Response from the Expert

Response from Adam Kawalek, MD Resident, Internal Medicine, Mount Sinai School of Medicine, New York, NY

Away rotations can be an amazing way to spend a month of elective time. First of all, you gain experience

maneuvering through another academic institution. Everything from the hospital staff to computer

technologies will likely be different. Learning how to adapt to a new healthcare delivery system not only

gives you something to compare with your own medical center, but it also teaches you that medical

training can vary dramatically from city to city or state to state.

Some hospitals have computer systems that can enhance your experience by making it easier to care for

patients with conveniences, such as paperless consent forms and clinical reminders. Oftentimes, the

residency structure itself will be different, with some institutions having eliminated overnight call and

others still adhering to more traditional residency hours.

The level of formality between attendings and residents can also vary widely; cities, such as Boston and

New York, are notorious for formal interactions and "more serious" residents, whereas the Midwest is

known for a more casual work environment.

Also, away rotations are a great way of making contacts in a field of medicine that may interest you.

"Auditioning" at a prospective program is a great way to challenge your clinical acumen and enhance your

educational experience. I, for one, elected away to 4 different programs!

Whatever your motivation, here is what you need to do:

1. Decide 3-4 months in advance that you would like to do an away elective.

2. Contact your dean's office and ask for a list of common away electives, either in your home

country or abroad. If the list does not exist or you can't find what you are looking for, then do the

Page 67: Medscape articles

I'm thinking about doing an elective away from my school during fourth year. What do I need to do to arrange that?

66

following: Log on to the Internet and start looking around. If your interest is dermatology, for

instance, do a Google search and find programs that accept away rotators. Other resources to

check include lists prepared by the Association of American Medical Colleges[1] and by the

American Medical Student Association,[2] which include international electives.

3. Contact the institution that you would like to attend. Rotations of this sort are usually arranged

through the dean's office, not through the actual department.

4. Fill out all the appropriate paperwork. This can be tedious and time-consuming. Occasionally, a

letter of recommendation may be required.

5. Request information about housing.

6. If you plan to go abroad, make sure that you have obtained the appropriate visa and

immunizations or other prophylaxis.

7. Finally, enjoy your month away. Remember to take pictures or keep a journal of contacts,

experiences, and even procedures that may come in handy for credentialing purposes.

When you return, be sure to let your medical school know about your experiences so that future students

can follow in your path.

Page 68: Medscape articles

Can You Learn How to Break Bad News?

67

Can You Learn How to Break Bad News? Introduction

Not long ago, I wrote a column about how I told a family that their loved one had died in the ED. I

specifically talked about the awful feeling I had after the experience -- how my words did not remotely do

any justice to what the family had just suffered. My language seemed blunt and my interactions with the

family awkward.

I've thought a lot about that interaction and wondered whether it is possible to learn how to break bad

news in a different way. My medical school tried to teach us. I remember sitting in small group sessions

doing role-plays of the task. I also have some vague recollection of a 6-step process, which included

finding out what the family already knows and getting a sense of how well you think they will react. Years

later, several themes from those classroom lessons have remained with me as I now practice medicine:

Never say, "I'm sorry"; never use words that may be interpreted incorrectly (such as we "lost" your wife or

she "passed away").

Every month in my residency, a guest lecturer spends 2 hours giving a grand rounds session. Essentially,

this person speaks about an area of interest or a topic about which they have extra knowledge. Shortly

after I wrote that column about my experience, I noticed the topic of the upcoming grand rounds:

"Breaking Bad News."

The speaker had gained national recognition for his work on a national committee on how doctors could

improve their ability to tell family members that someone had a devastating traumatic injury or death. As a

result, he read the literature, studied reports, and developed a style he now shares widely. I was skeptical

about whether one could actually learn the skill, but looked forward to his talk.

His 2-hour lecture included a number of anecdotes, both good and bad, leading to a clear method he

advocates. The lecturer insisted that his technique works to ease the emotional pain of hearing of a loved

one's death, and that some of the people with whom he has interacted still send letters and holiday

greetings each year.

Trying a Different Approach

Here are methods he outlined:

1. Put on a clean white coat (one with no blood, because it most likely will be interpreted as being

from their loved one).

2. Clearly introduce yourself when entering the room.

3. Find out who is in the room and identify the next of kin.

4. Get down to eye level with that person.

5. Reach out and make some contract with that person, such as their hand or knee.

6. Make eye contact.

Page 69: Medscape articles

Can You Learn How to Break Bad News?

68

7. State, "Your (husband, daughter, brother) has died."

8. Allow up to 2 minutes of silence.

9. Ask if they have any questions about what happened.

10. Never leave the room before asking a personal question about the person who died or the life

that person had with their family.

I'll admit that some of this sounded crazy. My gut instinct has always been to skirt around the issue and

explain what happened (the ambulance arrived and he was not breathing; they inserted a breathing tube;

we gave him a number of medications; we did CPR...). The lecturer's ideas were relatively nontraditional

compared with what I had learned, but I wanted to give them a try.

I was working in our ED on a Saturday morning several weeks ago -- the morning of the anticipated (and

actual) blizzard of 2005. The secretary asked me to pick up the phone for an ambulance call-in alerting us

that they were on the way. On the other end of the line, the paramedic told me that they were 3 minutes

away with a 56-year-old man who was found in cardiac arrest. From their 2-line report, his potential

outcome sounded very poor. I prepared myself for the resuscitation.

The details of our efforts are not important, as it was clear when the patient arrived that we were not

going to be able to save him. The nurse told me that the patient's wife was on the way. Apparently, the

patient had been driving in his car but was found by a bystander. The local police department went to the

wife's home and drove her to the hospital.

Putting Theory Into Practice

The nurse found me when the patient's wife arrived. They had escorted her to the "family room" -- the

quiet room at the end of the hall away from the patient area where family members wait for an update. I

told Karen, the nurse, that I was planning on trying a new technique for breaking bad news. I described

exactly what I was going to say so that she would not be surprised.

We entered the small room (2 chairs and an end table in there) to find the patient's wife waiting. Karen

introduced me and I walked inside. We closed the door and I sat down on the end table next to her. I

reached out with both of my hands and held the woman's hand while I looked her in the eyes. Before I

had a chance to say a word she said, "Oh my God, you're going to tell me that he's dead." She knew.

My lecturer had said that this would happen -- our body language and actions alone tell family members

what we are going to say. The words were much less hard for me to say as his wife had already said it. I

essentially repeated what she said, followed by, "I am so sorry that this is happening to you." She cried

for a good minute and a half holding onto my hands. I allowed the silence. It took everything inside of me

not to retell the whole sequence of events (and Karen the nurse couldn't withhold the urge and started

detailing the ambulance arrival, medicines, and CPR). I planned on asking the wife what she wanted to

know without our own interjections.

Instead, she sat up and said, "You just can't possibly know how much we love each other. This can't be

happening." I then was easily able to ask her, "How long have you been married?" In the next 10 minutes,

I learned about their life together, their children, and how much they loved each other. I never had that

experience before under those circumstances.

Page 70: Medscape articles

Can You Learn How to Break Bad News?

69

When I left the room, I did not have that usual hollow feeling in the pit of my stomach I had felt so many

times before. Instead, I felt like something that traditionally has been one of the worst parts of this job

became one of the most important and moving patient interactions I had ever had. Maybe it was luck that

things played out exactly as predicted by the guest lecturer. Or, maybe you actually can teach someone

how to break bad news. Unfortunately, I know I will be presented with similar situations in the future, and I

will wait to see what the response is before I am completely convinced.

Don’t forget to google the keyword “spikes protocol”

Page 71: Medscape articles

Are Board Scores Important When Applying for Residency?

70

Are Board Scores Important When Applying for

Residency? Question

Do residency programs really use cut-offs when reviewing applicants' board scores? Are board scores

especially important for international medical graduates?

Response from Sarah Bernstein, MD Resident, Department of Obstetrics and Gynecology, St. Luke's-Roosevelt Hospital, New York, NY

When I was a medical student, I remember having palpitations and breaking out in a cold sweat every

time I thought about the Step 1 exam. As a foreign medical graduate, I believed a good score was the key

to my future and the only way I could attain a decent residency position. Looking back, I would still agree

that board scores are important, but I realize now that they are only one of many components that

residency programs consider in the admissions process.

Residency programs receive numerous applications each year, and board scores are the easiest way to

limit the number of people they will interview. Some of the more competitive residency programs

reportedly do use cut-offs for board scores, not because they believe that "a good score equals a good

resident," but because they have too many applications to sort through and need a simple and fair means

of limiting their interview numbers.

Residency programs that interview international applicants may place even more emphasis on board

scores. The logic is that they have a better idea of how US medical students are educated and what skills

they will possess from the start, while the international applicant's level of preparation may be unknown.

With a strong board score, a program will feel more confident that the international applicant will perform

at the same level as his or her US counterpart.

If you have already taken Step 1 and finished your core clerkships but are afraid you just won't measure

up... there is still hope! An away elective (a rotation at an institution other than those affiliated with your

school) may be the best way to achieve your goal, especially if you do it at a hospital where you hope to

do your residency. Low board scores may be overlooked if you impress the faculty and residents with

your hard work, intelligence, and exceptional interpersonal skills.

Many program directors think that the most important factor in resident success is compatibility with the

other residents. The away elective can be compared to a first date and should be taken very seriously.

Each resident you meet is thinking of the future and considering what it would be like to spend 80 hours a

week, in sickness and in health, by your side. Keep in mind that performance on the job is a better

indicator of future success than a test score.

Page 72: Medscape articles

Are Board Scores Important When Applying for Residency?

71

In summary, there is no need to take a beta-blocker as you study for Step 1. Take a deep breath, let the

palpitations subside, and remember that if your score is low, you will just have to spend some time

strengthening the other aspects of your application.

For more information about applying for a US residency as a foreign medical graduate, I recommend the

following Web sites:

Educational Commission for Foreign Medical Graduates. This site contains all the rules and

regulations for applying to a US residency program.

Residency Guide for Foreign Medical Graduates. This is a site put together by a foreign medical

graduate, and it's full of great advice.

International Medical Graduates. You can submit your own questions on this blog.

American Medical Association.Their site has a special section for international medical graduates.

Page 73: Medscape articles

How Can I Increase My Confidence in Doing Physical Exams?

72

How Can I Increase My Confidence in Doing Physical

Exams? Question

What should I do if I lack confidence when examining patients, regardless of how knowledgeable I am?

Response from Adam Kawalek, MD Internal Medicine resident, Cedars-Sinai Medical Center, UCLA, Los Angeles, California

What a wonderful question! One of the most important traits that a physician should nurture is humility. I

want to point out that the reader who sent in this question identified her lack of poise, related it to her fund

of knowledge, and then sought advice on how to improve. I often find that medical students who question

their abilities and look for ways to improve their skills are more respected by their peers and appreciated

more by patients for their humility and self-awareness.

Examining the human body is a privilege, and an intimate act that truly distinguishes medical

professionals from any other profession on earth. No wonder you are nervous! Realize first and foremost

that good exam skills take years to develop. Any encounter with a patient represents, in a sense, a

snapshot of their physical history and a moment in time for a certain disease process.

Recognizing the manifestations of that disease and applying what you have seen before is the ultimate

goal. For example, liver cirrhosis can manifest with many subtle physical exam findings. If you synthesize

and apply the knowledge you learn from examining several hundred patients with cirrhosis, I promise that

by the end of medical school you will be able to accurately diagnose liver cirrhosis without any laboratory

or diagnostic imaging help! Cultivating your physical diagnosis skills will serve you throughout your

career.

Medical school is a time for curiosity, inquiry, and building a strong foundation or "fund of knowledge" that

will continue to grow. Take the time to understand and experience what is in front of you. I encourage you

to practice your skill set as often as possible. If you are too nervous during your clerkship rotations, try

volunteering in a free clinic; they tend to be a lot more casual. Alternatively, practice your physical exam

skills on a friend or family member. Any environment that will allow you to explore what you see, hear,

and feel without the constraints of time or pressure will work to your benefit.

Another suggestion is to focus your examination skills on a single organ system at a time. From a

practical standpoint, you may need to document or present an entire physical exam; however, if you focus

your energies on a single system, you may find yourself more confident in less time.

Try to read several pages of a physical diagnosis book and apply what you have learned the next day.

Attempting new techniques or maneuvers when examining patients will enhance your comfort level. Also,

make an effort to apply new vocabulary or adjectives to describe what you have examined.

Page 74: Medscape articles

How Can I Increase My Confidence in Doing Physical Exams?

73

Finally, you owe it to yourself and to your patients to be honest. There is nothing wrong with prefacing a

physical exam by stating humbly that you are a medical student doctor and that you are learning skills

that will make you a great doctor. Feel free to apologize for your overzealous abdominal exam, or for

blinding your patient with a lengthy funduscopy. Patients are aware that they are participating in a

centuries-old ritual. Most are eager to subject themselves because they are inspired by our dedication.

Use your inexperience and lack of self-confidence to make a connection with your patient. You just may

find that their acceptance is all you need ... to feel confident.

Page 75: Medscape articles

I didn't enjoy my third year of medical school and now I'm not sure if I want to do a residency anymore. What should I do?

74

Question:

I didn't enjoy my third year of medical school and now I'm not sure if I

want to do a residency anymore. What should I do?

Response from Sara Cohen, MD

Fellow, Department of Physical Medicine and Rehabilitation, Harvard

University, Boston, Massachusetts; Fellow, Department of Physical

Medicine and Rehabilitation, VA Boston Healthcare System, Boston,

Massachusetts

Near the end of my third year of medical school, I went out to dinner with a few of my classmates.

Naturally, the conversation quickly turned to which residencies we wanted to get. My friend Ben, who had

just finished his surgery rotation and still had circles under his eyes, said, "I don't think I want to do

residency. I hate patient care."

We were all aghast. "So what are you going to do?" someone asked.

"I don't know," Ben admitted. "But there must be some options out there for an MD who doesn't want to

do a residency, right?"

Since my conversation with Ben, I've heard that question posed many times by exhausted medical

students. Some students thrive on the excitement of third year, but some hate waking up early, working

weekends, being constantly sleep-deprived, and missing out on time with their families. Other students

find that they hate procedures or dealing with difficult patients.

The first question to ask yourself is why you don't want to do a residency. Is it because of the long hours

and difficult call schedule? If so, you might consider some of the specialties that have less grueling

residencies. The infamous ROAD specialties (radiology, ophthalmology, anesthesiology, and

dermatology) are known for combining high pay with reasonable hours, although for that same reason,

these residencies are generally very competitive. Emergency medicine also has reasonable shift work,

even during residency. Other specialties with reasonable hours that tend to be less competitive include

psychiatry, pathology, and physical medicine and rehabilitation (PM&R). As a PM&R resident, I worked

mainly 8 to 5 with rare weekends and call from home. It was nowhere near as grueling as, say, a surgery

residency, and I didn't miss my family or feel sleep-deprived. (Learn more about PM&R in an article I

wrote previously for Medscape.)

If your main concern is that you dislike direct patient interaction, consider one of the specialties where

patient care is minimal, such as radiology, pathology, or preventive medicine. Whereas radiology and

preventive medicine require a clinical transitional year, pathology does not. That means that if you enter a

pathology residency, you'll never have to see another patient.

If you like patient care but hate procedures, consider psychiatry. If you love procedures but hate long-

term patient care, consider anesthesiology or emergency medicine.

Page 76: Medscape articles

I didn't enjoy my third year of medical school and now I'm not sure if I want to do a residency anymore. What should I do?

75

The field of medicine is incredibly broad, with many specialties, and it is likely that a good "fit" can be

found for everyone, depending on individual likes and dislikes. Residency is probably the best option for

someone who has completed medical school, and just because you complete a residency doesn't mean

that 100% of your future job must involve patient care. A lot of clinicians do some research, teaching, or

administrative work in addition to their clinical duties, so you can divide your time and create the lifestyle

you want.

Try to remember why you went to medical school in the first place, and allow yourself some time to

recover from your exhaustion. Medical students can get discouraged when they don't immediately love

one of the core clerkships during third year, which is why you should try to set up interesting electives that

are potential career choices. Also, you might see things differently when you've had a few good nights of

sleep.

If after thinking it through thoroughly, you still decide that residency isn't for you, you can pursue several

options. It's a good idea to complete at least 1 year of residency so that you can get a medical license.

That way, if you ever decide to return to medicine, you will be in a better position if you already have a

license.

A physician who doesn't want to do clinical work has many other options. If you have a mind for business,

you may want to consider getting an MBA (Master of Business Administration) and working in the

administrative end of medicine. Alternately, you can get an MPH (Master of Public Health) and find work

in public health. Careers in medical writing, informatics, engineering, and consulting are also possible.

Dr. Joseph Kim hosts an extensive Website about nonclinical medical jobs, including actual job

opportunities as well as general guidance about pursuing a nonclinical career. But keep in mind that each

of these fields has its own set of challenges, so you should thoroughly investigate these options before

making a dramatic career switch.

As for my friend Ben, after a nice long shower and a nap (and possibly some research), he decided that

residency wasn't such a bad option after all. He matched with the rest of us.

Page 77: Medscape articles

How Can I Be Successful on the Job?

76

How Can I Be Successful on the Job?

Question

I was recently dismissed from my first residency position. I was told that I was not advancing as fast as

expected, I was not asking questions enough, and my confidence was "scary." I need to find another

position but I don't know how to become more employable.

Response from Anne Vinsel, MS,

MFA

Project Administrator

University of Utah School of Medicine

Salt Lake City, Utah

First, congratulations on seeking information instead of crawling into a defensive shell. From the feedback

they gave you, I can derive a pretty familiar pattern, and if you don't change it you will really hurt yourself

in medicine. A friend of mine who is a division chief calls this pattern "the 6-month great hire," meaning a

person who has lots of great qualities but who bombs quickly.

"Not advancing as fast as expected" usually refers to your learning curve, although it can also refer to not

fitting in socially. Every medical site has expectations, usually unstated, about how quickly a new

employee should be able to pick things up. The skills they are talking about are usually some combination

of processing speed and rote memory. Some supervisors may tell you something once and expect you to

remember it forever. Some might not plan on training you at all and just expect you to pick up everything

you need by watching and learning for a short period.

Your choices are to either find a job with less demanding expectations (eg, slower pace) or to find a way

to crank up your learning speed on the job. Some possible ways to do that would be to carry a small tape

recorder so that you can review what people have told you (at home at night), practice improving your

rote memory in your off hours, and selectively get instruction from people who are organized but kind.

"Not asking enough questions" is more difficult to interpret. A legitimate concern is that a resident who

makes too many assumptions could kill patients. It's a fine line in medicine between asking too many

questions (and thus wasting professionals' time) and not asking enough questions (which can create

chaos among the staff and endanger patients and staff). Our office is always wary of new employees who

just nod and never ask questions, particularly if they don't seem to be learning quickly (we can tell this

from their performance). It usually signals cognitive overload and that the person isn't grasping the

complexity of the situation.

You are in a bit of a "3 bears" situation here. You don't want to overreact and start making a pest of

yourself with millions of questions; rather, you want to hit a "just right" level of questioning. Realize that

many people in medicine were raised on the Socratic method, which requires the learner to ask questions

to help the teacher fine-tune what they're teaching.

Page 78: Medscape articles

How Can I Be Successful on the Job?

77

Now, "scary confidence" implies that you take on tasks you're not ready for, and in a hospital setting, that

is just plain bad. Often, there can be disagreement about whether "scary confidence" is justifiable. You

might think you are appropriately confident, but if your supervisor thinks you are overreaching, the higher

status person will always win. You are a fresh graduate, you are probably young, and you may have

cultural or generational differences with your supervisors.

For example, I personally worry about young people who say "okay" very quickly, particularly if they cut

off some of the information I'm trying to convey. I have noticed that people who are assessed as "scary

confident" also tend to be verbally expressive, prone to joking, impertinent with superiors, and they don't

listen very well (before they disappear). Some medical students and residents, possibly due to

nervousness, are so eager to impress that they are always getting a response ready instead of listening

carefully to the person who is talking. Do you see yourself in this picture?

Think about how you'll want to change in your next position. I recommend laying back, listening hard,

being sensitive to hierarchy (even if their ideas about computers are wildly outdated), and withholding any

suggestions about how to do things better. Ask someone you trust to give you honest feedback once a

week, then adjust your persona as needed.

You don't have to stay in this cautious, earnest mode forever, but it's a good way to start out. Eventually,

as people learn to trust that you know what you're doing, you can be more yourself again.

Page 79: Medscape articles

How Do You Handle Pre-examination Stress?

78

How Do You Handle Pre-examination Stress?

Question

I often panic before an exam because I am so worried about my performance. This makes it even harder

to do well. What should I do?

Response from Sohil H. Patel, MD

Sohil H. Patel, MD, Intern, St. Vincent's Hospital, New York, NY

There is an obvious answer to this question: prepare well for the examination. But of course, it is hard to

find anyone who would actually claim to be well prepared in the days before an exam. This is particularly

true for exams that encompass a vast amount of information. No student knows every detail about the

topic being tested. That is why it may always seem like you need more time to study, and why you never

feel fully prepared.

The examinations themselves can generate anxiety as well, whether because of the time limits for

finishing or perhaps because of an oral performance component. Of course, no two people handle stress

in the same way, but here are some things that helped me.

The first step in destressing involves gaining some perspective. Having been in the clinical setting for a

few years now, I have been exposed to life and death situations that legitimately stress people out. Given

those experiences, I now find it hard to believe that I ever stressed out over my medical school

examinations to the extent that I did. In fact, I remember developing a different (and more healthy)

attitude toward studying and exams after I started my clinical rotations, which made me realize just how

insulating the pre-clinical years can be. So, if you are a first- or second-year medical student, shadowing

a physician in the hospital might help put your exams in proper perspective.

Another way to gain perspective is to remember why you chose to attend medical school (hopefully, doing

well on medical school examinations was not the reason). Studying really should be viewed as a means

to a much larger goal than just passing a test. Exams should never be treated as ends in themselves.

They are really just little check marks of approval on your way to becoming a physician -- nothing more.

Certain study techniques may also be effective in reducing your stress level. Cramming facts into your

brain at random is not effective; if your mind goes blank during an exam, you won't have any broader

framework on which to anchor your thoughts. Before memorizing details, I always tried to build a

framework. Take pulmonology, for example. Instead of learning pulmonary diseases at random, learn the

basic categories of disease (eg, obstructive, restrictive), then learn the common features within each

category, and finally, fill in the categories with particular diseases. This way, if the answer to an

examination question is not immediately obvious, you have a systematic way of proceeding and

narrowing your choices.

Another helpful method, particularly for oral exams involving standardized patients, is to develop routine

practices. Before the exam, run through common scenarios or complaints you might encounter and think

of the basic steps you should always take in those scenarios. Do this a few times so that, if a similar

situation is encountered in the exam, your brain can run on auto-pilot as you follow through with your

prepared steps.

Of course, there are many ways to address the stress itself, and what works for one person may not work

for another. For me, it seemed to help if I stuck with my workout schedule regardless of how close I was

Page 80: Medscape articles

How Do You Handle Pre-examination Stress?

79

to an examination; this helped me set aside some time when I was not thinking about the exam. I also

had the peculiar habit of taking a shower if I thought I was getting overly stressed out and I needed a

break. But these methods vary for everyone. In general, it is probably a good policy to keep your life as

close to normal as possible before an exam. This avoids giving the examination undue significance.

Finally, it is important to be confident. Medicine is not the single most complicated field of study, and most

minds are similarly suited to learning it (unlike, perhaps, philosophy or some other fields that some people

seem more inherently suited for than others). It means that there is a good correlation between how much

effort a person puts into learning medicine and how well that person masters the subject. It is less a

matter of "getting it" and more a matter of "knowing it," and almost anyone can learn the material with a

certain amount of effort. So, work hard and take confidence that your honest effort will pay off!

Page 81: Medscape articles

What If I Match a Residency Program I Don't Want?

80

What If I Match a Residency Program I Don't Want?

Daniel J. Egan, MD

Attending Physician, Emergency

Medicine Department, St. Vincent's

Hospital, New York Medical College,

New York, NY

The residency application season is by far one of the most stressful times for a medical student. You

spend months thinking about where to apply, large sums of money traveling around the country, and then

days to weeks struggling with the list. Who should I rank first? Do I have a chance of matching there?

What happens if I don't get my top choice?

The last question is one that I can vividly recall asking myself and hearing my medical school friends

repeat time and again. The entire process of the match creates a sense of success or failure based on

where you wind up. Schools have match ceremonies. The lists are published and bragged about. It is,

after all, somewhat of a competition, with the ultimate prize being your "first choice."

Nearly every student who enters the match worries about not getting their first choice, or worse, not

getting one of their top 3 choices. What if that actually happens?

The first thing I have to say is, "Congratulations!" you matched. Approximately 6% of US graduating

medical students do not match at all and have to scramble. A stressful process has thus become even

more stressful for those individuals. Furthermore, according to the National Resident Matching

Program www.nrmp.org, in 2007, 58.6% of US students matched at their first choice. That may seem like

a lot, but it also means that many students are not going to their first choice.

In my experience, most people will tell you that the match actually works. You are going to a program that

wants you to be there. Remember, the same way that you were told not to rank any program where you

cannot imagine doing your training, residencies also do not rank students who they do not want working

for them. If you have a spot, it means that both you and the program managers agree that you fit in there.

This is exciting when you think about it. Of course you will be disappointed when you first get the news,

which is a normal response because of the amount of time you spent thinking about that list. However,

allow that disappointment to be brief, because residency is challenging by design and you will need

positive energy.

Residency is an exciting time. Any residency program is going to introduce new aspects to your life.

Regardless of the program's ranking on your list, you will be embarking on a journey that will form you

into a true physician. Your knowledge base and skill set will increase exponentially. You will forge

friendships like you never had before. Each new resident begins the journey alone, but you will soon have

a group of people with whom to share that journey. You will bond, rely on each other, vent to each other,

disagree with each other, learn from each other, and ultimately graduate with each other. Many of you will

start a life in a new city and realize that it may be the beginning of the rest of your life.

Page 82: Medscape articles

What If I Match a Residency Program I Don't Want?

81

So fear not. Your residency program is excited to have you. You are finally a doctor. Keep in mind that

almost half of the applicants do not get their first choice. It's okay. The match works. People will tell you

that, somehow, everything seems to be fine in the end. Try to move on from the initial disappointment

because, literally, these are the most important years in your future. Embrace them and enjoy them!

Page 83: Medscape articles

Why Doesn't My Resident Trust Me?

82

Why Doesn't My Resident Trust Me?

My resident seems to be a "micro-manager," constantly double-checking my work. What should I do?

Response from Adam Kawalek, MD

Adam Kawalek, MD

Resident, Internal Medicine, Mount

Sinai School of Medicine, New York,

NY

Residents, fellows, and attendings repeatedly review any work done by students whom they are

supervising. They may ask for patients' daily lab values and then go and check the results on the

computer themselves. Residents may want to review lab tests, imaging studies, and (yes!) even diets that

you order for your patients. Understandably, this is one of the most frustrating elements of life as a third-

or fourth-year medical student -- and even as an intern. You may be in your 11th month of internship and

still feel the lingering eyes of your resident on you.

Why this apparent redundancy? Simply put, it is a built-in safety mechanism to prevent medical errors.

This does not mean that your work is subpar or untrustworthy. In fact, as an intern, most residents

consider you the primary care provider for your patients. As a third-year medical student, you have less

official responsibility, but during the fourth year of training you begin to take on the role of the intern. Still,

even the best interns and students will make mistakes, and mistakes in the medical world can carry grave

implications that may not always be apparent to a novice. This is why your resident checks up on you so

often.

My recommendation is this: Instead of perceiving your resident as a mistrusting "micro-manager," try to

think of him or her as a safety net. Ignore the redundancy, and consider it a necessary evil. Most

important, try not to personalize it. Your resident is not there to make you feel like your work is deficient or

that you cannot be trusted to accurately recite a patient's lab values. Take solace in the fact that you are

not being left all alone and that someone is there to cover your back.

That said, there are elements of your work style that can impress a resident and garner trust. First, the

importance of organizing your work cannot be emphasized enough. Figure out a system that works for

you and stick to it! A systematic approach to gathering data (the same as reading an EKG or interpreting

a chest x-ray) will help you pay attention to detail and prevent you from missing important elements.

Second, refrain from answering questions when you don't know the right answer. There is nothing worse

than an intern or student who provides inaccurate information. If you are uncertain about the answer to a

question, just say so. In fact, one sure way to make your resident check and recheck your work is to

report something incorrectly. We have all been tempted to say that we completed a task or got the results

of a test just to avoid getting in trouble, but once trust is broken, it is difficult to restore, especially with

someone who is ultimately responsible for your work.

Page 84: Medscape articles

Why Doesn't My Resident Trust Me?

83

Finally, instead of waiting for a resident to check your work, take the initiative yourself. Find your resident

and show them what you have done. Outline the clinical decisions you would like to make based on the

data at hand, and ask for their input.

An intern or student who follows these simple steps is sure to shine. If you make an effort to establish

yourself as a reliable clinician, your supervisors will learn to trust you. Remember that medical training is

a time for you to develop your skills and clinical acumen in a safe (and yes, redundant) environment.

Page 85: Medscape articles

What is more important in practicing medicine, the theoretical knowledge of the science or the practical intricacies and skills?

84

Question

What is more important in practicing medicine, the theoretical

knowledge of the science or the practical intricacies and skills?

Response from the Expert

Lisa Jacobson, MD

Resident, Emergency Medicine, Mount

Sinai School of Medicine, New York,

New York

I must admit that, at first glance this question seems to fit into the category of "which came first, the

chicken or the egg?" -- perfect for stimulating philosophical discussion, but ultimately unanswerable.

However, it is a question that is discussed frequently among physicians and while a bit philosophical is

important to address.

Just days before I began contemplating a response, some colleagues and I were discussing a related

topic -- Doogie Howser, or more specifically, what Doogie has come to represent: extremely young

medical students or physicians. While reminiscing about classmates in medical school, one person

recalled a teenager who had started medical school in the class behind her and the awe and hype that

had surrounded him. This prompted a rather passionate discussion about kids in medical school. The

general consensus was that despite the likelihood that this kid would do better on his board exams than

anyone else we knew, none of us would feel comfortable with him as our doctor or as a family member's

doctor. Our concern stemmed from our belief that other life skills and experiences influence our success

as physicians, not just an IQ.

Being an effective physician is a balancing act. Patients may look sick but be healthy or look healthy and

be sick or anything else in between. Some will lie, exaggerate, and abuse the system; others will avoid

seeking care because of fear or cost or apathy. Most importantly, none will enter your practice

environment with a big arrow pointing to the problem or with a list of symptoms straight out of a textbook.

Sifting through stories for truth is a skill that we all learn in our personal and professional lives. Assigning

importance to various pieces of the puzzle comes along with that skill. Identifying what is actually wrong

at times depends very little on textbook knowledge.

This is not to say that just being able to recognize a problem, or the lack of a problem, is enough. Once

you've broken through the glossy coating and identified the truth, you then must know how to treat. This is

where the textbook comes into play. If you haven't studied, passed your boards, and read the current

literature, you will not be able to effectively and accurately address the problem at hand. In fact,

attempting to do so without the appropriate knowledge is more than a disservice, it's a violation of your

oath.

Page 86: Medscape articles

What is more important in practicing medicine, the theoretical knowledge of the science or the practical intricacies and skills?

85

As much as we don't like to admit it, there's probably a little nerd in all of us. We study hard, but we were

all pretty smart to begin with. We all can probably remember Horner's syndrome, though we don't

remember who Horner was. At one point we all passed organic chemistry, memorized the Krebs cycle,

and identified histologic slides. Learning this theory has used quite a few neurons, and at the time, our

professors told us it was important. Now, while practicing medicine, we all know it wasn't a complete

waste of time (although there are some useless factoids stuck in my brain that I wish I could purge for

other things). We also know, however, that all this knowledge can never be enough. Medicine is an art as

well as a science, and we have to function well within both worlds to be effective.

Page 87: Medscape articles

How can I remember and integrate the basic subjects of medical school with real clinical practice?

86

Question

How can I remember and integrate the basic subjects of medical school

with real clinical practice? Response From the Expert

Sohil Patel, MD

Intern, St. Vincent's Hospital, New York,

NY

There are a number of transitions that take place when a medical student enters the clinical phase of

training. Foremost among them is an essential shift in learning. This shift entails a move from learning

about physiology, pathophysiology, and pharmacology (which are rather tidy and intuitive fields of study)

to learning evidence-based medicine (which includes information that often is not intuitive, or even

counterintuitive).

While the basic sciences provide a necessary framework for understanding clinical medicine, they most

certainly do not provide a sufficient framework. I recall 2 patients I encountered early in my clinical

experience who drove this point home.

The first was a young girl I saw in the emergency room during my pediatrics rotation. I noticed

immediately that her breathing was labored. When I spoke with her, she could not complete a full

sentence without pausing to take a breath. I listened to her lungs and found that she had expiratory

wheezes throughout her lung fields. This was a clear case of asthma. My mind buzzed with all the facts I

knew about asthma (having just studied for step 1). I recalled that asthma is a disease of airway

inflammation and bronchoconstriction from smooth muscle hyperresponsiveness. The treatment, logically,

included administering corticosteroids (an anti-inflammatory agent) and beta-agonists (bronchial smooth

muscle relaxants). Also, recalling the pharmacology of steroids and the physiology of the adrenal glands,

I suggested tapering the steroids slowly in this patient -- and my resident, of course, agreed.

This case was particularly satisfying early in my clinical experience. I was able to apply my basic science

knowledge successfully and tidily to the diagnosis and treatment of a sick patient. The transition to the

clinics felt seamless.

A month or so later, I started my medicine rotation. I was assigned to the heart failure service. My first

patient was an elderly man who came to the hospital short of breath, a condition that worsened when he

tried to walk or exert himself. His examination was significant for inspiratory crackles at his lung bases,

swelling around his ankles, and prominent neck veins. The resident had already seen the patient and had

written orders, so I reviewed them myself after I examined the patient. I was surprised to see that he

ordered the continuation of a beta-blocker. From what I recalled, beta-blockers were negative inotropes

Page 88: Medscape articles

How can I remember and integrate the basic subjects of medical school with real clinical practice?

87

and chronotropes. In a patient whose systolic function was likely compromised, how could we start an

agent like that?

When I asked, my resident referred me to clinical studies that have definitively shown a significant

mortality benefit of beta-blockers in chronic heart failure (CHF) patients. In fact, he told me that the

ejection fraction of CHF patients on beta-blockers actually increases after about 6 months of therapy. I

was confused.

That night I did go to the literature. He was right, of course. When I sought a physiologic explanation, the

best I could find were unverified a posteriori hypotheses regarding the beneficial effects of beta-blockers

on the sympathetic tone of the heart; most researchers said that there is no good explanation as of yet. A

further surprise was learning that positive inotropic agents, such as milrinone, actually increase mortality

in CHF patients! All of this information left me a bit frustrated.

I soon realized that this is what clinical medicine is all about. In medicine, the basic sciences only provide

hypotheses and predictions. It is left to clinical researchers to evaluate these hypotheses in the real world,

in real patients. Sometimes they hold up very neatly (eg, asthma and its treatments). Sometimes they do

not (eg, CHF and some of its therapies). These counterintuitive examples add an extra layer of

complexity that students must master. The fact that not all clinical phenomena fit the expectations of basic

science is an essential challenge for medical students to tackle as they enter the clinics.

So, does this mean that all we need to know are the results of clinical trials? Certainly not. First of all,

without a basic science background, clinicians would have a nearly impossible time interpreting the wide

range of signs and symptoms they see in patients. Second, the basic sciences help provide an essential

memory tool for clinicians; it is much easier to remember asthma therapy by knowing the pathophysiology

and pharmacology involved than it is to simply memorize a list of symptoms and names of drugs. Third,

not all clinical questions have yet been rigorously tested in randomized trials; in these cases, it is often

our basic science understanding, as well as this vague notion called the "art of medicine," that inform our

clinical decisions.

Take, for instance, beta-blockers in CHF: it is recommended that they be initiated "gradually and with

great care."[1] Knowing this drug's mechanism of action helps us understand why this recommendation

makes sense. But the recommendation itself is rather vague and leaves plenty of discretion to the

clinician. The clinician's medical knowledge and experience will guide him or her as he or she acts "with

great care" in starting these medications, since no optimal starting regimen has been determined. In the

end, it is truly a combination of basic science, evidence-based guidelines, and clinical experience that

shape a physician's practice.

Page 89: Medscape articles

I just finished medical school and am now an intern. How can I already be expected to teach medical students?

88

Question

I just finished medical school and am now an intern. How can I already

be expected to teach medical students? Response From Expert

Daniel Egan, MD

Attending Physician, Emergency

Medicine Department, St. Vincent's

Hospital, New York Medical College,

New York, NY

It seems crazy, right? But as an intern, you are "responsible" for the education of medical students. I am

sure you can remember talking to other medical students about which residents and interns were good

teachers. Now you are thrown into the role of being the teacher. I have always found it interesting that,

although we are expected to be lifelong teachers of students, patients, interns, residents, and fellows, we

infrequently receive formal training in how to teach.

But back to the question: What it is like to work with a medical student as an intern? I can remember the

first time I had a medical student working with me. I recall the struggle I had internally about making sure

that I gave the student independence to feel like she was a part of the team, while at the same time

staying actively involved in the patient's care. I still struggle with that dilemma, as I hate to be labeled a

"micro-manager" by residents. In using that label, they imply that I am too hands-on and do not give them

the freedom to make their own clinical decisions. As you will see, though, it is difficult to relinquish control

of something that is critical to a patient's outcome as well as to your own medical license.

I think the first thing to remember when you have a medical student working with you is that they are

there to learn. We can all think of times as a student when we worked with a resident who took the time to

teach us something worthwhile, whether it was telling a quick vignette on rounds or teaching the intricate

details of a procedure (even a nasogastric tube is exciting as a student). My first piece of advice is to find

the teachable moment. Almost every patient encounter can provide a small topic for teaching. Although

students are close to you in their training, there is a substantial learning curve that takes place through

the clinical years and then during your intern year. You actually do know more than them at this point

(maybe not about the pathophysiology of the Krebs cycle, but definitely about clinical medicine). Students

love to learn. They will respond well to your taking the time to talk about a topic or procedure with them.

Second, give the student independence. This does not mean neglecting your patients, but try to find

some part of the patient's care for which the student can feel responsible. Perhaps this means a blood

draw if your nurses don't draw bloods. Perhaps it means obtaining the reading from the radiologist.

Perhaps it means spending time at the bedside to update the patient and/or family on what is going on.

Although we all have suffered through the scut of being a student, giving them a task that is directly

beneficial to the patient as well as something that will help them in the long run is highly worthwhile.

Page 90: Medscape articles

I just finished medical school and am now an intern. How can I already be expected to teach medical students?

89

Empowering a student with something lets them know that you value their membership on your team and

are relying on them for some portion of your patient's care.

Third, remind yourself that this is your patient. Students are students. They are learning and figuring out

how to practice medicine. You will see as you go through your training that it becomes easier and easier

not to reexamine someone if you are told the lungs are clear. Or, perhaps, you are told that the labs are

normal but have not verified them yourself. There are some tasks that are appropriate to give to a student

rather than doing them yourself (such as submitting a requisition for a test). However, in all examples

directly related to your patient and their well-being, you need to check and double-check your student's

work. Yes, this creates more work for you, but it is part of the deal when working in a teaching setting.

Remind yourself that someone once did this for you, and now it is your turn to do the same.

As a busy intern, you will resent everything that gives you more work. But try to avoid letting this

additional work cause any bad feelings toward your student. He or she is part of your team, and as you

know, they will do practically anything you ask of them. They can be a huge asset. In addition, you will

learn from teaching. The more we move away from the "see one, do one, teach one" mentality and

embrace our teaching responsibilities, the stronger we will become as clinicians and the better training we

will provide to future interns and residents.

Page 91: Medscape articles

How Can I Predict My Success in the Residency Match?

90

How Can I Predict My Success in the Residency Match? Question

I've spent several months searching for average board scores and other statistics for residency programs

I'm considering, but I haven't had any luck. Do you know where I can find such information?

Response from the Expert

Daniel J. Egan, MD

Attending Physician, Department of Emergency Medicine, St. Vincent’s Hospital, New York, NY

First of all, congratulations on reaching the point in your education where it is time to start searching for a

residency. It is a huge transition but also a very exciting time.

When you were first considering medical school, you may have gone to the local bookstore and picked up

some thick book describing all of the schools around the country. The statistics were all nicely laid out for

us in tables and graphs: What was the average grade point average of students accepted into a particular

school? How did students there do on the MCAT examination? How many students came from Ivy

League colleges? Every piece of information was available to help you determine if you had a chance at

acceptance.

Residency is quite different. The reason why you are having trouble finding objective data about residents

in particular programs is that, on the whole, it does not exist. Unfortunately, you cannot buy a single book

that provides all the relevant data. So, where does that leave you? How can you determine where you

have a chance of matching? I will offer some suggestions that may help you navigate the process.

The first place to look is the online directory of residency programs compiled by the American Medical

Association (AMA).[1] Using the database provided on this Web site, you can get some limited information

about different specialties. For instance, you can see what the average work hours are for surgery

compared with pathology. You can identify what percentage of residents is male or female, and what type

of career path different graduates wind up choosing.

Next, you can check the Web site of the National Resident Matching Program.[2] On this site, you can find

statistics for the most recent 'match,' as well as information for previous years. There is a great deal of

interesting information, such as the percentages of applicants who were US or foreign graduates, and the

number of programs and positions that did not fill or match. However, there is no information about

specific programs.

Finding specific information about individual programs is a more involved project. Most residency

programs do not publish data about their residents. You may find out when you interview where residents

got their medical degrees, and some programs even provide this information on their Web sites. Beyond

that, however, you will be limited in the amount of information you can obtain.

As you travel the interview trail, you may hear statistics quoted by program directors. Most of the time,

their job is to sell you on the idea that residents in their program want to be there, and vice versa. But,

you will probably never hear information about average board scores. The reality is that, in this process,

there are so many factors that contribute to a residency match, and picking one small aspect is unlikely to

be predictive.

Page 92: Medscape articles

How Can I Predict My Success in the Residency Match?

91

It is worthwhile to speak with faculty at your medical school about your career plans. As they have

watched students go through this process for many years, they likely will have realistic knowledge of the

types of students who have successfully matched at various programs. Most of them will be more than

willing to share that information with you.

Additionally, it is always helpful to speak with students in the class ahead of you. Now is the perfect time

of year for third-year students to begin inquiring, as the current fourth-year students have just completed

the match process and have all of the details about specific programs still fresh in their minds.

Rumors circulate widely throughout the residency application and interview season. People will tell you

that certain programs filter their applications based on applicants' board scores. While that may be true, it

is unlikely that you will ever know what that number is. You will hear that there are all sorts of formulas

used to identify how competitive your application is. Again, you will probably never see that formula

published anywhere.

The ultimate answer to your question, unfortunately, is that the match process remains slightly

mysterious. The wealth of information you had for choosing a medical school is not readily available for

this stage of training. However, the more you talk with people and surf the Internet, the more information

you will acquire.

My advice: Always aim high. If you have done well in medical school, you should be a competitive

applicant. Have confidence in yourself, but also ask for assistance from advisers and faculty to set

realistic goals. Because the information is not easy to find, it is critical to speak with people who have

done this before, both as applicants and as faculty. It is never too soon to schedule an appointment with a

faculty member to discuss your career. It is never too soon to send an email message to an older student

who just went through the Match to get advice. Most people are willing to help, because most assuredly,

someone also helped them along the way.

Page 93: Medscape articles

How Can I Improve My Surgical Skills?

92

How Can I Improve My Surgical Skills? Question

How can I improve my surgical skills as a medical student? Are there courses I can take?

Response from Danielle Gottlieb, MS, MD

Surgical training involves the acquisition of both technical and nontechnical skills. During a surgical

residency, technical skills improve with practice, and surgical judgment deepens as a result of exposure

to evidence, patients, and outcomes. That is why surgical education has remained an apprenticeship

model since it was developed by Sir William Halstead more than a century ago. Surgical technique is a

product of hours spent with expert teachers, and surgical judgment is a product of seeing thousands of

patients and thousands of operations.

However, recent changes in resident work hours, the popularity of minimally invasive surgical

approaches, and the litigious nature of medical errors have led to more didactic teaching for surgical

residents and a greater reliance on technologies for acquiring surgical skills. In short, this is a time of

transition for surgical education, shifting from a "brute force" approach to a more didactic and simulated

experience.

Modern research in surgical education has examined the steps involved in attaining motor skills. The

bottom line: deliberate practices makes perfect, or at least better. The Fitts-Posner theory of motor skill

acquisition sequences the steps of technical skill adoption[1]:

Stage I: Cognition. You must understand the task. Buy a basic book on knot-tying and suturing

techniques, and one that describes the steps and relevant anatomy involved in many basic surgical

procedures. (See Suggested Reading below.) The New England Journal of Medicine has a series of

videos on procedures (available with subscription)[2]; your medical school may also have videos or

multimedia presentations. Some resident and continuing education courses address surgical skills, but

due to the apprenticeship nature of surgical education, short courses are relatively rare (residency = the

long course).

Stages II and III: Integration and Automation. Practice! One suture company makes boards for knot-

tying practice. As a medical student and intern, I practiced tying knots on this board, first with the large

rope provided by the company, then with smaller suture, and I progressively practiced new and more

difficult skills. In the next few years of residency, I made the board into a model carotid artery and

practiced sewing a patch into a glove using 6-0 Prolene. Many people also practice with fruit (oranges or

bananas). Use the laparoscopic simulators in your school's surgery department, and practice holding the

camera. Carry silk ties in your pocket. Practice the movements until they become automatic.

Since surgical training is a "long course," it's important to set realistic technical goals for the introductory

medical student level. Familiarize yourself with:

1. Holding a surgical instrument;

2. Knot tying (2 hands first!);

Page 94: Medscape articles

How Can I Improve My Surgical Skills?

93

3. Basic wound closure principles and techniques ("approximate, don't strangulate!"; subcuticular,

simple interrupted, running, vertical mattress);

4. Suturing a drain, chest tube, central line into position securely;

5. Recognizing patterns in operations (ie, steps in transecting a vessel bigger than a capillary); and

6. Assisting movements (taking clamps off, "following" with suture).

Finally, make yourself available and solicit feedback. Ask a resident or attending to watch your practiced

movements and to make suggestions to help you improve. Offer to suture wounds late at night in the

emergency department. Show your interest and curiosity. And remember, it's a marathon, not a sprint!

Page 95: Medscape articles

How Can I Find a Mentor?

94

How Can I Find a Mentor? Question

I have so many questions about my future career. How can I find someone to help advise me?

Response from Megan L. Fix, MD

The journey you are undertaking to become a doctor is not meant to be accomplished alone. It is

important to find mentors who can help you get through medical school and beyond.

The first step to finding a mentor is to ask yourself certain questions: What are my interests? Can I see

myself doing "X"? Am I interested in researching "Y"? Pay particular attention to professors at your

institution and identify ones you respect, then find out a little more about what they do. Only you know

what inspires you. Once you find something, write it down and make a list of potential interests.

The next step is deciding what your needs are from a mentor. You may be looking for someone to help

you maintain your excitement about medicine. You may be trying to decide what career path to take. You

may need a mentor to show you how to find balance between career and community service. You may be

interested in research, and need someone to help develop your critical thinking skills. Once you have

decided on an interest, write down your list of expectations for your potential mentor.

The third step is to identify potential mentors. One way is to meet with your dean and say, "I'm interested

in doing some research on X, do you know of someone who would be a good mentor for me?" Another

approach is to find an interest group at your school and identify the advisor for that group, then ask him or

her who might be a good mentor. If you were inspired by a lecture, you can walk up to the podium at the

end and say, "I'm very interested in your work -- would you have any time to meet with me to discuss

opportunities to get involved?" Again, keep a list and collect emails and phone contact information.

The fourth step is making contact with your mentor. Believe it or not, most faculty would love the chance

to talk with you about what they do. It is a compliment to them that you are interested in their work. I

would encourage you to send an email stating, "I am very interested in your [research or specialty or

lecture topic]. I wonder if we could set up a time to meet and talk about opportunities for me to get

involved." Your potential mentor does not have to be the most respected person in their field, but they

must be someone you can talk to and be honest with about your goals.

Remember to think outside the box. You may find that there are no mentors interested in what you like at

your school. The next step, then, is to meet with a faculty member you trust and ask where you can find

resources. In my case, I was interested in exploring a topic that was not mainstream in my field. I asked

my program director to recommend someone, and he gave me the name of the Associate Director for

Graduate Medical Education! Even though I had only heard her speak once before and I had never had a

conversation with her, I went into her office and scheduled a meeting. That was 2 years ago, and we have

been meeting and working on projects together ever since then. I was initially intimidated by the fact that

she was much higher up on the medical hierarchy and not in my field, but what I found was that she was

approachable, interested, enthusiastic, and most of all, the perfect mentor for me.

Finally, don't give up and don't forget to follow up. If you can't find someone at your school to mentor you,

try one of the many mentoring services. For example, the American Medical Student Association's Career

Page 96: Medscape articles

How Can I Find a Mentor?

95

Development Program has a mentoring service for members. Many other medical associations offer

similar services on their Web sites (see below).

You may find that as you progress through school, your interests change and you might need more than

one advisor. Mentors understand this, and a true mentor wants what is best for you. Be appreciative,

enthusiastic, and honest with your mentor, and you likely will develop a relationship for medical school

and beyond.

Resources

American College of Physicians Online Mentoring Database

http://www.acponline.org/srf/med_mentor.htm

American Psychiatric Association Minority Mentoring Program

http://www.psych.org/edu/other_res/apa_fellowship/summerfellowship.cfm

MomMD Connecting Women in Medicine

http://www.mommd.com/medstudent.shtml

Page 97: Medscape articles

How Should I Choose a Residency Program?

96

How Should I Choose a Residency Program? Question

Now that I've decided what specialty I want to pursue, how can I figure out which particular residency

program will be best for me?

Response From the Expert

Response from Danielle Gottlieb, MS, MD

Resident, Department of Surgery, Massachusetts General Hospital, Harvard Medical School, Boston,

Massachusetts; Research Fellow, Children's Hospital Boston and Massachusetts General Hospital,

Boston, Massachusetts

Residency is a time to gather a collection of experiences that will help you develop sound clinical

judgment. The best training programs are broad and offer high patient complexity. As you search for the

right program for you, keep the big picture in mind: This decision will influence many subsequent career

decisions, in both tangible and intangible ways. So it's important to find a good fit. Here are some tips:

1. Clearly define your own goals. Make a list of your most valued features in a program. These are some

examples of features and ways to measure them:

Geographic location of the hospital. Hospitals will tell you about their catchment area, which

refers to the population they serve (local area or distant, international referral base). A wider area

is generally better, as it will expose you a variety of patients and pathology.

Prestige of the program. US News and World Report's annual ranking offers a good indication.

This is only one source, but one to which people frequently refer. The annual report can be found

athttp://www.usnews.com/usnews/health/best-hospitals/tophosp.htm. Prestigious programs are

correlated with the advantages of large academic centers: well-trained, research-oriented faculty,

diverse patient populations, and a broad, strong education.

Residency graduates. Find out the percentage who went on to fellowships or faculty positions;

many programs provide this information on their Web sites. Understanding the trajectory of past

residents will help you understand whether your career plans fit with those of the graduates who

usually complete a particular program. Also, determine the percentage of residents at a given

program who passed the boards on their first attempt. This information is found on residency Web

sites and can be obtained during the interview process.

Depth and breadth of hospital system. Check a program's Web site to determine the number of

residents in the program, the number of procedures performed per resident, and the complexity of

procedures. Also look for the number of subspecialty residencies or services (such as pediatric

burns, orthopaedic oncology, level of accreditation for trauma, etc.) This information is a good

topic for questions during informal meetings with residents during the interview tour. The tour is a

great time to ask residents about their life in the program and what they see and do.

Page 98: Medscape articles

How Should I Choose a Residency Program?

97

2. Study the available residency programs. Collect some initial data about those features that matter most

to you. Start a spreadsheet, making a row for each program and a column for each feature; put the

features in order of importance.

3. Consider your lifestyle preferences. Usually, urban areas have higher patient complexity. However, this

advantage may be offset by the cost of living in a city, especially if you have a large amount of debt.

4. Interview at programs that interest you. The interpersonal nature of the interview visit yields particularly

good information about the organization's culture. Look at:

The top of the hierarchy, particularly the clinical and research interests of the department chair

and faculty, as well as where they trained. If you have research interests, aligning yourself in a

department with research expertise is valuable. The ongoing academic activities of the faculty will

give you a sense of the opportunities you may encounter. Also, look at the age distribution of the

faculty to see whether there is a good representation of new techniques and management styles,

as well as multiple perspectives. Training programs have philosophies; try to find a program with

a philosophy of training and an approach to new techniques that suits your style.

The residents. A common but relatively unproductive question that many applicants ask the

residents is whether they are "happy." Instead, ask about more objective measures that you can

rank on your scale, like how much time residents spend outside the hospital, or how the workload

is offset by the educational aspects of the program. Determine whether the residents chose this

program for reasons similar to yours. Notice the level of formality between residents and faculty;

do residents feel that the faculty invest in their futures? Get a feeling about who the residents and

faculty are, and whether you would enjoy the culture and environment. Once you get there,

residency will be more like a marathon than a sprint, so you should make sure that you will enjoy

the company.

The hospital system. Determine the number and type of hospitals where residents spend

rotations; exposure to varying levels of patient acuity and complexity is important.

5. Score the programs that you are considering. Rank each one for each factor on your spreadsheet. Try

to record scores soon after you visit the programs, when the information is fresh.

6. When your spreadsheet is complete, see how you feel about it. Many important decisions cannot be

made purely mathematically. Allow your "gut feeling" to interpret the scores and direct your choice.

7. Use additional strategies, such as doing an "away" rotation and making "second-look" visits, to solidify

a choice or to choose between close competitors. Both strategies demonstrate your interest in the

program.

Page 99: Medscape articles

What Is the Best Way to Learn Procedures? Doing Them

98

What Is the Best Way to Learn Procedures? Doing Them Question:

What is the best way that I can learn to do procedures during medical school?

Response from Thomas E. Robey, MD, PhD Resident, Emergency Medicine, Yale-New Haven Hospital, New Haven, Connecticut

If you are part of the same educational lineage of most physicians in the history of Western medicine, you

are familiar with the "See one, Do one, Teach one" approach to learning medical procedures. The idea is

that you watch one procedure before you do one, and then you are prepared to teach the next person

how to do it. Simple, right? But anyone learning to intubate or to place a central line knows this is easier

said than done, especially when you are one of the most junior members of the medical team. How many

of you have been a part of a conversation akin to the following?

"Hello, I'm student doctor Chen. I'll be draining that fluid in your belly for you today."

"Nice to meet you. Doctors are so young these days! How many times have you done this?"

"Well, to be honest, you are my first. But I've watched videos about it, practiced on a mannequin, and

watched my resident do it on a patient down the hall yesterday!"

Imagine yourself as the patient. You would expect the best person available to do the paracentesis. That

a fresh medical student wants to stick a giant needle into your belly is a reasonable source of anxiety.

This situation stresses the student as well. You want to do the right thing for the patient and are worried

about complications; all the while, you know you need to learn these skills to succeed in your future

career. There is a reason medicine follows the apprenticeship model of clerkship and residency: it is best

learned by practicing on humans! Here are some strategies to help you succeed with more procedures as

a medical student.

1. Placement. If you want to learn procedures, sign up for rotations and clerkship sites known to

encourage medical student participation. Anesthesia, emergency medicine blocks, surgical

subinternships, and busy county hospitals are often good options. Rotations in clinical settings

where there are few residents are also an option, because you may learn directly from present

attendings, making the patient at more ease. After you've secured a procedure-rich rotation,

make sure you "show up." This means going out of your way to be present when procedures are

being done. Stay late, delay your lunch, and put away your mobile device!

Once you work with a resident, that person will be more comfortable with you doing the task.

Keep in mind that it's helpful, but often not sufficient, to say that you watched so-and-so do the

procedure before. Different people have different styles and skill levels. Channel any frustration

about not putting your hands on the patient to enthusiasm for your supervisors' willingness to

teach; you can always learn something from watching. (Gently remind them later that you are the

Page 100: Medscape articles

What Is the Best Way to Learn Procedures? Doing Them

99

kind of person who learns by doing! But this will check off the "See one"" step in learning

techniques.

2. Preparation. As the medical student dutifully concerned about the welfare of the patient on which

you are about to complete a procedure, you owe it to the team and the patient to be prepared. At

the beginning of rotations where you might be doing interventions, read books like Roberts and

Hedges' Clinical Procedures in Emergency Medicine, watch videos on The New England Journal

of Medicine Website, and look up procedure descriptions on eMedicine.

Test yourself by writing down the steps and going through the motions with your supervisor.

Another way to prepare is to get your hands on an educational ultrasound machine. Visualizing

anatomy today will help you do the procedure tomorrow. Increasingly, more interventions are

done under ultrasound guidance. Learning to use the machine is vital to doing such procedures.

3. Practice. There are different ways to practice procedures before you get to the real thing, ranging

from poking needles through cardboard boxes to intubating a recently deceased patient. Many

schools provide training with computerized simulators such as SimMan® to help students and

residents develop the manual dexterity needed for interventions. In addition, pig tracheas are

used to teach tracheostomy, fellow medical students are useful for intravenous line placement,

and freshly dead (not preserved) educational cadavers can be used for central line placement.

And you may have attending physicians in the intensive care unit or emergency department who

call you over to "practice" cardiopulmonary resuscitation or intubation on patients who just died.

With regard to practicing on newly dead patients, the Council on Ethical and Judicial Affairs of the

American Medical Association recommends that physicians request permission from the family of

the deceased prior to performing procedures on recently deceased patients.[1] My experience and

the literature[2] suggest otherwise; this important vestige of an older tradition for learning

procedures only occasionally follows these guidelines. When confronted with such an opportunity,

you have to be prepared to learn from it. Keep in mind that use of mannequins or simulators to

teach procedures may be helpful to reinforce the steps learned in reading about the procedure or

watching a video, but rehearsals are not a replacement for interventions on living patients.[3]

4. Confidence. Whether you are scared out of your wits, it is important to exude cool confidence in

front of the patient. And if you feel ready to try a new procedure, make sure your supervisor

knows it. At the same time, don't be foolhardy in your approach. (You have no right to be

confident without completing the previous 3 steps.) Know when to step back and say, "This isn't

working for me. I think you should take over." Your resident will be quick to help you out, whether

that means guiding your hand or taking over completely. Some fields and training sites require

residents to be signed off after 3 procedures; for patient safety, the "Do one" step has evolved to

"Do three!"

5. Graciousness. After your successful procedure -- or even an attempt -- be sure to express your

graciousness to the patient for permitting your doing the procedure, and your supervisor for

helping you through it. Both people will be more likely to let you or future trainees "practice" their

procedures with them. Pretty soon you will be in a position to "Teach one."

Like many other things in medicine, learning procedures requires study and hands-on practice.

And if after giving it a shot, you are bored by, never get comfortable with or cannot stand doing

procedures, consider exploring disciplines in which physicians need not stick tubes in vessels or

cavities. For me, the gratification of hearing the "pfffft" of a needle decompressing a

pneumothorax or the relief patients feel after being drained of 7 L of ascitic fluid is enough to keep

at it.

Page 101: Medscape articles

What Is the Best Way to Learn Procedures? Doing Them

100

Medical School Admissions, Residency Match, and Medical Education

Residency Match: Avoid Getting Burned

6. Jessica Freedman, Medical Student, Emergency Medicine, 06:25PM Oct 24, 2009

Dear Applicant,

It is obvious to all of us here at Academic Medical Center that you are an outstanding residency

applicant, and we would be delighted if you matched with us and would be proud to play a role

in your professional development. We have ranked you "to match" in our program and

encourage you to contact us to express any specific interest you may have in our program.

Regards,

Academic Medical Center Program Director

You read the email in disbelief and think you are set. Academic Medical Center is your top

choice program. You hit the reply button to say that you will be ranking Academic Medical

Center #1 on your list and submit your rank list with confidence.

To your shock on match day, you do not match with Academic Medical Center but instead match

with your #11 choice. You feel a little cheated and wonder why the program director at

Academic Medical Center sent you this misleading email. You start to think that this

communication from Academic Medical Center may have been a violation of the Match

Participation Agreement (MPA).

In my work with residency applicants from a variety of schools applying for different specialties,

I am often surprised that some of the correspondence and communication from programs is

remarkably similar to the example above from the fictitious Academic Medical Center. I am

even more surprised when applicants receive overwhelmingly positive feedback from programs

where they don't match. I decided to blog on this topic when I read an article by Carl Erik Fisher

in the Journal of the American Medical Association in which he reflected on his own experience

as a residency applicant going through the match.

The MPA is designed to make the match fair for everyone involved.Item 6.0 of the MPA,

Restrictions on Persuasion of the MPA, specifically states: "One of the purposes of the Matching

Program is to allow both applicants and programs to make selection decisions on a uniform

schedule and without coercion or undue or unwarranted pressure. Both applicants and programs

may express their interest in each other; however, they shall not solicit verbal or written

statements implying a commitment. Applicants shall at all times be free to keep confidential the

names or identities of programs to which they have or may apply."

I find that many interviewers and program directors may violate the terms of this agreement,

sometimes unwittingly. Interviewers often casually ask applicants during interviews, "So, where

else did you apply?" or suggest, "If you are really interested in matching here, please be sure to

let us know." Other programs routinely call applicants who are within "matching range" to

recruit them after they interview. Does this constitute persuasion or pressure? The 2008 NRMP

Program Director Survey suggests that post-interview contact does not have a great impact on

ranking, but is this really true? Earlier research on this topic, cited in Dr. Fisher's article, suggests

otherwise.

Page 102: Medscape articles

What Is the Best Way to Learn Procedures? Doing Them

101

These subtle and not so subtle manipulations place applicants in difficult positions. Applicants

are often confused, and these recruitment efforts on the part of program directors create an

atmosphere of distrust and suspicion. Applicants are in a subordinate position during these

interactions; they feel vulnerable and anxious. Of course, they often feel obligated to tell a

program what it wants to hear by saying, "I loved your program and I will be ranking you really

highly." Or, applicants may feel pressured to tell a program that they will be ranking it #1, even

if that's a white lie. Sometimes applicants are so nervous about matching that institutions'

recruitment efforts may influence the way they rank programs.

So, how should applicants deal with these situations? If you receive a residency match "love

letter," take its sincerity and truthfulness with a grain of salt. You certainly will want to respond

graciously, but do not say you are ranking the program first if that is not your intention. As for

fielding questions or comments that may violate the MPA, I suggest being diplomatic and

somewhat vague without confronting or offending your interviewer. For example, if the

interviewer asks where else you are applying or interviewing, it is acceptable to answer, "I am

applying to and interviewing at a variety of programs, mostly on the West Coast" or whatever

situation applies to you. If pressured to express specific interest in a program, it is fair to say, "I

am very interested in this program and have not yet decided on my final rank order, but I will be

ranking your program highly." As with the "love letter," do not tell a program that you will be

ranking it first if this is not true.

Although some programs do indeed violate MPA standards, many programs behave ethically in

the match, and it is the obligation of all program directors and everyone who interviews

applicants to uphold MPA tenets. Values and ideals start at the top so, if those in positions of

power do not demonstrate ethical behavior, applicants are also encouraged to be insincere. Such

behavior not only violates the MPA but also corrodes some fundamentals of our profession:

honesty, good will and professionalism.

Page 103: Medscape articles

Do I Need To Do Research?

102

Do I Need To Do Research? Joel Schofer, MD, Emergency Medicine, 07:50PM Jul 5, 2010

Students often wonder about the importance of research and whether they need to get involved in

a research project while they are a student. This is my opinion...

I don't think students need to do research. What they need to do is find something they enjoy

that makes them stand apart from the competition. If research is what you enjoy, then use

research to make yourself stand out from the crowd. If you don't enjoy research, then use

something else. It could be publishing case reports, writing articles for a medical newsletter,

doing an on-line blog, running for leadership positions in medical student associations, doing

community service, etc. I don't think it matters what you do as long as you like it and it is

something that can be put on a CV somewhere.

If you are like a lot of medical students and you are unsure whether you would like doing

research, you should strongly consider getting involved in a research project being run by a

resident or attending in the specialty that interests you. Most all departments have some kind of

research going on. In addition, almost everyone is looking for someone to help them out. All

you have to do is ask.

In the end, if the idea of doing research is not enjoyable to you, don't feel pressure to do

it. There are plenty of other things you can do to distinguish yourself. If you're lucky, you might

even win the research award at your future residency without having done any research. I did!

Page 104: Medscape articles

You will Survive

103

You will Survive Roger KA Allen; Gayathri Rabindra; Robin Som; Yee Teoh; Susan Kersley; Tiago Villanueva; Maryam

Ahmed; Farhat Mirza; Sam Thenabadu

Posted: 02/02/2010; Stud BMJ. 2009;17:b3175 © 2009

Abstract and Introduction

Abstract

Tips for students and junior doctors

Introduction

The BMJ's social networking site, doc2doc, has just published an e-book that resulted from an online

discussion on how to survive as a junior doctor. Here is a selection of tips. Download the whole e-book for

free at http://doc2doc.bmj.com/.

Using BODEX and SOAP

Remember the mnemonics BODEX and SOAP. BODEX stands for bloods, obs, drugs chart,

electrocardiography, and x rays or imaging. This provides you with a good safety net for ward rounds.

When writing in the notes, remember SOAP. S stands for subjective. How is the patient feeling? Retake

any relevant parts of the history—for example, does he or she have chest pain? O stands for objective.

How does the patient look? Write down the observations and your examination findings. A stands for

assessment—your impression of what's going on—for example, "pulmonary oedema improving, no new

issues." Lastly, document your plan.

Will Buxton, F1, Sussex

Cannulating Difficult Patients...

Oedematous Patients

Grossly oedematous patients are difficult to cannulate because it is hard to even see a vein to puncture.

You can get around this easily by placing the tourniquet tightly, high up on the patient's arm, and then

pressing very firmly but gently on the dorsal surface of the patient's hand for, at the very least, one

minute—the longer the better, though. This pushes all the fluid away and should leave you with a clear

view of a juicy fat vein. You must have your needle ready, though, because the fluid can return quickly

and obscure the vein again.

Warm Water

Warm water can make veins visible and palpable. Get a small bowl or beaker (the ward will have plastic

ones) and fill it with water that's hot but bearable. Naturally explain to the patient what you would like to

do and why you are doing it. Then place the tourniquet high up on their arm, and ask them to submerge

their hand in the warm water. Keep it there for five minutes. The heat should bring the veins up for you to

puncture.

Gloves

Page 105: Medscape articles

You will Survive

104

Gloves can have a great tourniquet effect—not by using them around the arm but rather by getting the

patient to wear one. Estimate what size glove the patient might wear, perhaps by comparing with yours,

then get a glove one size smaller. Ask the patient to put the glove on, explaining that it will be quite tight.

Naturally, explain what you are hoping to achieve. Keep the glove on for at least five minutes. Then

remove it and cannulate away. You can combine this with the warm water effect.

Robin Som, CT1, Cambridge

...and How Not to Do It

Tell the patient that it's just a tiny scratch before you go digging into their flesh in every possible

direction.

Prepare your patients by telling them that they've got difficult, narrow, and wiggly veins.

Tell the patient that you got into the vein, but the tiny little valves on the veins are blocking your

plastic cannula from moving in.

Tell your patients that they have fragile veins when you give them a great big haematoma.

If all else fails, call the friendly on-call anaesthetist.

Don't call the same anaesthetist twice in the same day. Never call them within the same hour.

Get your fellow house officer to be the bad guy.

You know that you're in trouble when the on-call anaesthetist tells you that they are not a

cannulation service...

...and then you think subcutaneous morphine or fluids, supplemented with regular diclofenac

intramuscular injections, might just be the easier or, realistically, the only option you have left.

Yee Teoh, F1, Kent

Notes and the Ward Round

A three or four line problem list at the top of each entry helps give me an idea of what I have pending for

the patient and helps structure my plan—especially when alone on a ward round. It gets easier if you do it

every day.

Tim Baruah, ST2, London

Whenever I make an entry I print my name, with my job description, date, and time (to the minute). It is

legible, and I then sign. Many years later this may save my rear end. Some contemporaries are less

anally retentive, but I like my rear end intact.

Professor Roger KA Allen, thoracic and sleep physician, Brisbane, Australia

A Low Note

One of my lowest moments as a house officer was when I was on a cardiology ward round. I was the lone

junior surrounded by clever consultants and registrars. It was a few months into my first year as a junior

doctor, and I felt that I was in the swing of things. Just as I was starting to write the notes for a patient, my

registrar grabbed the notes from me and started writing. I felt so embarrassed—writing notes is one of the

few things that a house officer is expected to do without supervision, and yet it's something that I was

Page 106: Medscape articles

You will Survive

105

obviously rubbish at. And this registrar was really nice, so he wasn't being cruel. He was just obviously

frustrated at my incompetent note taking.

I have since learnt that writing notes is a more important job than it seems at first. When you're doing a

busy on call, a good last entry from a diligent house officer can make all the difference and save you

precious time that would otherwise be spent leafing through the entire set of notes.

So what makes a good note entry?

Write a problem list (see above).

How is the patient today? Note down vital signs, any history and examination that is done on the

ward round, as well as any discussions that you have with the patient (see SOAP, above).

Detailing what the patient is told can be useful, especially for anyone asked to see the patient on

call.

Write a clear management plan. Mark each task as "done" in the notes once completed.

Different consultants and registrars like different styles of note keeping, so find out early on what

they expect. If you are in any doubt over what has been said on the ward round, don't be afraid to

ask for clarification—it's far better than writing something that makes no sense to anyone else.

Gayathri Rabindra, GP ST1, London

Food

Don't drink too many caffeinated drinks, you will feel better with fewer. Don't be tempted by sugary foods

when stressed. Instead eat some protein with some fruit or other healthy combination. Take a break for

meals; don't skip them. Finally, when things are getting to you on a busy shift, take half a minute to take a

few deep breaths and release some tension before diving back into the jobs.

Susan Kersley, life coach and retired GP

Have food with you at all times, to avoid protracted periods of hypoglycaemia. That liquid yoghurt or those

all bran biscuits in the pocket of your white coat are priceless.

Tiago Villanueva, GP trainee, Portugal

Always have breakfast as you just never know when lunch will be.

Maryam Ahmed, CT1, Wolverhampton

Get Rid of the Bubble

Arterial blood gas analysis of a patient with severe pneumonia showed a normal PO, which I was initially

satisfied with. However, on reviewing the patient I found him clinically worse than the blood gas

suggested. I repeated the arterial blood gases test myself and found that patient was severely hypoxic. I

later found out that the person who ran the first blood gas analysis had not removed the gas bubble from

the syringe. Lessons learnt: always remove the bubble from blood gas syringe, and treat the patient not

the test result.

Farhat Mirza, F2, Gillingham

Page 107: Medscape articles

You will Survive

106

"The good doctor will treat the disease. The great doctor will treat the patient with the disease." Wish it

was my quote but some bloke called Osler with his damn nodes got there first.

Page 108: Medscape articles

Spicing up medical education

107

Spicing up medical education Henry P O’Connell

Posted: 08/26/2009; Stud BMJ. 2009;9(6):b2390 © 2009 BMJ Publishing Group

Introduction

The acronym SPICES refers to six main concepts in medical education—student centred teaching,

problem based learning, an integrated curriculum, community based teaching, electives with a core, and

the use of systematic methods ( table ). An awareness of these principles means that medical students

can take a more active role in their learning. Students who are well informed about medical education

principles, such as the SPICES criteria, are more likely to be able to provide constructive feedback about

their own medical education experience, contributing in the long term to course improvements.

Table. The SPICES Model and the Traditional Alternative

SPICES Traditional approach

Student centred Teacher centred

Problem based Information gathering

Integrated Discipline based

Community based Hospital based

Electives with a core Uniform

Systematic Apprenticeship

Different aspects of the SPICES model are adopted to a greater or lesser degree by medical schools. It

has been suggested that each medical school should be scored according to how much of the SPICES

model they use in their curriculum.[1] However, there is a dearth of evidence on how different medical

schools perform with regard to the SPICES criteria and how they compare with each other. The level of

adoption by a medical school of the SPICES criteria can be measured objectively in terms of the

scheduled number of hours allocated for each educational strategy. What are the six concepts in the

SPICES model, and what are the advantages and disadvantages of each?

Student Centred Teaching

This moves the emphasis of the medical curriculum away from being teacher centred and towards being

more student centred.[2] Two approaches that can help this change are the creation and use of study

guides[3] and the adaptive curriculum.[4] Study guides show students what to learn, identify educational

resources available to help, and give activities to help students understand and remember the material.

Page 109: Medscape articles

Spicing up medical education

108

The adaptive curriculum involves working collaboratively with everyone involved (students, dean of

faculty, and all other staff) to create a flexible and relevant curriculum.

Advantages—The emphasis is on what the student learns and not what the teacher teaches, and

students are more motivated. Students are also better prepared for continuing education after graduation

because they are allowed to be more actively involved in their own learning and so take more

responsibility for it.

Disadvantages—Teachers who have experienced a predominantly teacher centred approach may find it

difficult to teach in a more student centred approach. Likewise, students may find a student centred

approach more threatening and demanding initially, especially if their previous experience was in the

teacher centred model. More demands are also placed on teachers in the preparation of a wide range of

learning experiences and resources, as opposed to the relatively easily prepared and perennially

repeatable lecture series.

Problem Based Learning

First introduced in McMaster University in Canada in the 1970s by Howard Barrows, problem based

learning is a student centred approach to learning that aims to help students develop critical thinking and

clinical judgment and to provide a clinically relevant focus for their learning.[5] In this process students are

usually facilitated by a tutor ( box 1 ).

Box 1. Problem Based Learning

1. Clinical scenario or problem presented to students 2. Students determine learning outcomes and objectives 3. Students decide knowledge needed to solve problem 4. Students attempt to achieve self determined learning outcomes (individually

or in small groups) 5. Allocation of tasks among group members 6. Students present their work or contribution to group 7. Further learning points may be generated 8. Information and competencies synthesised

Advantages—Real world clinical scenarios are used, preparing students for clinical practice. This form of

learning encourages early development of decision making skills, a holistic approach to clinical problems,

and the development of team working skills.

A study that compared costs in faculty time of problem based with conventional pathology programmes

indicated that problem based learning is feasible for schools with class sizes of less than 60-100 students

a year.[6] It is not possible to look at cost and effectiveness in a meaningful way in this article. The main

costs relate to hours of time for teachers and facilitators to run PBL sessions. The main effectiveness

relates to the positive nature of the student experience, something that is more difficult to quantify. The

ideal total class size, as per the study quoted, is 60-100, with PBL groups within the class generally

consisting of about eight students.

Page 110: Medscape articles

Spicing up medical education

109

Task based learning may be used if problem based learning is difficult in a clinical setting.[7] In task based

learning, students’ learning is based around real cases encountered in different healthcare settings. Using

study guides to help learn about clinical conditions, task based learning integrates theory with practice.

Students can individually customise their learning and can learn when they need to. One recent study

showed that students involved in a task based, community oriented teaching model of family medicine

outperformed a control group in terms of acquisition of knowledge and skills, and reported high degrees

of relevance and student satisfaction.[8]

Disadvantages—Some students may feel insecure, especially if used to an information gathering

approach. Problem based learning, with requirements for print based scenarios and small group work,

may be difficult to introduce in clinical situations. Task based learning was developed to tackle this

shortfall.

PBL may be difficult to implement in the clinical situation because the print based scenarios and small

group work do not lend themselves to the clinical context. This difficulty was resolved with the introduction

of TBL. TBL is an educational strategy that focuses student learning around real cases that the students

encounter in the wards, out-patient departments, ambulatory care clinics, or general practice surgeries.

The students make use of study guides for each of the clinical cases that help them to learn about the

condition for themselves.

Integrated Curriculum

Integration of the curriculum promotes a holistic and cross discipline approach to patients and their

problems. It can also help promote learning in context. Discipline based learning focuses on clinical

problems that are largely confined to individual medical specialities, for example, cardiology or psychiatry

cases. Box 2 lists some of the methods in integrated curriculums.

Box 2. Teaching Methods That Promote an Integrated Curriculum

Problem based learning Task based learning “Hot case” learning—Used in primary care, this involves learning around

recently encountered real world clinical scenarios Spiral curriculum—Topics revisited, increasing levels of difficulty, new

learning related to previous learning, student competence increases Multiprofessional learning—Students of different healthcare professions learn

together

Advantages—Integrated curriculums promote learning in context. Higher learning objectives, such as

application of knowledge and problem solving skills, are more easily met and irrelevant information is less

likely to be included in the curriculum.

In a spiral curriculum topics are revisited during the course; difficulty levels increase as topics are

revisited; and new learning is related to previous learning, increasing students’ competence. An aim of

multiprofessional learning, where students from different healthcare professions learn together, is to

promote multiprofessional team working in graduates, but evidence is lacking as to whether this works.

Page 111: Medscape articles

Spicing up medical education

110

Medical courses are less fragmented in an integrated curriculum. This encourages a more holistic view of

a patient’s problems; increased motivation levels among students; better educational effectiveness of

teaching because learnt material is applied; more emphasis on higher learning objectives, such as

application of knowledge and problem solving skills; and promotion of staff communication and

collaboration, with a more efficient use of teaching resources.

Disadvantages—The fundamentals of a discipline may be neglected; some topics may be omitted;

teachers may be less enthusiastic and less comfortable when not teaching in their own discipline;

discipline based teaching may be cheaper; and students may develop a clearer picture of a discipline as

a career in the discipline based method.

Community Based Learning

Students’ education is far more geared to specialised experience in a hospital, and this is something that

needs to be tackled, by increasing community based learning. Recently, medical education has shifted

from the perspective of hospital inpatient to healthcare delivery in community settings. Two terms are

used in this area: community based education and community oriented education. In community based

education, students are taught in community facilities, such as day hospitals and general practices.

Community oriented education emphasises the planning and delivery of diagnosis and medical treatment

outside of the acute hospital setting. Community based treatment is more feasible than before because of

medical technology.

Advantages—This method of teaching is often cheaper than in hospital, and most people with medical

disorders are treated in the community. In the community, students are less likely to meet patients who

have already been interviewed and examined by medical students in the past. Patients who have had

less contact with students in the past present more authentic clinical problems in that they are less likely

to direct the student to what they may perceive as the correct parts of the history or examination.

Students get a broader and more realistic introduction to all aspects of a healthcare system, such as

screening programmes and continuity of care.

Disadvantages—It can be difficult to recruit and fund primary care doctors. They must be able to change

their practice to balance clinical time with teaching. Some learning experiences, such as in surgery,

cannot be gained if the student learns only in the community setting, and some students may wish to

spend more time exploring specialist areas as opposed to primary care as a career choice.

Electives With a Core

The core content of the course is determined by the outcomes the students must achieve by the end of

the course. Using this system, some of the material conventionally taught in the undergraduate years or

in basic training is moved to the postgraduate phase of training. The time freed by the more rigorous

definition of a core curriculum may be used by students to study courses of their own choosing, known in

the United Kingdom as student selected components. The more traditional approach is of each student

completing a uniform or identical course. This concept was developed in response to an overloaded

curriculum. It focuses on defining the “core” of a curriculum and implementing it.

Advantages—Using a core curriculum, time can be freed for students to study extra topics or subjects of

their own choosing, referred to as student selected components. Furthermore, some topics traditionally

taught in undergraduate years may be moved to postgraduate years. This system gives students more

Page 112: Medscape articles

Spicing up medical education

111

responsibility for their own learning and to facilitate career choice. Electives may also help students meet

their individual aspirations and help bring about changes in attitude because of exposure to real world

clinical problems and the often challenging social and cultural problems that may contribute to such

problems.

Disadvantages—Teachers who facilitate the extra parts of the curriculum can become overloaded with

work or unable to support the student adequately. Electives may impinge on other coursework. These

components may be difficult to assess fairly and uniformly.

Systematic Methods

Two approaches to making the curriculum more systematic are outcome based education and curriculum

mapping. In outcome based education, student learning focuses on predefined learning objectives.

Curriculum mapping is a comprehensive approach to designing and delivering medical curriculums by

defining what is taught, how it is taught, when it is taught, and when it is assessed or examined. The

curriculum map makes the curriculum transparent to all students, teachers, and the public. Areas in a

curriculum map include the expected learning outcomes, the curriculum content or required areas of

expertise, assessment procedures, learning opportunities, learning resources, a timetable, the staff

involved, and measures for future curriculum development.

Advantages—Exposure to a wider variety of health problems should lead to more awareness and the

understanding of the most important competencies. It is much less likely to miss required areas of

knowledge and competencies than the traditional apprenticeship model of medical education.

Disadvantages—It is less easy to arrange than the traditional apprenticeship model of education and it

may also be easier to ensure continuity of teaching.

Conclusions

Many of the ideas in the SPICES model of medical education are already in place to some extent in

medical schools around the world. An evidence base for effectiveness of the newer approaches is only

emerging and will take several years to accumulate.

However, the newer approaches seem to have some validity compared with the often at times ineffective

and anachronistic models of learning and curriculum development employed in more traditional settings.

Therefore, rigorous and regular application of concepts within the SPICES model is recommended in the

development of current and future medical curriculums, and this is likely to lead to better student

outcomes and satisfaction. Medical students should be actively involved in the process of advocating for

and introducing the innovative principles involved in the SPICES model of curriculum development.

Page 113: Medscape articles

My First Day as a Doctor

112

My First Day as a Doctor Omar Barbouti; Nida Gul Ahmed; Clinton Vaughn; Ali S Hassan; Tabassum A Khandker; Yasmin Akram

Posted: 11/25/2009; Stud BMJ © 2009

Abstract and Introduction

Abstract

This year's doctor intake started their first jobs earlier this month. Omar Barbouti and colleagues give an

account of their experiences.

Omar Barbouti

My alarm clock rang at 6 am. I automatically slapped the snooze button, having been used to getting up

at around midday for the last month. In fact, I had just got back from a long vacation with my family and

was still slightly jet lagged. Ten minutes passed and the dreaded alarm went off again. I lay there for a

few minutes thinking; I had put work and medicine to the back of my mind while on vacation, but reality

started to kick in. I felt my heart rate go up and I got a burst of adrenaline that made my stomach feel like

it was doing cartwheels; it was the same feeling I had before finals. I thought about what this day meant—

the first day as a doctor, the first day of my career, the first day of not being able to say, "I'm just a

medical student."

I got out of bed, had a shower, and got ready. I didn't eat any breakfast. I drove across south London,

arriving an hour later at the hospital. The first part of the day involved meeting the other new doctors, then

lots of paperwork, and health and safety lectures. After an IT training session I was sent to the ward,

where I arrived feeling nervous, like a medical student on the first day of a placement. I located the house

officer I would take over from. After a long conversation about the job, what to watch out for, and what to

expect, she gave me a brief handover. She then handed me the bleep—this felt like a defining moment,

like I was being given the baton in a relay—and I was now responsible for 15 or so inpatients. The day

ended, and as I left the house officer told me that the ward round would start at 7 30 am the next day.

I drove home that night, slept early, and arrived early the next day and met the consultant and registrar.

After very brief formalities they got cracking with the ward round. The senior house officer was due to start

the following day so I was on my own to guide the ward round; however, they were easy on me and didn't

expect me to know much about the patients. After I had written up all the notes and ordered bloods,

several nurses approached me in turn with requests, "Doctor, we need fluids for Mr B." My mind went

blank—all those lectures on fluid balance and I still couldn't think of what to give. I froze for a few seconds

until the nurse suggested I prescribe normal saline. I then had a moment of genius and decided to check

the patient's urea and electrolytes, which were normal, and hey presto I wrote my first prescription. The

rest of the day was hectic—as soon as I finished one job, I would get another bleep from a nurse

requiring a cannula to be sited or a drug chart to be rewritten. I decided to get organised the following day

and came in with a clip board.

I was on call the very first weekend. I was bleeped by the radiology department, who wanted a doctor to

be present as they administered contrast for a computed tomography pulmonary angiogram. When I

arrived a technician said, "There's the box with the anaphylaxis equipment." I stood there, thinking to

myself, "I'm actually a doctor, I'm not here just to watch, this is it." And so began my career; I look forward

to my next day and my next challenge.

Page 114: Medscape articles

My First Day as a Doctor

113

Nida Gul Ahmed

As I approached Lincoln County Hospital for my first day on the job as a foundation year 1 doctor (F1), I

experienced a variety of emotions. I couldn't wait to get started and felt all the excitement that only a first

day can induce. I felt nervous, responsible, and apprehensive. Questions such as "What will be expected

of me?" and "Will I be able to do the job well?" kept cropping up in my mind.

My first rotation was in critical care, involving time in theatre with the anaesthetists and in the intensive

care unit. I appreciated having had two weeks of shadowing before starting F1 and it helped take the

edge off my nerves. I had fond memories of an enjoyable anaesthetics attachment at medical school, and

ever since had considered a career in anaesthetics. Having critical care as my first rotation was a

privilege.

The day started at 8 am with a morning trauma list. I introduced myself to the consultant anaesthetist,

who was very friendly, and I felt the tight knot in my stomach ease slightly. The first task of the day was to

see patients for pre-anaesthetic assessment. I assessed two patients and enjoyed myself immensely.

Once the patients were brought to theatre I was asked to cannulate them; the fear of messing up

cannulation on my first day under the eyes of an expert was immense. However, it went smoothly (phew),

with the consultant giving me some very helpful tips throughout. It was a great morning with lots going on:

I inserted laryngeal mask airways and also helped to monitor the patients intraoperatively.

In the afternoon I went to gynaecology theatres. The operations involved spinal anaesthesia, and the first

question the anaesthetist asked me was, "Would you like to do the spinal block?" I just looked at him in

shock, having seen many but never done one myself. It was such a great buzz to do something so

exciting on my first day. I observed the first one and on the second was watched closely during the

procedure. When I saw droplets of cerebrospinal fluid emerge through the needle, it was exhilarating.

It is definitely worth all those hard months of studying for finals to be at the other end doing the job of your

dreams and to be learning something new every day.

Clinton Vaughn

Thursday came: the day I started work.

My on-call bleep went off at about 11 am saying "cardiac arrest." My heart stops and I listen intently—

where is it? How do I get there? It's muffled and then I hear—"testing." My heart slowly descends from my

mouth.

I set off to the wards. My senior house officer is out of action because she has to go to her own induction.

My friendly registrar leaves me with only four drugs to take out (TTOs) to do. Simple, right? Then I learn

about the people in blue uniforms who ask you to resite cannulas. It's fine because they all go in

successfully, hurrah. "Now back to the TTOs," I thought, but my med student then tells me that one of my

many patients (who, by the way, I have no clue about because I have never met him before) is

desaturating and breathless. I order four litres of oxygen on nasal prongs and happily watch his sats go

up to 97%. Even so, he is still faint while I try to cannulate him. My specialist registrar does an arterial

blood gas—"What's the result?" he asks. "I think the sample was venous," I reply. My med student then

reminds me that it's type two respiratory failure. Am I going to have to get used to students knowing more

than me?

Page 115: Medscape articles

My First Day as a Doctor

114

The day trundles on and I still have not done those darn TTOs. I get to work but then discover how slow

and inefficient I am at doing everything—never mind TTOs. One five minute job takes 30 because I keep

rechecking everything, fearing that I have made a mistake.

The nurses tell me that a family member wishes to speak to me about her mother, at which point I

shamefacedly admit that although I am looking after her mother I know nothing about her history so I ask

her to wait for my registrar, who is on call.

5 pm. I still have not completed those TTOs. The patients now have to wait until tomorrow for the

pharmacy to dispense the drugs, and hence they have to stay another day. I don't care at this point. I

want to go home. Will I ever be good at this?

The on-call bleep switches on: "That's a crash bleep; you need to answer it," another specialist registrar

tells me. "Is it really a crash call?" I reply. "Yeah, definitely." I call the number. I'm tense. As it is ringing

the reg smiles and says, "Only joking." "That's not funny." "Yes it is."

The on-call jobs come. Not too bad at the moment. Most of them are low priority until finally I get a call

from another F1 to review a patient with haemoptysis. Thank God for ABCDE. I start oxygen, take a short

history, listen to the chest, check obs, whack a cannula in, and take some bloods. I'm worried that it could

be pneumonia or a pulmonary embolism. Feeling clever I ask the reg on call whether I should start

prophylactic dalteparin.

"No 'cos she's got a brain tumour," he replies.

Feeling a bit stupid because my basic science took a nose dive I check the patient's chest radiogram. It

looks like consolidation. I then go to handover. The time is 9 15 pm.

It's handover and I have not stopped; I'm literally running on one chocolate bar. After mumbling that I'm

only an F1 who knows nothing, the night F2s respond with a genuine empathetic "aww." I retire to bed.

But I still can't stop thinking whether I did the right thing for my patients.

They're still alive the next day (thank God). I am too. My aspirations of becoming an emergency medicine

consultant are also alive. For some strange reason, I can't wait for the next on call.

Ali S Hassan

Having already shadowed the former F1 for a week, I felt gently eased into my first day. This was a great

relief because it could easily have been more like a cold plunge. Most of the staff on the unit already

knew me, and I knew the current patients and why they were there. The downside to the week's

shadowing was that I had seen just how good the previous F1 was, and had been told that I had a hard

act to follow.

I knew to be in for 8 am; I knew there was a meeting at 9 am and where it was; I even knew how the

particular consultant on that day liked his ward rounds to be run. What I didn't seem to know was any

medicine. I felt that during the time between finals and starting my job all knowledge had seeped from my

brain—to be replaced with dysfunctional doubt and fright that I might do or say something so thick that all

confidence in me would be lost instantly.

Page 116: Medscape articles

My First Day as a Doctor

115

I started on the same day as a new senior house officer so I knew more about the running of the unit than

her, which was a double edged sword. Although it made me feel like I knew what was going on, when I

didn't know something there wasn't that experienced senior house officer to fall back on. Luckily she had

worked in the same hospital for the last year and so she knew her way around—this was to prove very

useful when my bleep yelled out "cardiac arrest Foxbury ward." Of course having no idea where Foxbury

ward was or how to get there I would have gone to reception.

All the staff at the unit were very sympathetic to the fact that it was my first job—probably because I

reminded them so often. The intensive care nurses are highly specialised, so they were able to translate

the mass of blinking screens and digits that surrounded each patient and explain where the many tubes

and lines came from and what they were for. This was another great relief as my medical course had

covered very little about intensive care medicine.

My feelings of ineptitude were only made worse when a nurse handed me an electrocardiogram and

asked me what was wrong with it. The longer I looked at the black scribbles against the pink background

the more awkward it became. I don't know why but I seemed to forget all basic principles—I opted for the

easy way out saying I'd like to show it to the senior house officer for her opinion. I kicked myself when she

pointed out the glaringly obvious ST depression. There is just something about the pressure of being

expected to know that seemed to make me crumble—having to assume authority is probably the biggest

leap from medical student to doctor. I imagine everyone experiences it to some degree and the only way

through it is the exposure that an F1 job gives.

Tabassum a Khandker

By the time I started my first day on the paediatric ward, I was confident about my job and abilities; I had

shadowed the departing F1s for one whole day, attended induction and acute illness training, and knew

where the wards were. This confidence was then destroyed when I was told that the previous F1s were

the best the consultants had seen, and had worked to the level of senior house officers.

I began work on Wednesday with two other F1s in paediatrics; we were promptly separated, and one

person went to the neonatal intensive care unit, one stayed on the paediatric ward, and the third covered

paediatric accident and emergency and the ward. The F2 and senior house officers were having

corporate induction and were absent on the first day, so I felt very exposed as we wandered behind the

consultant and registrar looking lost. After introducing ourselves to as many of the nurses as possible, the

pharmacist, and anyone who walked on to the ward, we became more comfortable; at least people

stopped looking suspiciously at us. We were lucky because the medical students were nearing the end of

their attachment, so were doing our jobs better than we were, and helped orientate us on to the ward and

computer systems.

During the shadowing day, a team meeting turned into a pub quiz and this helped the new F1s to bond

with some of the consultants and registrars. I realised that in paediatrics the consultants and registrars

are very involved with patient care and very willing to share their knowledge, however basic my

questions. This impression was reinforced during my first day because everyone patiently explained what

to do, when to do it, and whom to contact if I was still confused. This period of grace doesn't last very long

in other departments apparently, but I've heard that paediatrics is different—I hope that will be the case.

My day involved writing notes on the ward round, ordering tests and checking results of previous tests,

and, of course, the dreaded TTOs. After stumbling through my first one, I have become faster and more

Page 117: Medscape articles

My First Day as a Doctor

116

confident about completing them. For me the golden rule is to look up every drug I prescribe for children

because children are very different from adults, for whom there may be standard doses for certain drugs.

I make no assumptions with children and drugs, although that may change with experience.

By the end of my first day I felt enthusiastic and confident about my rotation in paediatrics; the team were

very supportive, my role in the team seemed clear, and the more involved I was the more I got out of my

day. The fact that this rotation is unbanded without on calls or weekends, may also contribute to my job

satisfaction.

I have been told that these few weeks are the lull before the storm: soon schools will restart and the next

wave of people with swine flu will grace our wards—so who knows how I'll feel in a month or two?

Yasmin Akram

Having made the autocratic decision not to spend any of my last long holidays shadowing for my new job

I was cursing my stupidity as I met my fellow F1 in paediatrics, who sensibly had shadowed. As we

walked through the hospital she knowingly greeted staff while I looked on sheepishly like a child who

hadn't done her homework. With the butterflies running riot in my stomach we knocked on the consultant's

door; he greeted me by my name. At that point I wanted the ground to open up and swallow me whole:

my paranoia was telling me the only way he could have known my name was because I was the

troublemaker who had emailed demanding her rota so she could organise it around her life, even before

she had started work. Had I forgotten that medicine was not a job, but a lifestyle?

But having feared the worst, I found that the consultant was jovial and welcoming. He immediately

recognised me and put me at ease, and I prayed that the rest of the people in the department would be

as nice.

Then on to the wards. As I took histories I was surprised that differential diagnoses were running through

my head; I actually remembered something! Having half expected to be a glorified medical student, I

realised that something had changed; although I was well supervised there was an increased sense of

responsibility, brought on by the faith that patients and colleagues put in me. Additionally, I realised that I

receive automatic respect from medical students simply by virtue of the fact that I am on the other side of

finals to them. Getting them to do jobs for me could easily be a mini power trip, except in the back of my

mind there is the constant reminder that their theory is probably fresher than mine, given my long,

mindless, shadowing-free holiday. Also, not long ago I was in that position myself, and I honestly

appreciate their help.

As the day went on I had moments of panic and disorientation, but also satisfaction at micro

achievements such as successfully taking blood from a wriggling and screaming two year old. I also

slowly picked up the lingo; finding out what is meant by "a capillary" and "a heel," for example. I finished

my shift exhausted, although after votes of confidence from my seniors and generous offerings of

chocolate and biscuits I was looking forward to my next day at work, but only going home and collapsing

on the sofa for a while. It was a long day.

As for the future? Indeed, medicine is not just a job, but a career. For now I'm finding my bearings, taking

it a day at a time.

Page 118: Medscape articles

Writing A Medical Case Report

117

Writing A Medical Case Report Aimun A B Jamjoom; Ali Nikkar-Esfahani; J.E.F. Fitzgerald

Posted: 09/16/2010; Stud BMJ © 2010 BMJ Publishing Group

Abstract and Introduction

Given the unpredictable and challenging nature of medicine, many medical students will have come

across a patient who has not been a textbook case. The patient may have presented in an unusual way,

had a strange new pathology, or reacted to a medical intervention in a manner that has not been seen

before. The publication of these novelties and curiosities as case reports has for many centuries been a

fundamental way of sharing knowledge and conveying medical experience, and throughout history there

have been famous case studies that have helped shape the way we view health and disease ( box 1 ).

Box 1. Famous Case Reports

Multiple Myeloma

William MacIntyre and Henry Bence-Jones contributed greatly to our understanding of multiple myeloma

by recording the effect of this disease on Thomas Alexander McBean. They found that Mr McBean’s urine

was “abound in animal matter.” It is owing to this observation that the protein found in urine of patients

with multiple myeloma is called Bence-Jones protein.[2]

Mental Health Disorders

Sigmund Freud, best known for his psychoanalysis and theories of the unconscious mind, also had a

special interest in recording the case histories of his patients. Many of his case reports helped further our

understanding of a number of mental health disorders, such as obsessive compulsive disorder (rat man),

dissociative disorder (Anna O), phobias (Little Hans) and post-traumatic disorder (wolf man).[3]

Dysphasia

Broca’s area, the speech production centre in the brain, was named after Paul Pierre Broca, a French

physician and anatomist. He made his discovery while studying the brains of dysphasic patients. The first

patient that he examined was nicknamed Tan because of his inability to say any word apart from “Tan,”

which he repeated over and over. On postmortem examination Broca discovered that Tan had a fluid

filled cavity in the left hemisphere caused by syphilis and concluded that this area is responsible for

speech production in humans.[4]

There are those who argue, however, that case reports are increasingly irrelevant in current medical

practice and education.[1] Their obscurity and rarity appeal only to the specialised few, and they add little

to everyday medical practice. Their anecdotal nature lacks the scientific rigour of large, well conducted

studies, and they have therefore fallen down the hierarchical ladder of medical evidence ( box 2 ). Sadly,

many medical journals now refuse to publish case reports.

Box 2. Evidence Needed

Category I—Evidence from at least one properly randomised controlled trial

Category II-1—Evidence from well designed controlled trials without randomisation

Category II-2—Evidence from well designed cohort or case-control analytic studies, preferably

from more than one centre or research group

Category II-3—Evidence from multiple case series with or without intervention or dramatic results

in uncontrolled experiments

Page 119: Medscape articles

Writing A Medical Case Report

118

Category III—Opinions of respected authorities, based on clinical experience, descriptive studies,

and case reports, or reports of expert committees.

Despite their limitations, case reports still have a role to play in furthering medical knowledge and

education. Their ability to flag up novelty means that they are still used to identify emerging pathologies.

Case reports also hold valuable educational worth as brief clinical caveats. Many people find abstract

medical knowledge easier to remember when linked to a patient. Different areas of medical education

such as physiology, pathology, pharmacology, and anatomy are brought together in case reports and

help students and doctors to develop a more holistic approach to patients.

Should I Try and Write a Case Report?

Case reports help students gain a deeper understanding of a medical topic, but they also act as an

excellent introduction to academic writing. Doing a literature review, structuring a manuscript, and

learning how to submit and revise your article are skills worth developing early on. In addition to this, a

published case report is a contribution (though small) to medical science and a way to get your name in

print; it is also something that can be put on your curriculum vitae and help you stand out in future job

applications.

How Do I Go About Finding a Suitable Case to Report?

As a medical student it is difficult to know when a case is unique or interesting enough to warrant

reporting, and it is therefore important to make the most of help from your seniors. If you’re in a clinic or

on the ward and you hear a consultant mention that a case is rare, or that they’ve never seen one like it

before, it’s worth inquiring whether you could write it up as a case report. If you don’t have the good

fortune of coming across any suitable cases it is also worth asking if any of the senior doctors have cases

that you could help them research and write up.

Preparation

When you have found a suitable case it is vital to do a number of tasks before you can start putting pen to

paper.

Literature Review

A literature review on a medical database such as PubMed, Ovid, or Medline can be used to check if

there have been any similar cases; this helps you gauge how rare your case is.

Liaising with Doctors in Charge

It is very important to discuss writing a case report with the senior clinician in charge of the patient’s care.

Not only must you gain their permission, but they will also help to provide guidance and advice. In

addition, it is also worth discussing the case with clinicians of other specialties (such as pathology,

radiology, and microbiology) who have played a role in the patient’s care.

Gaining Consent

Check the journal guidelines carefully. It is extremely important to gain written consent from the patient if

you wish to include pictures or clinical details from which they may be identified. During the consent

Page 120: Medscape articles

Writing A Medical Case Report

119

process you must explain why you wish to share their case with others, the risks and benefits of doing so,

and you must answer any questions they may have. Get senior help for this.

Data Collection

When you have done your literature review, liaised with the senior doctor, and gained consent from the

patient you can start bringing your patient’s case together. This involves finding his or her medical notes,

laboratory results, imaging, and any other relevant material.

How Do I Structure a Medical Case Report?

The basic structure of a medical case report is as follows:

Abstract—This section is read by people trying to decide whether they wish to go on and read

your full case report. It is therefore vital to keep it concise (no more than 150 words) and snappy,

and to encompass all the important aspects of your case, particularly highlighting what this adds

to medical knowledge.

Case report—In this section you are presenting your patient to the reader. This should include a

concise history and any relevant examination and investigation findings. It is important to include

any relevant negatives based upon the potential diagnoses, but do not clutter this section with

unnecessary detail.

Discussion—When you have presented your case to the reader you are expected to put it into

context in the discussion section. Here you give information about the condition or intervention in

question, such as the basic epidemiology, pathophysiology, clinical presentation, investigations,

and treatment. As you paint this picture it is extremely important for you to show how your case

has differed from the norm and how this is contributing to medical understanding.

Where Can I Get My Case Reports Published?

Given the reluctance of many journals to publish case reports these days, where are you to turn when

you wish to share your newly written up experience of an interesting case?

Fortunately, the vastness of cyberspace has allowed for the development of a new breed of medical

journal. A number of new online journals such as BMJ Case Reports (www.casereports.bmj.com), Cases

Journal(www.casesjournal.com), the Journal of Medical Case

Reports (www.jmedicalcasereports.com), Radiology Case

Reports (www.radiology.casereports.net/index.php/rcr), and the Journal of Dermatological Case

Reports (www.jdcr.eu) allow the publication and dissemination of notable case reports. These allow

doctors from all over the world to share their experiences of new and interesting cases. Although still in

their infancy, these journals have the potential to act as large case banks that allow doctors to search for

cases similar to ones that may be puzzling them, to help guide their management.

Page 121: Medscape articles

PDAs and Smartphones: Clinical Tools for Physicians

120

PDAs and Smartphones: Clinical Tools for Physicians Andrew E. Craig, MSN, FNP-C

Posted: 10/09/2009

Introduction

Today's cell phone technology has advanced to the point that a cell phone can do much more than just

place calls. Along with the personal digital assistant (PDA), these devices can now play a major role in

making clinical information portable and readily available.

Robert Jones, MD, is a family practice physician with a subspecialty in sports medicine practicing in

Charlotte, North Carolina. He serves as the medical director for the student health center of a large public

university, he's the team doctor for the athletic department at that same university, and he's on the faculty

of a family practice residency program. Filling all these roles keeps Dr. Jones constantly on the move, and

he relies on handheld technology to keep him connected and informed in several ways.

Smartphones and PDAs can be extremely helpful to physicians caring for patients. Whether in your office,

a clinic, or the hospital, the ability to check drugs and dosages instantly at the point of care results in

better patient care and can sometimes be lifesaving.

In testimony before the US Senate in 2000 regarding adverse drug events, the President of the Institute

for Safe Medication Practices quoted a 1995 study by Leape and colleagues in JAMA that reported that

over 40% of adverse drug events can be tied to a lack of critical patient information and drug information

at the time of prescribing, dispensing, and administration of medications.[1] Smartphones and PDAs can

provide this information to physicians instantly at the point of care.

Smartphone or PDA?

Both smartphones and PDAs allow you to perform many functions that were once reserved for the

desktop computer. Simply stated, a smartphone is a cell phone that has the ability to add additional third-

party software. In addition to running medical software programs that can help you, smartphones can play

videos, take pictures, and play music.

A BlackBerry is a particular brand of smartphone that is very popular. It made its mark providing

outstanding corporate email support to business users, yet it is very popular with noncorporate users as

well.

In contrast, a PDA is a mini-personal computer without cell phone capability. As a clinical tool, either a

PDA or a smartphone will accomplish what you need and will help you throughout your workday. With

either type of device, you can access clinical reference programs that give you instant information when

you're with a patient.

If you work a lot in the hospital and plan to access the Internet regularly, you may be better off with a

smartphone. One advantage to having a smartphone is that you can access the Internet from wherever

you have a good cellular signal. In contrast, a PDA requires a Wi-Fi connection to do this. Even if Wi-Fi is

available, you may not be permitted to access the hospital's Wi-Fi network with a personally owned

device; that could prevent you from using the Web-based programs that you want to use. Or you may be

Page 122: Medscape articles

PDAs and Smartphones: Clinical Tools for Physicians

121

able to convince the hospital's IT department to provide you with a handheld device for this purpose,

particularly if you spend a lot of time there.

At your own practice, setting up Wi-Fi access for your device is a fairly simple matter for your IT person to

do. Make sure you employ security features such as data encryption and strong passwords to ensure the

safety and security of your patient data. Here's an additional tip for better security: have your IT technician

configure your office network's firewall to only allow wireless traffic to pass to and from your particular

device, as opposed to any device that knows the password. Every computing device has a unique code

called a "MAC address" assigned that is not changeable. By configuring your firewall to only allow traffic

from a device with your MAC address, you add an extra layer of protection should your password ever get

compromised. Even if a rogue user attempts to crack your system using your stolen password and their

own PDA or laptop, the firewall will refuse access if the MAC address doesn't match up.

How to Choose?

Several factors will influence the type of device you choose. The first is the operating system (OS) that

runs the device. There are 4 major OSs: Palm, Windows Mobile, BlackBerry, and iPhone. While most

major medical software is available for any of these platforms, there are a few that may only be available

for some of them.

Most Windows Mobile devices available now are smartphones, although some Windows Mobile PDAs still

exist. Both Windows Mobile smartphones and Windows Mobile PDAs use the same OS. There are still

Palm PDAs in the marketplace, though this once-dominant brand is not nearly as prevalent as it used to

be.

The Palm Pre, their newest smartphone, runs on a new OS that is Web-based. Additionally, if you have

an older Palm app that you can't bear to part with, you can buy an application ("Classic") that allows the

Pre to emulate an older Palm OS device and run your legacy app. At this time, the Pre does not have

much medical software written for it, though this will likely change in time.

Sources for obtaining applications for your device may be limited as well. Applications for the iPhone are

only available at Apple's iTunes store, and all programs must be approved by Apple before becoming

available. Programs for Windows Mobile smartphones, PDAs, and BlackBerries are available from a

plethora of online sources. For all these devices, some applications are free and some have a cost. This

may be a one-time charge or a recurring subscription fee.

Dr. Jones currently uses 2 handheld devices: a Hewlett-Packard PDA running the Windows Mobile

operating system and a BlackBerry smartphone. While he acknowledges that some favor using one

device for everything, this particular combination best suits his needs at present. His PDA was issued to

him by the hospital system where he teaches family practice residents; it has been set up by their

computer department so that he can directly access the hospital's electronic medical record (EMR). Dr.

Jones says, "This is a tremendous help for me because I can access patients' lab results and chart data

in real time." He uses it a lot when performing inpatient rounding with the residents. His PDA is set up so

that whenever he walks into the hospital, the device synchronizes with the hospital's EMR, allowing him to

pull up the latest data for his patients on demand. "Because of this, I no longer have to ask who a

patient's nurse is, find them, and then have them run a printout of, for example, last night's labs. I can just

pull it up on my PDA as soon as I'm in the hospital." His device allows him to view data but not enter or

Page 123: Medscape articles

PDAs and Smartphones: Clinical Tools for Physicians

122

change it. It is already set up with wireless security features like data encryption and password protection

to protect his patients' data, and the device can be wiped clean remotely if it ever gets lost or stolen.

In the past, accessing an EMR through a PDA or smartphone was not possible. However, more vendors

are now making this type of access available. Allscripts, eClinicalworks, and Medent allow you to access

their EMRs through a handheld device; you can review medical summaries, check appointments, and

perform other patient-related tasks.

Dr. Jones' other handheld device is a BlackBerry smartphone. He doesn't use clinical applications on it

because they are already loaded on his PDA, but he finds the BlackBerry invaluable for accessing his

email and staying in touch via phone when he's on the go. With all his different roles, Dr. Jones spends a

lot of time in transit. With the BlackBerry, he can be reached by phone and can also receive emails from

both his hospital and university email accounts no matter where he is. "I find this valuable because I can

get my email in real time, which helps me to prioritize which issues need to be addressed first. Often, my

email arrives on my BlackBerry before my PC!"

A couple of years ago, only a few medically themed applications were available for the BlackBerry;

however, the number of available programs has grown steadily. Now there are a reasonable number of

medical applications available for this platform.

While most BlackBerries are similar enough to each other that applications written for that OS work fine

on any model BlackBerry, the new BlackBerry Storm uses a touch screen and therefore is a bit different.

Some apps written for the BlackBerry may have problems running on the Storm. Be sure to check before

you buy.

Choose Your Carrier

Buying a smartphone means you'll have to arrange for service with a cellular carrier. While you can

theoretically buy some handheld devices outright (ie, without affiliation to any one carrier), this is usually

cost-prohibitive. Carriers offer substantial discounts on the cost of the device when you sign up for service

with them at the time of purchase; you typically have to sign a service contract for 2 years.

Unfortunately, not all phones are available from all carriers. For example, if you want the iPhone, you'll

have to go with AT&T. If you want a BlackBerry Storm, you'll have to choose Verizon. For the Palm Pre,

you'll have to sign up with Sprint. This is because the device makers sign exclusive contracts with one

particular carrier.

It's also important to be aware that different handhelds use different types of cellular technology to place

and receive calls. Your device's cellular technology must match the type of technology used by your

carrier. For example, a smartphone that uses GSM (General Service Mobile) technology will not work on

another cellular carrier's network if that network uses CDMA (Carrier Detect Multiple Access) technology,

even if the smartphone is "unlocked."

I also recommend that you research a carrier's reliability and customer satisfaction in your area before

signing on the dotted line. Make sure the carrier that you're thinking of signing up with gives good

coverage where you live, work, and travel.

Page 124: Medscape articles

PDAs and Smartphones: Clinical Tools for Physicians

123

Clinical Software

Once you've purchased your handheld, you'll want to either download or access programs and

applications that can help you in your practice. Software available includes drug references, clinical

references, medical calculators, and others.

Drug Reference: One of the most popular types of handheld software, this allows you to look up drugs

on your handheld device. Dosing, side effects, drug interactions, and drug prices can be reviewed. Some

programs even offer color pictures of pills and allow you to filter drug choices by a particular insurance

formulary.

Say you're talking with a patient, and he wants to know if the new medication he's being discharged with

is covered under his insurance or whether it will interact with his blood thinner; with a handheld computer,

you can answer that question in a few seconds without leaving the bedside.

Some of the many programs available include:

Lexi-Comp, which offers a number of products for PDAs or smartphones. Products within the suite

include Lexi-Interact, a drug interactions database. Other products provide information on drug dosing

and dosage forms, indications/contraindications, a toxicology database, and a medical dictionary.

The Sanford Guides, which provide guidelines and recommendations for treatment of infectious

diseases. Both The Sanford Guide to Antimicrobial Therapy and The Sanford Guide to HIV/AIDS

Therapy are available.

Epocrates, which includes drug monographs, a drug interaction checker, formulary information from

many health plans, CME, and medical news. Additional modules available for a cost include a disease

review database, symptom checker, and an herbal medicine reference.

Johns Hopkins Antibiotic Guide (ABX Guide), which contains information on diagnosis, drugs,

pathogens, management, and vaccines. Helps confirm diagnoses and helps identify the right

antimicrobial to prescribe.

Medscape for the iPhone, which contains over 6000 generics and brands, including herbals and

supplements; interaction checker; and continually updated drug database including notifications of

important prescribing changes. It can check for drug-drug interactions, or interactions between drugs,

herbals, and supplements, allowing for inputting up to 30 entries at once.

General clinical references also abound.

The 5-Minute Clinical Consult contains comprehensive and structured information covering over 700

medical conditions. Key terms are hyperlinked; so if I'm reading about diabetes and the article

mentions hypertension, I can jump directly to the article that discusses hypertension by tapping my finger

on the word.

Harrison's Principles of Internal Medicine is one of the best-selling medical textbooks worldwide. You

can buy this in electronic format from a variety of software vendors like Unbound

Medicine and Skyscape or access it online atwww.accessmedicine.com.

Page 125: Medscape articles

PDAs and Smartphones: Clinical Tools for Physicians

124

UpToDate is an evidence-based, peer-reviewed information resource, used by clinicians, medical

students, and academic medical centers for answers to their clinical questions.

Medical calculators are also quite helpful, and there are many available. These make medical

calculations a breeze; you can calculate everything from drip rates to BMI, predicted peak flow to the

Parkland formula, pregnancy due dates, and much more. You may find these as standalone programs

(some, like MedCalc, are even free!) or they may be packaged along with drug and clinical references

into one big clinical program suite.

CME -- Continuing education is offered on handheld devices as well. The Prescriber's Letter has a PDA

version, and some of the clinical suites and software vendors offer CME as either part of the suite or an

add-on. Medscape for iPhone and iPod touch enables you to take CME courses on your handheld, and it

keeps track of your CME courses taken.

In addition to accessing EMR, Dr. Jones uses several clinical tools on his PDA. He's a fan of 5 Minute

Clinical Consult(5MCC) for primary care and is actively involved in creating the second edition of 5MCC

for sports medicine, which will be the first edition of that title to be offered in an electronic version. While

Dr. Jones has many years of experience, he says having a clinical reference available at his fingertips is

extremely helpful. "There is so much in medicine to know, it's challenging to be able to recall in an instant

everything you've learned over the years. This helps," he says. "Having the information available in

bullets is fast and easy to access." He humorously refers to his PDA as his "peripheral brain."

Dr. Jones uses the Epocrates and Tarascon drug references. He finds this particularly helpful for

answering drug questions from a patient regarding if a medicine comes as a tablet or a capsule, or

whether it's available in a particular strength.

Dr. Jones gives this advice to physicians considering acquiring a handheld device for clinical use: "Yes,

get one. Do it! It's timesaving, it saves me trips to the bookshelf, and all the information I need is only a

few clicks away."

Page 126: Medscape articles

PDAs and Smartphones: Clinical Tools for Physicians

125

YAZID JIBREL

I take the time to collect these articles from the Medscape blogs from residents and medstudents that I

felt useful, informative, supportive and sharable.